面白い問題おしえて〜な 五問目

1 名前:132人目の素数さん投稿日:03/04/06 01:49
[1]面白い問題教えて
http://cheese.2ch.net/math/kako/970/970737952.html
[2]面白い問題教えて 第2版
http://natto.2ch.net/math/kako/1004/10048/1004839697.html
[3]面白い問題おしえてーな
http://science.2ch.net/math/kako/1026/10262/1026218280.html
[4]面白い問題おしえて〜な 四問目
http://science.2ch.net/test/read.cgi/math/1044116042/

2 名前:1投稿日:03/04/06 01:51
やべえ、本文書くの忘れてた

面白い問題、教えてください


っと。

3 名前:詩人数学者 ◆JDb6NSLgcY 投稿日:03/04/06 01:54
>>1-2
乙ー。これからもよろしくお願いします。

4 名前:(¬_¬)y―ξ~~ ◆7niWItYnQM 投稿日:03/04/06 01:55
>>1-2
乙ですー。

5 名前:反芻学者投稿日:03/04/06 01:56


6 名前:132人目の素数さん投稿日:03/04/06 04:21
 でつ < 前スレの未解決問題を貼っておきます
  ひ
 ∞

【問題】 整数pについて、次式の和を求めよ。 
 納n=1 to ∞]納k=1 to p-1](1/(pn-k)-1/pn)

7 名前:132人目の素数さん投稿日:03/04/06 04:28
  /ヘ;;;;;  
  ';=r=‐リ  
  ヽ二/       n  
 ̄     \    ( E) >>6 グッジョブ!!
フ     /ヽ ヽ_//

8 名前:132人目の素数さん投稿日:03/04/06 15:00
http://www.geocities.co.jp/AnimeComic-Ink/5125/trisection/trisection.htm
前スレに出てた実用的な角の三等分法だけど、これ合ってるよね?

9 名前:132人目の素数さん投稿日:03/04/06 15:55
>>8
作図に致命的欠陥がある。

10 名前:132人目の素数さん投稿日:03/04/06 16:02
>>8
「実用的な」三等分なら問題は無い。
しかし、このページの作者はこんな文を書いている。

>週末は三等分三昧というのも悪くないでしょう。もう一工夫で完全な幾
>何三等分法になります。貴方の余生を完全な幾何三等分の追求に賭して
>みてはいかがでしょうか?!

メモリのついていない定規とコンパスで任意の角を三等分する方法は存在
しない、ということが100年以上前に証明されているのに・・・
このページの作者が哀れだ・・・全世界に向かって恥をさらすとは・・・


11 名前:132人目の素数さん投稿日:03/04/06 16:04
>>10
「全世界に恥をさらす」に訂正。スマソ。
何てこった・・・

12 名前:132人目の素数さん投稿日:03/04/06 16:08
1〜6の目がある普通のサイコロを振ることによって、5人の中から1人を不公平無く選び出す方法は、
サイコロを何度振ってもよいときは存在する。
1〜5の目を各人に割り当てて6がでたら振りなおすようにすればよい。しかし運悪く6ばかり出てしまうこともある。
では1〜6の目がある普通のサイコロを有限回数振ることによって、5人の中から1人を不公平無く選び出す方法は存在するか。
存在するならばそのなかで最短手数の例を1つ記せ。存在しないならばそれを証明せよ。

13 名前:132人目の素数さん投稿日:03/04/06 16:12
>>12
「数学の部屋」に類似問題あり。

14 名前:12投稿日:03/04/06 16:46
>>13
「数学の部屋」は見ましたが、具体的にどの問題でしょう?

15 名前:132人目の素数さん投稿日:03/04/06 21:52
>>14
これのことじゃないの?
http://web2.incl.ne.jp/yaoki/musakui.htm
3回と有限回のちがいはあるけど。6^nが5でわりきれないんだから実質おんなじだね。

16 名前:12投稿日:03/04/07 01:25
>>15
サンクス。自分でふと思いついた問題でまったく解き方がわからなかったのですが
ようやく理解できました。

17 名前:132人目の素数さん投稿日:03/04/08 00:00
教えてくさいage

18 名前:132人目の素数さん投稿日:03/04/08 05:43
論理パズル系でなんか目新しいヤシないかな?

19 名前:あぼーん投稿日:あぼーん
あぼーん

20 名前:132人目の素数さん投稿日:03/04/08 06:16
あるところに大発明家がいました。
彼はYesかNoで答えられる質問なら
どんなことにも答えてしまうロボットを設計しました。
しかし、彼は貧しかったので設計図をA社とB社に売り、それぞれに作ってもらうことにしました。

A社はYesなら青いランプ、Noなら赤いランプが点くようにして作りました。
B社はYesなら赤いランプ、Noなら青いランプが点くようにして作りました。
それぞれのロボットは見た目はまったく同じで、区別が付きません。


あなたは中古でこのYes・Noロボットを買うことにしました。
店に行ってみると、現在3台の在庫があるようです。
店員の話によると、3台のうち1台は壊れていて、まったくでたらめな答えを返してきます。
(正解するかもしれないし、間違えるかもしれない)
そして、その3台ともA社のものかB社のものかはわかりません。
店員は意地悪で、たった1回だけ、それも1台に対して
何か質問をしてよいといいました。

あなたは何とかして壊れていないロボットを買わなければいけません
どんな質問をすればいいでしょう?

21 名前:132人目の素数さん投稿日:03/04/08 06:23
この手の問題、大好きだけど、結局わからずに答えを見てしまうんだよね
コツとかあるのかな?

22 名前:132人目の素数さん投稿日:03/04/08 08:10
>>20
「この質問の答えはNoですか?」

23 名前:132人目の素数さん投稿日:03/04/08 08:58
>>20
1+1=2ですか?で良さげ

24 名前:132人目の素数さん投稿日:03/04/08 09:46
>>23
壊れた奴を質問相手に選んでいたらどうすんの?

25 名前:132人目の素数さん投稿日:03/04/08 10:03
よく分からんが、例の手でいくのかな?
右側のヤツに向かって、「真ん中にこう尋ねたらyesというか?」って聞くタイプかな?

26 名前:132人目の素数さん投稿日:03/04/08 11:23
判断不能の場合は、ランプは点かないと考えていいんだよね?
もちろん正常なロボットでの話。

あと壊れたロボットは、デタラメではあるが、同じ質問を
繰り返した場合は常に同じ答えを返すものとしていいのかな。

27 名前:132人目の素数さん投稿日:03/04/08 12:01
>>21
質問が二回でいいなら、まずは壊れたロボットを排除して判定可能な
方法を考えておく(質問内容は変えられるように)。そうしておくと一回目は
ある一台に対して他の二台に関する質問をすればうまくいく、というのが
セオリーだったかと。

28 名前:27投稿日:03/04/08 12:42
あ、問題読み間違えてた。
確かに「故障してないのを選ぶ」だけなら一回でいいね。

「(向うの右側の)ロボットが壊れているか? と尋ねられたら
赤いランプを点けるか?」

29 名前:20投稿日:03/04/08 15:40
回答不能の質問に対する動向がちょっと曖昧だったね。
ごめん。

>>28
正解。
赤いランプがついたら左を、青いランプがついたら右を取ればいいんだね。

30 名前:20投稿日:03/04/08 17:33
第2問。

【正常なロボット】と【壊れたロボット】を次のように定義する。

【正常なロボット】
YESが真、NOが偽となる質問では青、NOが真、YESが偽となる質問では赤のランプがつく。
それ以外の質問をされると自爆する。

【壊れたロボット】
上記の正常なロボットの思考回路の「青」と「赤」が質問の都度ランダムで入れ替わる。
これは各質問の直前に設定される。
またどちらに設定されたのかは他のロボットにも瞬時に伝わる(ロボットに知らない事は無い)。


今、正常なロボットと壊れたロボットが1台ずつある。
1回だけ1台に対してだけ質問することができる。
正常なロボットを手に入れるためにはどのように質問すればよいか?

31 名前:あぼーん投稿日:あぼーん
あぼーん

32 名前:132人目の素数さん投稿日:03/04/08 22:10
age

33 名前:132人目の素数さん投稿日:03/04/09 01:25
>>6
log p

34 名前:132人目の素数さん投稿日:03/04/09 05:42
あげ

35 名前:あぼーん投稿日:03/04/09 05:53
あぼーん

36 名前:20投稿日:03/04/09 14:58
誰も答えてくれない・・・
(´・ω・`) ショボーン

37 名前:132人目の素数さん投稿日:03/04/09 15:29
あぼーんされたのか?
告白したのか?

38 名前:132人目の素数さん投稿日:03/04/09 18:08
>>33
正解っす。
 
−解答例−
1/m=∫[0,1]t^(m-1)dtとルベーグの収束定理より
 納n=1 to ∞]納k=1 to p-1](1/(pn-k)-1/pn)
 =∫[0,1]納n=1 to ∞]納k=1 to p-1](t^(pn-k-1)-t^(pn-1))dt
 =∫[0,1](1+2t+3t^2+・・・+t^(p-2))/(1+t+t^2+・・・+t^(p-1))dt
 =∫[0,1](log(1+t+t^2+・・・+t^(p-1)))'dt
 =[log(1+t+t^2+・・・+t^(p-1))]_0^1
 =logp
 
おそまつさまですた。

39 名前:132人目の素数さん投稿日:03/04/09 19:12
2つの袋A、Bが用意されてます。
どっちかの袋にはどっちかの袋の二倍の金額が入っているらしい。

さて、Aの袋をあけると10000円入っていた。
で、このままこの10000円を持ち帰ってもいいんだけど、
Bの袋と交換することもできる。(もちろんBの金額はまだわからない)

さぁ、取り替えるべきでしょうか?


期待値を考えてみる。
Bに入ってる金額は20000円かもしくは5000円。
その確率はともに1/2だから、Bに取り替えることで得られる金額の期待値は、

20000×1/2 + 5000×1/2 = 12500円> 10000円

じゃあ常にこれは取り替えた方が得ってことですか??
それでいいんですかねぇ?
なんか、 = にならないとおかしな気がしません?
どうでしょう?

40 名前:132人目の素数さん投稿日:03/04/09 19:18
√(20000×5000)=10000
20000×(1/2) + 0×(1/2)=10000

41 名前:132人目の素数さん投稿日:03/04/09 19:37
>>38
納n=1 to ∞]納k=1 to p-1](t^(pn-k-1)-t^(pn-1))
=(1+2t+3t^2+・・・+t^(p-2))/(1+t+t^2+・・・+t^(p-1))

ここの式変形が分かりません。 (´・ω・`)ショボーン

42 名前:27投稿日:03/04/09 19:49
>>30
「お前は正常なロボットかまたはYESが真の時赤いランプを点けるか?」

43 名前:27投稿日:03/04/09 19:53
いや、なんか違うね…

44 名前:27投稿日:03/04/09 20:26
いや、やっぱりokな気がする。要するに

 正常ロボット
 異常で正常と同じ
 異常で正常と逆

という三パターンで一番上だけ違う結果になればいいんだよね。

45 名前:20投稿日:03/04/09 22:08
>>42
good job

46 名前:132人目の素数さん投稿日:03/04/10 21:45
>>41左辺と右辺をそれぞれ変形していったら、確かに同じ式になったが…。
しかし、右辺の形になることを知らずに、この問題は解けたのだろうか?
それとも、左辺から普通に変形していって右辺の形を得るのでしょうか?
教えてたも  。・゚・(ノД‘)・゚・。

47 名前:あぼーん投稿日:あぼーん
あぼーん

48 名前:132人目の素数さん投稿日:03/04/10 23:07
重複もろあり ?

49 名前:132人目の素数さん投稿日:03/04/10 23:30
http://science.2ch.net/test/read.cgi/math/1049683075/778

50 名前:132人目の素数さん投稿日:03/04/11 18:14
>>46
正しいことがわかったんならそれでいいと思うんだけど。
最初この問題別スレででたときオレもこの変形なかなか思いつかなくててこずった。
式でかくとむずくみえるけど
納n=1 to ∞]納k=1 to p-1](t^(pn-k-1)-t^(pn-1))
=納k=1 to p-1](t^(p-k-1)-t^(p-1))/(1-t^p)
ここまではなんてこた−ない。くくって
=納k=1 to p-1]t^(p-k-1)(1-t^k)/(1-t^p)
ここも普通にできる。分子分母1-tでわって
=納k=1 to p-1]t^(p-k-1)(1+t+・・・t^(k-1))/(1+t+・・・+t^(p-1))
=納k=1 to p-1](t^(p-k-1)+t+・・・t^(p-2))/(1+t+・・・+t^(p-1))
で分子は狽つかわずかけば
1+t+t^2+・・・+t^(p-2)
+t+t^2+・・・+t^(p-2)
・・・
+t^(p-2)
=1+2t+3t^2+・・・+(p-1)t^(p-1)
=分母の微分
までくればああ、対数微分だなと、面白いなと、出題してみもそと。

51 名前:132人目の素数さん投稿日:03/04/12 08:26


52 名前:132人目の素数さん投稿日:03/04/12 10:30
>>50
何度見ても、いい問題ですね。
上の上です。

53 名前:AAAd投稿日:03/04/12 13:22
でたらめって・・・






例えば、あたってるかはずれか分からないってこと?

54 名前:132人目の素数さん投稿日:03/04/12 14:21
>>53
本屋に行って国語事典でも開いて(ry

55 名前:132人目の素数さん投稿日:03/04/12 21:56
わざわざ本屋行かなくても
ttp://dictionary.goo.ne.jp/search.php?MT=%BD%D0%C3%AD%CC%DC&kind=jn&mode=0&type=stick

56 名前:132人目の素数さん投稿日:03/04/13 05:31
いつまで待たせんだ!
さっさと教えやがれ!
ヽ(`д´)ノ ガロァ!

57 名前:mathmania ◆uvIGneQQBs 投稿日:03/04/15 15:37
>> 30
「あなたは現在のランプの設定で、『あなたは正常なロボットか?』と訊かれたら青のランプを点けますか?」
で青のランプが点いたら訊いたほうのロボットを、赤のランプが点いたらもう一つのロボットを選ぶ。

58 名前:132人目の素数さん投稿日:03/04/15 19:18
暇な人用。賢い香具師は流せ。簡単だから
ココに10枚のカードが有り、表に1から10まで番号が打ってある。
この10枚は裏に○か×が打ってあり、以下のことが分かっている。
「連番で○○と来れば、次は×。」
「×より3大きい数字のカードは×。」
「×が連番で奇数枚並ぶことはない。」
「3番か6番の少なくとも一枚は×。」
「5か8の少なくとも一方も×。」
「7番が×なら、1番も×。」
「○より×のほうが多いが、○は4枚ではない。」
これらから、カードの内訳を答えよ。 15秒で解く香具師も居そうだ・・・

59 名前:あぼーん投稿日:あぼーん
あぼーん

60 名前:27投稿日:03/04/15 19:31
貯金箱の中に10円玉と5円玉が沢山入っています。金額は今日にちなんで15,415円です。

貯金箱から適当に硬貨を二枚取り出し、合計が15円だったら箱の中に10円玉を、合計が10円または20円だったら箱の中に5円玉を入れます。

この操作を繰り返して最後に残るのは5円玉?10円玉?

61 名前:あぼーん投稿日:あぼーん
あぼーん

62 名前:60投稿日:03/04/15 20:18
あ、出題ミス。箱の中に入れる硬貨は逆にしてください。

63 名前:132人目の素数さん投稿日:03/04/15 20:22
「箱」と「貯金箱」は別のものですよね?
「箱」に入れなかったほうは貯金箱に戻す、ということですか?

64 名前:132人目の素数さん投稿日:03/04/15 20:24
5円玉

65 名前:132人目の素数さん投稿日:03/04/15 20:41
プリプリサプリ

66 名前:60投稿日:03/04/15 20:43
>>63
あ、いや一緒です
>>64
正解

67 名前:132人目の素数さん投稿日:03/04/15 21:06
>>58
直感で全部×。
で、合ってた。15秒以内。

唯一解か検証。

6,8,9は確実に×
・10が○だとすると、1,4,7も○
 ○は3枚以下のはずなので不可
・10が×だとすると、
 8〜10が×なので7は×(×は奇数枚並ばない)
  7が×だと1も×
  1が×だと4も×
 6〜10が×なので5は×
 4〜10が×なので3は×
 2は○でも×でも良い・・・・・・あれ?(;´Д`)アッテマスカ?
 

68 名前:132人目の素数さん投稿日:03/04/15 21:30
・・・
・・・
・・・

直線4本で9個の点全てを結べ。

…超基本問題スマソ

69 名前:132人目の素数さん投稿日:03/04/15 21:32
-----+-
-----+-
-----+-

70 名前:68投稿日:03/04/15 21:37
>>68 「一筆書きで」という条件忘れてますた。

71 名前:詩人数学者 ◆JDb6NSLgcY 投稿日:03/04/15 21:44
>>68は既出問題ですね。たしか線分の長さを限定していないところがミソ。

72 名前:132人目の素数さん投稿日:03/04/15 21:56
似たような問題でこんなのがあった。
(あんまり似てないかも知らんけど)

一筆書き、直線で、
すべての@と*を交互に一回ずつ通るように

   @ *

@ * @ *

* @ * @

   * @

ttp://www.mathpuzzle.com/
より抜粋

73 名前:132人目の素数さん投稿日:03/04/15 22:02
>>72
直線何本?


74 名前:132人目の素数さん投稿日:03/04/15 22:06
あ、ごめんなさい。肝心なこと書いてなかったっすね。
えっと、5本です。4回曲がっていいということ。

75 名前:132人目の素数さん投稿日:03/04/15 22:20
リンク先に答えあった。
http://www.mathpuzzle.com/Star_Spiral.gif


まだの人見ちゃ駄目よ。

76 名前:132人目の素数さん投稿日:03/04/16 02:29
面白い問題キボンにゅ

77 名前:詩人数学者 ◆JDb6NSLgcY 投稿日:03/04/16 12:43
AB=3、AC=4、BC=5である三角形ABCの辺AB上に任意に点Pを置く。
点Aを中心に半径APの円を描き、この円と三角形の共通部分の
面積をSとすると、Sの期待値を求めよ。

今しがた思いついた問題。

78 名前:132人目の素数さん投稿日:03/04/16 13:07
>>77
「任意が曲者」という例のアレ?

79 名前:132人目の素数さん投稿日:03/04/16 16:10
―――――――――――― ←のように幅1cmの罫線が引いてあるノートの上に
――――――――――――  長さ2cmのシャーペンの芯を落としたときに、
――――――――――――  芯が1本の罫線の上にかかる確率を求めよ。
――――――――――――  ただし、罫線の幅、芯の太さは0としてよい。
――――――――――――



80 名前:132人目の素数さん投稿日:03/04/16 16:12
>>78
例のアレとは?

81 名前:78投稿日:03/04/16 16:37
Pをどのように「任意に」配置するかによってSの期待値も(当然)変化するっていうアレ。

82 名前:132人目の素数さん投稿日:03/04/16 18:10
>>79
1/3でいいのかえ?

83 名前:132人目の素数さん投稿日:03/04/16 18:24
>>79
ビュフォンの針の問題に似てるようで違う・・・

84 名前:132人目の素数さん投稿日:03/04/16 20:46
>>79
垂直に落ちれば必ず接するし、それから、60度ずつ左右に傾けて落ちた状態も大丈夫。
上より、三分の二以上は確実。
残りは上下のスライドを含めた考慮をしないといけない。

85 名前:82投稿日:03/04/16 20:56
少なくとも1本なのか・・・・?

86 名前:132人目の素数さん投稿日:03/04/16 20:57
そんな事、知らん・・・

87 名前:79投稿日:03/04/16 21:00
1本だけ



88 名前:132人目の素数さん投稿日:03/04/16 22:04
サインとコサインを掛け合わせるの?

89 名前:132人目の素数さん投稿日:03/04/16 22:44
既出ならスマンが、
1つの立方体を、9つの立方体に切り分ける時、
最低6回は、切る必要があることを証明せよ。



90 名前:132人目の素数さん投稿日:03/04/16 23:02
5回で切れるような気がするのだが、
問題の意味を取り違えてるのかな。

91 名前:132人目の素数さん投稿日:03/04/16 23:05
>>89
ひとつの立方体を九つの立方体に切り分けることは可能なのか?

92 名前:89投稿日:03/04/16 23:05
>>90
あっ、すみません。
×9つ
○27個
でした。


93 名前:132人目の素数さん投稿日:03/04/16 23:17
真ん中にできる立方体を考えればほぼ自明

94 名前:132人目の素数さん投稿日:03/04/17 02:45
次の場合についてサイコロは何種類あるか考えよ。
・刻印の向きを区別しない場合
・刻印の向きを区別する場合
ただし裏と表の目を足したら7になるとする。
また実際に存在するサイコロが何種類なのかを
2chで情報交換して調査するための簡便な表示方法を提案せよ。

95 名前:132人目の素数さん投稿日:03/04/17 02:57
>>94
1,2,3の面を同時に見た場合、

 1     1
2 3   3 2

この二つとも考えなきゃ駄目?

確か実際には左しか存在しなかった気がするけど。

96 名前:132人目の素数さん投稿日:03/04/17 03:02
>>95
まあ、両方勘定して下さい。実際には左型のみらしいのだけど、
可能性がその2種類に分類出来ることを示すのも問題に含まれるものとします。

97 名前:132人目の素数さん投稿日:03/04/17 03:14
>>96
>可能性がその2種類に分類出来ることを示すのも問題に含まれるものとします。

これは自明でいいっしょ。これは本題じゃないかと。

98 名前:あぼーん投稿日:あぼーん
あぼーん

99 名前:132人目の素数さん投稿日:03/04/17 03:23
>>97
そうですね。それは示さなくて構いません。

100 名前:97投稿日:03/04/17 03:24
スマン。一応本題だったね。
でもめんどいから証明は略。

>・刻印の向きを区別しない場合

>>95の2種類。

101 名前:132人目の素数さん投稿日:03/04/17 03:26
>>100
> スマン。一応本題だったね。
> でもめんどいから証明は略。

分かれば当たり前ですが、書くのはめんどうですよね。
もちろん2種類で正解です。

102 名前:97投稿日:03/04/17 03:26
>・刻印の向きを区別する場合

こっちがむずいな。
もう少し時間くれ。

103 名前:132人目の素数さん投稿日:03/04/17 03:27
>>102
んじゃそろそろ寝ますんで明日の夜にでも…

104 名前:97投稿日:03/04/17 03:28
>>103
おやすみ〜

105 名前:97投稿日:03/04/17 03:46
簡単だったわ。

2,3,6は互いに隣り合う。

 2
3 6

の場合。2^3=8通り。

 2
6 3

の場合。2^3=8通り。

よって全16通り。

106 名前:97投稿日:03/04/17 04:06
情報交換の方法。

2,3,6の面が全部見えるように置く。その3つの面が交わる頂点をPとおく。


● 2                 2
  3 6  だったら「0」     6 3 だったら「1」

●2の刻印を通る直線が、Pと交わったら「0」  交わらなかったら「1」

●3の刻印を通る直線が、Pと交わったら「0」  交わらなかったら「1」

●6の刻印のうち3点を通る直線が、2の面と交わったら「0」  交わらなかったら「1」


以上の四つの数字を調べ、この順に組として表す。

ex.(1,0,0,1)

107 名前:132人目の素数さん投稿日:03/04/17 05:50
おはようございます。>>105 は見てのとおり正解です。
>>106 を2進法と見て(1001 なら 9)、16進法 0〜F で表記するとさらにポータブル。
サイコロを見かけたら報告してみると面白いかも。

108 名前:山崎渉投稿日:03/04/17 08:46
(^^)

109 名前:あげ投稿日:03/04/17 17:02
√(2+√(2+√(2+…)))=?

110 名前:あげ投稿日:03/04/17 17:04
age失敗スマン

111 名前:132人目の素数さん投稿日:03/04/17 18:29
>109
2cos(0)=2
この問題知らずに他スレで書き込んでいた。。。

√(2+√(2+……+√(2+2 cos(x) )…)) =2cos(x/2^n)
http://science.2ch.net/test/read.cgi/math/1013175188/119

112 名前:あげ投稿日:03/04/17 18:48
おおう、なんという偶然。

ちなみにそのような公式を知らなくても
この問題だけ解くなら以下のようにすればできます。

√(2+√(2+√(2+…)))=Xとおく、
両辺2乗して
2+√(2+√(2+…))=X^2

よって2+X=X^2
これを解いてX=2,-1
Xは正なのでX=2

(解答の一行目で、
√(2+√(2+√(2+…)))=Xとおきましたが、
そもそも値は収束するのかどうか
わかっていないので、まずいのですが、
ここでは気にしないことにします)

113 名前:132人目の素数さん投稿日:03/04/17 18:50
f(x) = √(2+x)
x[n+1] := f(x[n])
x[n] < 2 ⇒ x[n] < x[n+1] < 2
2 < x[n] ⇒ 2 < x[n+1] < x[n]
x[n] = 2 ⇒ x[n+1] = x[n] = 2
∴ lim x[n] = √(2+√(2+√(2+ … +x[0]))) = 2 for ∀x[0] > -2

114 名前:132人目の素数さん投稿日:03/04/17 20:49
i = √-1 = √(-1/1) = √(1/-1) = √1 / √(-1) = 1/i = -i
うひゃあ
どこが間違っているか指摘せよ

115 名前:132人目の素数さん投稿日:03/04/17 20:51
√(1/-1) = √1 / √(-1) このへん

116 名前:132人目の素数さん投稿日:03/04/17 21:47
1円玉、5円玉、10円玉、50円玉、100円玉、500円玉がそれぞれあります。
全部の枚数を合計すると、100枚、全部の金額を合計すると、29000円。
この条件を満たすことは可能か、不可能かを答えよ。

117 名前:132人目の素数さん投稿日:03/04/17 22:07
可能、
500円玉50枚、100円玉50枚で30000円になる。
あとは微調整、100円玉20枚を50円玉20枚に変えてやればおわり。

118 名前:132人目の素数さん投稿日:03/04/17 22:30
可能

119 名前:詩人数学者 ◆JDb6NSLgcY 投稿日:03/04/17 22:55
なんか僕の出した問題何気に無視されてるなぁ。じゃ、もっと簡単な類似題考えておくわ。こうごきたい(藁

120 名前:132人目の素数さん投稿日:03/04/17 23:01
どの問題よ?

121 名前:詩人数学者 ◆JDb6NSLgcY 投稿日:03/04/17 23:09
ゴメソ>>77です。だいぶ前にだされたやつですね。ちょいムズい(僕にとっては)。

122 名前:132人目の素数さん投稿日:03/04/17 23:14
>>118
理由書かないのかよ!!?

123 名前:詩人数学者 ◆JDb6NSLgcY 投稿日:03/04/17 23:32
早速風呂上がりに例の類似題。 一辺が1である正方形ABCDのAC上に任意に点Pを置く。 点Pを中心とする、正方形に収まる最大の大きさの円の面積をSとする。 Sの期待値を求めよ。

124 名前:132人目の素数さん投稿日:03/04/17 23:38
>可能か、不可能かを答えよ。

125 名前:132人目の素数さん投稿日:03/04/17 23:41
>>123
0.25x3.14

126 名前:詩人数学者 ◆JDb6NSLgcY 投稿日:03/04/17 23:49
>>125 3.14は円周率だと思うけど、あってる?計算方法教えて。

127 名前:132人目の素数さん投稿日:03/04/17 23:51
0.25えっくす3.14

128 名前:132人目の素数さん投稿日:03/04/17 23:55
>>123
π/3

129 名前:詩人数学者 ◆JDb6NSLgcY 投稿日:03/04/17 23:59
うーん、計算方法も述べよ、て書けばよかったな。一応答えらしきものはでてきてるんだが、自信ないので計算方法も書いてちょ。

130 名前:128投稿日:03/04/18 00:04
>>129

πx^2 を 0から1まで積分しますた。

131 名前:積分範囲投稿日:03/04/18 00:07
(√2)/2を境に増加から減少に変わる、0から√2までじゃなく?


132 名前:132人目の素数さん投稿日:03/04/18 00:10
>>123
答えはπ/6かな?

133 名前:詩人数学者 ◆JDb6NSLgcY 投稿日:03/04/18 00:12
>>130 どう解釈したかはしらないけど、点Pが正方形の中心にある時面積は最大値pi/4をとるから多分間違いかも。

134 名前:128投稿日:03/04/18 00:14
π/3って1越えてんじゃん(w

πx^2 を 0から1/2まで積分して π/24

135 名前:132人目の素数さん投稿日:03/04/18 00:16
うーん、ちょっと問題が読めない。
移動点Pを中心とする、内接する円の面積?

136 名前:132人目の素数さん投稿日:03/04/18 00:16
ついでに最大ってのも(略)

137 名前:128投稿日:03/04/18 00:17
>>131

x(円の半径)は0から1/2まで等確率で現れると解釈した。

138 名前:詩人数学者 ◆JDb6NSLgcY 投稿日:03/04/18 00:18
>>134 それじゃ積分範囲が半分になっちゃうかな。だから・・・

139 名前:詩人数学者 ◆JDb6NSLgcY 投稿日:03/04/18 00:24
ちょいと問題わかりにくかったかな。点Pが中心になってる例の円は正方形の少なくとも二辺(AB、ADあるいはCD、CBまたは全ての辺)に接してなおかつ正方形からはみでていない。こうかけばわかるかな?

140 名前:128投稿日:03/04/18 00:33
そっか、確率だから全体を1にしなきゃいけないんだった。


x(円の半径)は0から1/2まで等確率で現れるので、
πx^2 を 0から1/2まで積分して π/24
全体を1にするために1/2で割って、 π/12

141 名前:詩人数学者 ◆JDb6NSLgcY 投稿日:03/04/18 00:38
>>140  多分正解。でも1/2で割る(2でかける)のは等確率にするためじゃなくて、図の対称性のためと書けば満点かな。みんな、あってるよね?

142 名前:128投稿日:03/04/18 00:40
>>141

それは

>x(円の半径)は0から1/2まで等確率で現れるので

ここに含まれてるってことで勘弁してけろ

143 名前:詩人数学者 ◆JDb6NSLgcY 投稿日:03/04/18 00:45
あ、そっかごめん。等確率にするためって文に気をとられてて気がつかなかった。悪かった。

144 名前:128投稿日:03/04/18 00:48
>>143
念のため聞くけど、

正方形の1辺を2にした場合、
πx^2 を0から1まで積分して、π/3

これでおしまいだよね?

145 名前:132人目の素数さん投稿日:03/04/18 00:51
>>128>>144
なぜそれが問題の意図を反映しているのか説明しなければいけないんじゃない?

146 名前:詩人数学者 ◆JDb6NSLgcY 投稿日:03/04/18 02:53
>>145は正しい。辺の長さを一般化して考えてみるといいかも。

147 名前:132人目の素数さん投稿日:03/04/18 13:46
>>112
三角関数を使用しない方法
(x+1/x)^2=x^2+2+1/x^2 より x+1/x=√(2+x^2+1/x^2) , x^2+1/x^2=√(2+x^4+1/x^4)
x+1/x=√(2+x^2+1/x^2)=√(2+√(2+x^4+1/x^4))=√(2+√(2+√(2+x^8+1/x^8)))=…
一方 i+1/i=0 のため
i^(1/2)+i^(-1/2)=√(2) , i^(1/4)+i^(-1/4)=√(2+√(2)) ,
i^(1/8)+i^(-1/8)=√(2+√(2+√(2))) ,…
極限として i^0+1/i^0=1+1=2

148 名前:詩人数学者 ◆JDb6NSLgcY 投稿日:03/04/18 14:29
>>112 外出かもしれないがx=sqrt(2+x)じゃだめなのか?

149 名前:112投稿日:03/04/18 17:40
まあ解き方は色々ありますわな。

150 名前:58投稿日:03/04/18 18:23
>>67
風邪引いて寝込んでたので亀レススマソ
正解です。因みに2が○だと1が×で孤立するのでダメです。
丁寧に解いてくださいましたね。
作為解としては、1,2の○×を考えるという方法か、(場合分けする)
○の枚数は2か0しかないことに気づく(でないと絶対奇数枚連×が発生する)
という意図でした。15秒で解かれましたが、数分で作ったのでしょうがないでしょう。


151 名前:132人目の素数さん投稿日:03/04/18 18:24
1本100円の牛乳があります。この牛乳は、空きビンを5本持っていくと、新品の牛乳1本と交換してくれます。

さて、問題です。200本の牛乳を飲むには、最低いくら必要か?
ただし、消費税などは考えないものとする。

152 名前: ◆ARIAzKN1wM 投稿日:03/04/18 18:45
500円で6本
∴198本飲むのに16500円
∴200本飲むのに16700円
あってるかな?

153 名前:132人目の素数さん投稿日:03/04/18 18:46
15700円だべ。

>>152
6本飲んでもさらに1本余るだぁよ。

154 名前: ◆ARIAzKN1wM 投稿日:03/04/18 18:49
あそっか、これはもらった牛乳を6本集めてまたもらうことを考えてない

あせったなぁ

155 名前: ◆ARIAzKN1wM 投稿日:03/04/18 19:16
あれ、ちょっとまって
冷静に式立てて考えてみたんですが
16700円になってしまった
自分では間違いが思いつかないんですが
もしよかったら指摘していただきませんか?

購入する牛乳の数をMとして
M+[M/6]+[M/36]ここからさらに補数合わせてもらう牛乳を考える

M=167のとき
以下合同式は(MOD6)
167≡5
[M/6]≡3
[M/36]≡4

つまりさらに2本もらえるから

167+27+4+2=200
M=167つまり167本購入の時200本飲むことができ、その時の金額は16700円


156 名前: ◆ARIAzKN1wM 投稿日:03/04/18 19:32
すいません、根本的な間違いに気づきました
6本で1本もらえると計算しています、5本でしたね。
すれ汚しスイマセンでした

157 名前:132人目の素数さん投稿日:03/04/18 19:33
16100だろ?

158 名前:132人目の素数さん投稿日:03/04/18 19:38
>>156
6本でも間違ってるよ、センスないね。

159 名前: ◆ARIAzKN1wM 投稿日:03/04/18 20:10
数学好きなのにセンスのない自分が嫌だ

下手の横好きってやつですか(´・ω・`)ショボーン

160 名前:132人目の素数さん投稿日:03/04/18 21:41
6本の等しい長さの線分を使って
合同な正三角形を4つつくれっての既出?

161 名前:67投稿日:03/04/18 21:45
>>150=>>58

異議あり!

>「×が連番で奇数枚並ぶことはない。」

この文章は×が一枚単独で現れることに関しては否定してないはず。
「数学だから」なんて甘えは許しませぬ。

162 名前:132人目の素数さん投稿日:03/04/18 21:45
>>160
外出どころではない罠

163 名前:132人目の素数さん投稿日:03/04/18 21:49
>>169
正三角形の大きさを指定しないと
6つでもつくれちゃいますよ。

164 名前:132人目の素数さん投稿日:03/04/18 21:50
>>163
どこに(略)

165 名前:132人目の素数さん投稿日:03/04/18 21:52
先回りするきか!

166 名前:163投稿日:03/04/18 21:53
あ、すまんレス番間違えた。
というわけで、>>169さんに期待。

167 名前:132人目の素数さん投稿日:03/04/18 21:57
  Λ_Λ  \\
  ( ・∀・)   | | ガッ プレッシャーかかるじゃんかよ
 と    )    | |
   Y /ノ    人
    / )    <  >__Λ∩
  _/し' //. V`Д´)/ ←>>163
 (_フ彡        /

168 名前:132人目の素数さん投稿日:03/04/18 22:00
ぬるぽ
るぽぬ
ぽぬる

169 名前:132人目の素数さん投稿日:03/04/18 22:06
      ___ ___           |  >>163
    , ´::;;;::::::;;;:ヽ         |  期待した?
    i!::::::::::::;ハ;::::::ヽ       |  世の中甘くはないのだよ!
    |:::::::ivv' 'vvvリ        .| 
   |:::(i:|─○○|::|       人_____________
   .|::::l:| ( o o):|   ./〉
   |:::::|:l\ 3/:::|,  ./iアノ
   !/^リ;;;;;;;个;;;;リ;;∨::/゙

170 名前:132人目の素数さん投稿日:03/04/18 22:09

  Λ_Λ  \\
  ( ・∀・)   | | ガッ
 と    )    | |
   Y /ノ    人
    / )    <  >__Λ∩
  _/し' //. V`Д´)/ ←>>168
 (_フ彡        /


171 名前:132人目の素数さん投稿日:03/04/18 22:25
          | | ガガガガガッ   .人
  ( ・∀・)   | |       人  <  >__Λ∩
 と    )    | |    人  <  >__Λ∩Д´)/
   Y /ノ    .人   <  >__Λ∩Д´)/   / ←>>166
    / ) .人 <  >__Λ ∩Д´)/   / ←>>167
  _/し' <  >_Λ∩Д´)/   / ←>>168
 (_フ彡 V`Д´)/   / ←>>169
            / ←>>170

172 名前:132人目の素数さん投稿日:03/04/18 22:27
あるおみくじがあります。クジの総数は100個。
その内「大吉」が5個。「中吉」が10個。「小吉」が30個。「吉」も30個。「末吉」が10個。「凶」が10個。「大凶」が5個。
このクジをA君とB君とC君が順番に引きます。

A君が「凶」「大凶」以外を引き、さらにその後B君が「大吉」を引き、さらにその後C君が「末吉」と「大凶」以外を引く確率を求めよ。

173 名前:132人目の素数さん投稿日:03/04/18 22:28
>>http://science.2ch.net/test/read.cgi/math/1047230494/l50

174 名前:132人目の素数さん投稿日:03/04/18 22:28

  Λ_Λ  \\
  ( ・∀・)   | | ガッ
 と    )    | |
   Y /ノ    人
    / )    <  >:;;;::::::;;;:ヽ
  _/し' //. V::::::::::;ハ;::::::ヽ ←>>169
 (_フ彡    |:::::::ivv' 'vvvリ 
          |:::(i:|─○○|::| 


175 名前:132人目の素数さん投稿日:03/04/18 22:33
  /ヘ;;;;;  
  ';=r=‐リ  
  ヽ二/       n  
 ̄     \    ( E) グッジョブ!! >>174
フ     /ヽ ヽ_//

176 名前:詩人数学者 ◆JDb6NSLgcY 投稿日:03/04/18 23:34
>>159 がんがれ!センスはないが数学大好きな僕たちで数学界をアツくしよう!!

177 名前:132人目の素数さん投稿日:03/04/18 23:36
>>176
いいこと言った!
今夜の君は最高だ!

178 名前:132人目の素数さん投稿日:03/04/18 23:55
>>176
やらないか?

179 名前:132人目の素数さん投稿日:03/04/18 23:57
>>177嘘つくなよ。 >>176>>175の下だよ
あっ、今日はエイプリルフールかっ

180 名前:132人目の素数さん投稿日:03/04/19 08:15
今日がエイプリルフールかどうかわからないとします。
目の前に二人の人がいます。
1人は常に本当のことを言う正直な人で、
もう1人は常に嘘を言う嘘つきな人です。
ただし、今日がエイプリルフールであった場合は
正直な人も嘘をつきます。
見た目ではどちらが正直でどちらが嘘つきかは分かりません。

(1)どちらかに一度だけ質問をして
今日がエイプリルフールかどうかを調べなさい。

(2) ((1)とは別で)
どちらかに一度だけ質問をして
どちらが正直でどちらが嘘つきかを調べなさい。

今テキトーに考えた問題。
自分がまだ答を考えていないので
もしかしたら凄い簡単とか解なしだとかかも。

181 名前:132人目の素数さん投稿日:03/04/19 08:18
こういうのって書き込んでからアラが見つかるんだよなあ。
補足。
今日がエイプリルフールだった場合は
正直な人は必ず嘘を言うとします。
       ~~~~~
        ↑ここ補足

182 名前:あぼーん投稿日:あぼーん
あぼーん

183 名前:132人目の素数さん投稿日:03/04/19 09:32
>>100
(1)は簡単。「今日はエイプリルフールですか?」と聞くだけ。
(2)は、………。ああ、ああ、目が、あああぁぁぁぁあーー!

184 名前:132人目の素数さん投稿日:03/04/19 09:41
>>183は(2)の簡潔でエレガントな解答を見出したが、
バルスされて、それどころじゃない様子。

185 名前:132人目の素数さん投稿日:03/04/19 09:42
>183
「今日はエイプリルフールですか?」「いいえ」
どう解釈するのよ?

186 名前:132人目の素数さん投稿日:03/04/19 09:42
>>183
>>100じゃなくて>>180だろと突っこんでみる。
あぁ、既にバルサンされていたのか・・・

187 名前:132人目の素数さん投稿日:03/04/19 09:48
「今日はエイプリルフールですか?」

4/1の場合  正直者「いいえ」、嘘付者「いいえ」
4/1でない日 正直者「いいえ」、嘘付者「はい」

だから分かるのは、2/3の確率でエイプリルフールであるということか

188 名前:132人目の素数さん投稿日:03/04/19 11:29
1+1=田んぼの田!

これを数学的帰納法を用いて証明せよ

189 名前:132人目の素数さん投稿日:03/04/19 11:37
せんせーい そんなの習ってません!

190 名前:132人目の素数さん投稿日:03/04/19 12:26
http://www.spacelan.ne.jp/~oh77/mamoru3/quiz/quiz.htm

これ解ける?
オレは無理だった・・・

191 名前:132人目の素数さん投稿日:03/04/19 13:07
ブラクラの予感

192 名前:132人目の素数さん投稿日:03/04/19 13:25
>>190
「H」に直線を2本書いて、三角形を7つ作れ、という方がもっと難しい。

193 名前:132人目の素数さん投稿日:03/04/19 13:29
>190

一時間経過・・・
まだワカラン。
答え見たい。うずうず・・・

194 名前:132人目の素数さん投稿日:03/04/19 13:30
2本の線を重ねるようにして・・・

195 名前:193投稿日:03/04/19 13:33
直線3本だった。スマソ。

「H」に直線を3本書いて、三角形を7つ作れ

196 名前:132人目の素数さん投稿日:03/04/19 13:35
(1)
「今日はエイプリルフールではない」かつ「あなたは普段嘘つき者である」ならば、1+1=3ですか?

条件文は前件が偽の時、全体として常に真であることに注意。

4/1の場合  正直者「いいえ」、嘘つき者「いいえ」
4/1でない日 正直者「はい」、嘘つき者「はい」

197 名前:196投稿日:03/04/19 13:40
(2)
「今日はエイプリルフールである」かつ「あなたは普段正直者である」ならば、1+1=3ですか?

条件文は前件が偽の時、全体として常に真であることに注意。

4/1の場合  正直者「はい」、嘘つき者「いいえ」
4/1でない日 正直者「はい」、嘘つき者「いいえ」

198 名前:132人目の素数さん投稿日:03/04/19 13:53
マンコの無限性を証明せよ。

199 名前:132人目の素数さん投稿日:03/04/19 17:47
>>198
まずマンコを定義してくれ

200 名前:132人目の素数さん投稿日:03/04/19 21:19
>>196-197
おーなるほど。
と出題者が感心してますが。


じゃ、おまけ。
エイプリルフールの逆で、
嘘つきでも必ず本当のことを言わなければならない日があるとします。
で、どちらかに一度だけ質問して
どちらが正直者でどちらが嘘つきかを調べなさい。

201 名前:132人目の素数さん投稿日:03/04/19 23:44
あれだろ、☆の下半分の∧の真ん中辺りに点を書いて、
そこを中心に横長のxを書くってやつじゃないの?

202 名前:山崎渉投稿日:03/04/20 04:01
   ∧_∧
  (  ^^ )< ぬるぽ(^^)

203 名前:132人目の素数さん投稿日:03/04/20 08:22
横長の>を書いてもよい

204 名前:196投稿日:03/04/20 15:01
>>200
エイプリルフールはエイプリルフールで存在するんだよね?
(そうじゃなきゃつまんないし)

必ず嘘つく日をA日、必ず正直な日をB日としよう。

「『今日はA日である』かつ『あなたは普段正直者である』」または、
「『今日はB日である』かつ『あなたは普段嘘つき者である』」ならば、1+1=3ですか?

A日       正直者「はい」、嘘つき者「いいえ」
B日       正直者「はい」、嘘つき者「いいえ」
それ以外の日 正直者「はい」、嘘つき者「いいえ」

205 名前:132人目の素数さん投稿日:03/04/20 19:22
http://science.2ch.net/test/read.cgi/math/1050682696/327

206 名前:132人目の素数さん投稿日:03/04/21 15:54
嘘付者と正直者の問題って、慣れてないとムズイですね。
パターン分けして紹介してある本って、ありますか?

207 名前:132人目の素数さん投稿日:03/04/21 23:34
パターン分けはどうかわからないが、
スマリヤンの本に腐るほど出てくるよ。

208 名前:132人目の素数さん投稿日:03/04/22 00:10
>>207
ありがとん。
検索したら一杯出てきた。
http://books.yahoo.co.jp/bin/search_key?p=%A5%EC%A5%A4%A5%E2%A5%F3%A5%C9+%A5%B9%A5%DE%A5%EA%A5%E4%A5%F3&pp=&sc=7

209 名前:196投稿日:03/04/22 00:59
>>206
おいら「スマリヤンの究極の論理パズル」って本持ってるよ。

210 名前:132人目の素数さん投稿日:03/04/22 01:23
今日から、1000000日後は、何年何月何曜日か?

211 名前:132人目の素数さん投稿日:03/04/22 01:30
4741年3月19日

212 名前:132人目の素数さん投稿日:03/04/22 01:32
水曜日

213 名前:132人目の素数さん投稿日:03/04/22 02:07
>>209
いい子だから、その本を私に…

214 名前:132人目の素数さん投稿日:03/04/22 10:53
77の問題の答えが気になる…

215 名前:132人目の素数さん投稿日:03/04/22 16:39
漏れも77がわからん

216 名前:132人目の素数さん投稿日:03/04/22 16:42
一生考えてろ

217 名前:132人目の素数さん投稿日:03/04/22 18:43
くぅ…

218 名前:132人目の素数さん投稿日:03/04/22 19:27
age

219 名前:132人目の素数さん投稿日:03/04/22 20:53
>>211
それは西暦か?平成か?

220 名前:132人目の素数さん投稿日:03/04/23 05:42
>>219 Gregorian calendar

221 名前:132人目の素数さん投稿日:03/04/23 14:41
解けそうで一つも解けない
ttp://www1.odn.ne.jp/konishi/Math/index.html

222 名前:132人目の素数さん投稿日:03/04/23 17:24
エンドレステープ T に録音された曲の集合を S(T) と記し、
T 録音された曲の延べ曲数を L(T) と記す。
テープ T を繰り返し演奏すると #S(T)! 個ある S(T) のすべての順列が
出現するときテープ T を #S(T)-順列完全循環であるという。

例えば T = <0,1,2,0,1,0,2,1> について S(T) = {0,1,2}、#S(T) = 3、
L(T) = 8 であり、T の添字 i から長さ n の部分列を T[i,n] で表すと、
T[0,3] = <0,1,2>, T[1,3] = <1,2,0>, T[2,3] = <2,0,1>,
T[4,3] = <1,0,2>, T[5,3] = <0,2,1>, T[6,3] = <2,1,0> は
S(T) のすべての順列を網羅しているので、この T は 3-順列完全循環である。
またこの T は3-順列完全循環のうち L(T) が最小のものである。
すなわち、L(T)<8 となる3-順列完全循環なテープ T は存在しない。
N-順列完全循環のうち L(T) を最小にする T の L(T) を LPCC(N) で表す。
例えば LPCC(3) = 8 である。

問題: LPCC(4)、LPCC(5) および LPCC(N) を求めよ。

223 名前:132人目の素数さん投稿日:03/04/23 19:49
http://science.2ch.net/test/read.cgi/math/1040540700/229

224 名前:しいたけ投稿日:03/04/23 20:06
1/2 * 1/3 + 1/3 * 1/4 + 1/4 * 1/5 + 1/5 * 1/6 + 1/6 * 1/7この問題をあっという間に解く次の一行があります。
その一行を見れば答えが瞬時にわかるくらいの驚くべき変形の仕方です。
まだそれを知らない人で、自力で思いつける人はいますか?

ちなみに普通の考え方ではありません。
知識は分数計算ができるレベルで十分なのですが、
変形の発想が特異で難しいのです。
みなさんに挑戦します。




225 名前:しいたけ投稿日:03/04/23 20:08
コピペされた方、ありがとうございます。

226 名前:132人目の素数さん投稿日:03/04/23 20:13
(1/2-1/3・・・・・


1/2-1/7=5/14

227 名前:しいたけ投稿日:03/04/23 20:19
>226
正解です。

228 名前:132人目の素数さん投稿日:03/04/23 20:44
>>226
厨にもわかるように説明して欲しいのですが

229 名前:132人目の素数さん投稿日:03/04/23 20:58
1/2*1/3=1/2-1/3

230 名前:132人目の素数さん投稿日:03/04/23 21:42
>>222
LPCC(4)=33 かな。漏れの能力ではここまでが限界です(;´Д`)

231 名前:132人目の素数さん投稿日:03/04/23 22:43
>>225
モマエ工房ダロ その考え方は数列で見たな。

232 名前:132人目の素数さん投稿日:03/04/23 22:43
>>231
中学入試じゃないの?

233 名前:132人目の素数さん投稿日:03/04/23 22:55
中学入試の頻出問題ですな

234 名前:132人目の素数さん投稿日:03/04/25 22:17
>>77>>221>>222のこたえ募集

235 名前:132人目の素数さん投稿日:03/04/26 21:51
age


236 名前:これでいいの?投稿日:03/04/27 14:48
>>77>>234
x=APとおく。
AからBCに下ろした垂線をAHとする。相似の関係からAH=AB(AC/BC)=12/5
1)0<=x<12/5のとき。
S(x)=(πx^2)/4

2)12/5<=x<3のとき、
円が、三角形の辺BCからはみだした部分の面積Q勘定する。円とBCの交点S、Tを、
B,S,T,Cの順になるように定める。∠SAT=αとする。
Q=扇形SAT-△SAT=x^2(α-sinα)/2
(ここでcosα=(12/5)/x=12/(5x)に注意)
従って、S(x)=(πx^2)/4-Q

期待値はx:0〜3での平均だから、[∫[0,3]dx(πx^2)/4+∫[12/5,3]dxx^2(α-sinα)/2]/3・・・*
但し、β=cos^(-1)(4/5)とする。(sinβ=3/5、tanβ=3/4)

*の第一項は、3π/4・・・・・・1
第2項の不定積分は、
(96/125)*[(α-sinα)/cos^3α+sinα/(2(sin^2α-1))+(log|(sinα-1)/(sinα+1)|)/4+tanα]
なので、積分範囲はα=0〜βより積分の値は、
(4/3)β-(43/200)-(log2)/2・・・2

1,2から、3π/4+(4/3)β-(43/200)-(log2)/2

237 名前:132人目の素数さん投稿日:03/04/27 17:03
age

238 名前:132人目の素数さん投稿日:03/04/28 00:39
巡回吉野家問題

国内にある吉野家を最短距離で巡回する為の経路を求めよ。

239 名前:132人目の素数さん投稿日:03/04/28 12:16
赤日の問題。
5を5つ使って5の平方根に最も近い演算を書け。
ただし、使える記号は(,),+,−,×,÷,.,のみである。
.5は0.5と見なさない。




240 名前:132人目の素数さん投稿日:03/04/28 15:09
55.5 / (5*5)

ってのを思いついた。55はありなんかなぁ。

241 名前:132人目の素数さん投稿日:03/04/28 17:48
a桁の自然数の中で、各位の数字の和がbである自然数はn個ある。
nをaとbの式で表せ。
今思いついた問題。
自分でも答えがわからん。

242 名前:132人目の素数さん投稿日:03/04/28 17:56
b個の○と(a-1)個の|を並べる並べ方は何通りか?
ただし、○は最大で9個までしか連続して並べられないとする。

って読み替えられるな。あ、加えて先頭は○である必要がある、と。


無理ぽ(;´Д`)
なんつーかねぇ、2個までしか連続しないとか言う問題がさくらスレに割とよく上がる気がするんだが・・・
それの類似形かいな

243 名前:132人目の素数さん投稿日:03/04/28 19:57
基本的にはその方針でやればできると思うよ。
まず9個までしかって条件をなしで求めて、
んでそこから9個以上並ぶ場合を数えて、それを引く。
さらには2箇所以上で9個以上並ぶ場合を2倍引いてしまっているので、
2箇所以上で9こ以上並ぶ場合を数えてそれを足す。
さらに3箇所以上で(ry

Σを使えば短い式でたぶん書けまつ。

244 名前:132人目の素数さん投稿日:03/04/28 20:47
>>236
解答thanks

245 名前:132人目の素数さん投稿日:03/04/28 20:54
>>238
巡回している間に新たに吉野家が誕生する可能性もあるので、
その予測が出来なければ、まず不可能。
巡回する人の腹の大きさなどにもよる。
何も食べずに常に立ち寄るだけで良いのか?
もう少し条件付け(ry

246 名前:132人目の素数さん投稿日:03/04/28 21:27
吉野屋の分布からしてむしろ最長経路の方が難しいのでは?

247 名前:132人目の素数さん投稿日:03/04/28 22:57
e^ix=cosx+isinx
を証明せよ

248 名前:132人目の素数さん投稿日:03/04/28 22:59
どうやるんだっけ?


249 名前:132人目の素数さん投稿日:03/04/28 23:01
証明ではありません。定義です。

250 名前:132人目の素数さん投稿日:03/04/28 23:02
1+1=2
を証明せよ

251 名前:132人目の素数さん投稿日:03/04/28 23:04
どうやるんだっけ? 
↓   

252 名前:132人目の素数さん投稿日:03/04/28 23:27
>>247
一致しただけのことではなかったか?

253 名前:132人目の素数さん投稿日:03/04/28 23:36
だから定義だっつーの 

254 名前:132人目の素数さん投稿日:03/04/29 01:13
e^ix は sin と cos を定義してそこから導く教科書もあるんじゃないかな

255 名前:132人目の素数さん投稿日:03/04/29 01:17
普通sinとcosは無限級数で定義するもんかな?

256 名前:132人目の素数さん投稿日:03/04/29 01:52
アールフォルスの教科書はなかなかカッコいい定義になってるので見るべし

257 名前:132人目の素数さん投稿日:03/04/29 01:54
>>256
是非おしえてください。

258 名前:132人目の素数さん投稿日:03/04/29 02:39
微分方程式の解として exp を定義し exp で sin と cos を定義する
オイラーの公式はそこから導かれる
漏れの手元にないから詳しくは自分で見てね

259 名前:132人目の素数さん投稿日:03/04/29 02:49
>>258
Thx

260 名前:132人目の素数さん投稿日:03/04/29 21:35
ageついでに駄問。

  \
\/\
/\/
  /

棒を2本だけ動かして魚の向きを変えて下さい。

261 名前:132人目の素数さん投稿日:03/04/29 21:38
>>260

 
\/
\/\/
\/


262 名前:132人目の素数さん投稿日:03/04/29 21:56
>>260 幼稚園向け、まさに駄問だな…

 /\
/\/\
 /\
 


263 名前:132人目の素数さん投稿日:03/04/29 22:10
ムチャクチャ難しいマッチ棒パズルないかな?

264 名前:132人目の素数さん投稿日:03/04/29 22:13
 /⌒\
(    ..)
|   |
|   |
|   |
|   |
|   |

次の図形に数学的言語を用いて インポにせよ

265 名前:132人目の素数さん投稿日:03/04/29 22:36
長さが1の42本のマッチ棒を使って、
一辺の長さが1であるような三角形を50個作れ、
とか。

簡単か。

266 名前:132人目の素数さん投稿日:03/04/29 22:37
あ、ごめ。三角形じゃなくて正三角形ね。

267 名前:132人目の素数さん投稿日:03/04/29 23:40
>264

デデキントの切断

268 名前:264投稿日:03/04/29 23:46


 /⌒\
(    ..)
| 解..|
| な...|
| し....|
|  ....|
|   |

269 名前:yokotee投稿日:03/04/30 00:01
マッチ棒を6本使って三角形を4つ作れ。

270 名前:132人目の素数さん投稿日:03/04/30 00:06

   \
\\/
//\
   /


271 名前:132人目の素数さん投稿日:03/04/30 00:07
>>269
6本も要りません

272 名前:詩人数学者 ◆JDb6NSLgcY 投稿日:03/04/30 00:25
4本あれば十分(かなりあほっぽいが)。ところで長さはすべて同じだよね?

273 名前:詩人数学者 ◆JDb6NSLgcY 投稿日:03/04/30 00:29
あ、長さ関係ないわ。スマソ・・・

274 名前:詩人数学者 ◆JDb6NSLgcY 投稿日:03/04/30 00:40
またまたスマソ。>>272は四角形と間違えました・・・寝ぼけてんな。もう寝ます。おやすみZzz・・・

275 名前:132人目の素数さん投稿日:03/04/30 00:44
まぁ>>269は正四面体と言いたかったんだろう。

276 名前:132人目の素数さん投稿日:03/04/30 01:01
いやわからんぞ、□に×重ねて上から見ると・・・

277 名前:132人目の素数さん投稿日:03/04/30 01:03
井に×か?

278 名前:132人目の素数さん投稿日:03/04/30 01:04
上から見ると、×のはじっこが□にとどいてない、鬱。

… #に×を重ねれば良かった (・ω・)ショボーン

279 名前:132人目の素数さん投稿日:03/04/30 01:07
同一平面上の6本の直線によって、最大でいくつの三角形ができるか?
ってスレが昔あったような。

280 名前:132人目の素数さん投稿日:03/04/30 07:46
>>279
http://science.2ch.net/test/read.cgi/math/1017267660/l50
はい、おはようさん。ここっすよ〜

281 名前:132人目の素数さん投稿日:03/04/30 11:44
長さが1の10本のマッチ棒を使って、
一辺の長さが1であるような正三角形を10個作れ、
とか。 どうよ。


282 名前:132人目の素数さん投稿日:03/04/30 11:52
1光年で頑張れば
1本のマッチで正三角形は作れると思た


283 名前:132人目の素数さん投稿日:03/04/30 12:17
>>282


284 名前:132人目の素数さん投稿日:03/04/30 15:51
長さが1の8本のマッチ棒を使って、
一辺の長さが1であるような正三角形を4個作れ、
ただし平面上に。
とか。 どうよ。

285 名前:あぼーん投稿日:あぼーん
あぼーん

286 名前:132人目の素数さん投稿日:03/04/30 16:57
>>284
ほんとに出来るの?

287 名前:132人目の素数さん投稿日:03/04/30 17:19
問:長さ1の棒に百科事典のデータをすべて詰め込むにはどうしたらよいか?

288 名前:284投稿日:03/04/30 17:20
>>286
できますよ。ちょっと反則くさいですけどね。

289 名前:284投稿日:03/04/30 17:20
>>287
村上春樹、世界の終わりとハードボイルドワンダーランドだったっけ?

290 名前:132人目の素数さん投稿日:03/04/30 17:22
>>289
正解。

291 名前:132人目の素数さん投稿日:03/04/30 17:41
>>287
表面積を大きくする

292 名前:132人目の素数さん投稿日:03/04/30 17:45
>>289
それ読んだこと無いけどこういう奴?

全ての百科事典を数値化し、頭に "0." を付けてこれを x とする。
その数は 0 < x < 1 だから長さ 1 の棒の x の位置に印を付ける。

293 名前:132人目の素数さん投稿日:03/04/30 17:54
>>292
正解です。暇だったらついでに村上春樹の「世界の終りと〜」読んでみましょう。

294 名前:132人目の素数さん投稿日:03/04/30 18:36
懐かし過ぎ。あの頃は村上春樹がこんなにメジャーになるとは思ってなかったyo

295 名前:132人目の素数さん投稿日:03/04/30 18:46
現実問題として、分解能をいくらでも小さくできるわけじゃないから
不可能な訳だけども。

世界の終りと〜で、一番良い暗号化法は辞書式暗号だって書いてあって、
萎えた。

296 名前:132人目の素数さん投稿日:03/05/01 00:45
>>288
降参です。教えて下さい

297 名前:132人目の素数さん投稿日:03/05/01 02:14
>>292
>全ての百科事典を数値化し、頭に "0." を付けてこれを x とする。
>その数は 0 < x < 1 だから長さ 1 の棒の x の位置に印を付ける。

うおー、目から鱗。
感動した!

298 名前:284投稿日:03/05/01 09:06
>>296
{(x,y)|x,y∈R}
(x_1,y_1,)と(x_2,y_2)との距離を
max(|x_1-x_2|,|y_1-y_2|)
で定義する

マッチ棒の始点−終点がそれぞれ
(0,0)−(1,0)、(0,0)−(0,1)、(0,0)−(-1,0)、(0,0)−(0,-1)
(1,0)−(0,1)、(0,1)−(-1,0)、(-1,0)−(0,-1)、(0,-1)−(1,0)
に来るように置く。終り。

299 名前:132人目の素数さん投稿日:03/05/01 11:46
袋の中にN個の白い玉と、1個の黒い玉が入っています。
以下の操作を繰り返したとき、最後に取った玉が
黒である確率と白である確率ではどちらが大きいでしょう。

取り出した玉が、
1) 白だったらそれを袋から除く。
2) 黒だったら一旦袋に戻してから、玉を取り出し
  その色の如何に関わらず袋から除く。

300 名前:132人目の素数さん投稿日:03/05/01 11:47
最後に取った -> 最後に残った

301 名前:132人目の素数さん投稿日:03/05/01 11:54
わずかに白?

302 名前:132人目の素数さん投稿日:03/05/01 12:06
と、勘で言ってみましたが。
計算してみると、白が残る確率が(N+1)/(2N)ですか。
勘で当たってたね。

303 名前:132人目の素数さん投稿日:03/05/01 12:16
あり? (N+2)/2(N+1) とかなっちまった。どこ間違えたかな…

304 名前:132人目の素数さん投稿日:03/05/01 12:17
あ、N'=N+1 で一緒か。

305 名前:301,302投稿日:03/05/01 12:21
ああそうですね。僕のほうのNがおかしいですね、すんまへん。

306 名前:132人目の素数さん投稿日:03/05/01 15:02
三角形ABCがある。
三つの頂点から降ろした垂線の長さはすべて2以上であるとする。
この三角形の周上を点P,QがPQ=2を満たして移動する。
PQの中点をRとする。

点Pが頂点Aを出発して、反時計回りに三角形の周上を一周する時、
Rの軌跡と、三角形の周によって囲まれる部分の面積を求めよ。

307 名前:132人目の素数さん投稿日:03/05/01 16:30
>>299
白N、黒1から黒を取る確率が{1/(N+1)}^2だから、
黒が残る=白をN個連続で取る
=Σ[k=1〜N] [ 1 - {1/(k+1)}^2 ]

こっから進まない(;´Д`)
{1-(1/4)} + {1-(1/9)} + ・・・ {1-(1/(N+1))} と考えはしてみるものの

>>302-303 解説きぼん

308 名前:132人目の素数さん投稿日:03/05/01 16:41
>>307
白 N, 黒 1 から白を取る確率は

 最初から白を取る確率 + 最初黒かつ次に白を取る確率

で、ずっと白を取りつづける確率はこれのΠを取ればいい。
そうするとどんどん項が消えて…

309 名前:132人目の素数さん投稿日:03/05/01 16:51
307の考え方でもあってるしそれで解けるんだけどね、
なぜか間違えて積でなく和をとっちゃってる。

310 名前:132人目の素数さん投稿日:03/05/01 19:51
>>306
部分的にわからないところがあるんだけど、設問だけで解が得られるという情報から(w 
π/2 かな?

311 名前:132人目の素数さん投稿日:03/05/01 19:54
>>310
残念だけど、πが正解だよ。

312 名前:132人目の素数さん投稿日:03/05/01 19:58
>>310
残念だけど、π=3だよ。

313 名前:310投稿日:03/05/01 20:02
P が三角形の角にあるとき R はそこから 1 離れたところにあるよね?
半径 1 の円の面積の半分じゃないの?

314 名前:132人目の素数さん投稿日:03/05/01 20:05
> P が三角形の角にあるとき R はそこから 1 離れたところにあるよね?
 ↓
> 半径 1 の円の面積の半分じゃないの?

ものすごい飛躍じゃないのか?

315 名前:名無しさん@Linuxザウルス投稿日:03/05/01 22:31
>>306
PがAB上に
QがBC上に
存在するものとして、考える

Rを通り、BCに平行な直線とABの交わる点をOとする。
ROをO側に延長して
SO=RO
OをRSの中点となるように点Sをとる
この時、明らかに四角形PRBSは平行四辺形になる
よってBS=1

この事から,点Sの軌道は円孤であると分かる。

SO=ROより、
求める面積は・・・・・

316 名前:あぼーん投稿日:あぼーん
あぼーん

317 名前:132人目の素数さん投稿日:03/05/01 22:39
>>315
なるほど。すごいな。

318 名前:132人目の素数さん投稿日:03/05/02 11:34
パナウェーブ研究所?
スカラー電磁波って( ´,_ゝ`)プッ
http://plaza16.mbn.or.jp/~satchel/




319 名前:132人目の素数さん投稿日:03/05/02 23:58
>>298
マッチ棒の解答、答えになってない。

320 名前:132人目の素数さん投稿日:03/05/03 00:13
298のはよく読んでないけど、要するに正八面体を真っ二つに割った感じでしょう?

321 名前:320投稿日:03/05/03 00:15
ごめん、勘違いしてた。平面上に作らないとダメなのね。

322 名前:132人目の素数さん投稿日:03/05/03 22:17
さんざん既出の問題だが、

あるパーティーで次のような催しをした。
「いまこのパーティー会場にいる人で、
 お互いに認識がある人の人数を紙に書いてください。
 もちろん、自分自身はカウントしないでください。」
さて、この紙を集計したとき、すべての紙に書かれた数が違うことがあるだろうか。
ただし、紙には嘘は書かれていないものとし、
「お互いに認識がある」ということは、一方のみの勘違いではないものとする。

323 名前:132人目の素数さん投稿日:03/05/03 22:43
類似問題。
4組の夫婦(つまり8人)でパーティを開いた。
その中の一人、Aさんが、ほかの7人に紙を配り、
今日握手をした人の数を書いてもらった。
(もちろん同じ人に何度か握手したのは一人と数える)
すると全員が違う人数を書いた。
さらに、夫婦で握手をした人はいないという。
このとき、Aさんの妻は、何人の人と握手したことになるか。

324 名前:132人目の素数さん投稿日:03/05/04 00:06
W杯4チーム増案は6月末に結論 36チーム前向き検討
http://www.asahi.com/sports/update/0503/137.html

W杯36チーム参加になった場合の合理的優勝チームの確定方法を求めよ。

325 名前:132人目の素数さん投稿日:03/05/04 00:17
>>324
何試合ぐらいを目どにするのかな?
決勝まで進むチームが5試合や、全体の試合数が50試合とか。
そのくらいか?選手の体力や移動や、期間なども考慮して。

326 名前:132人目の素数さん投稿日:03/05/04 00:31
一辺が3の正方形と
一辺が4の正方形を
合計で2002個使い、
大きな正方形を作れ

去年出すべきだた

327 名前:132人目の素数さん投稿日:03/05/04 00:34
3x3x2002=18018
4x4x2002=32032
この2値の間の平方数で(ry

328 名前: ◆ncKvmqq0Bs 投稿日:03/05/04 00:45
>>324
3チームで1グループ→12グループ作って、各グループ内で総当り(2試合・36試合)。
勝利チーム2チーム、計24チームが決勝トーナメント進出(4・5試合・23試合)。
┏┻┓   ┏┻┓┏┻┓   ┏┻┓┏┻┓   ┏┻┓┏┻┓   ┏┻┓
┃┏┻┓┏┻┓┃┃┏┻┓┏┻┓┃┃┏┻┓┏┻┓┃┃┏┻┓┏┻┓┃


329 名前: ◆ncKvmqq0Bs 投稿日:03/05/04 00:54
3位決定戦をいれれば、決勝トーナメントは24試合ね。

330 名前:132人目の素数さん投稿日:03/05/04 03:56
>>328
モアイかと思った

331 名前:あぼーん投稿日:あぼーん
あぼーん

332 名前: ◆ncKvmqq0Bs 投稿日:03/05/04 09:12
因みに前回日本開催時は
予選8グループ各4チーム(3試合48試合)
決勝トーナメント16チーム(4試合16試合)
1チーム7試合 大会合計64試合

>>328では、
1チーム6、7試合 大会合計60試合
かなり難しいかな最初のグループ分けのバランスが。
それに、シード8チーム(12欲しいのに)なんだよね>>328だと。これは大問題ですw
┏┻┓   ┏┻┓┏┻┓   ┏┻┓┏┻┓   ┏┻┓┏┻┓   ┏┻┓
┃┏┻┓┏┻┓┃┃┏┻┓┏┻┓┃┃┏┻┓┏┻┓┃┃┏┻┓┏┻┓┃


333 名前: ◆ncKvmqq0Bs 投稿日:03/05/04 09:31
3チーム総当りを2回繰り返すという手も有るけれど。
3チーム総当り36チーム→12チーム各1位のみ(2試合36試合)
3チーム総当り12チーム→8チーム各2位まで(2試合18試合)
決勝トーナメント8チーム(3試合8試合)
1チーム7試合 大会合計62試合
最初の篩落としが厳しいのだけれど・・・グループに偏りが有ると非難必至(汗)
かといって全体の試合数が・・・

334 名前:132人目の素数さん投稿日:03/05/04 16:35
>>322
ない。

全部でN人いたとすると、書かれる数字は0〜(N-1)が1個ずつ。
0と書いた人は知り合いがいない、(N-1)と書いた人は全員と知り合い。矛盾。

335 名前:132人目の素数さん投稿日:03/05/04 16:40
>>323
最大6人と握手できるから、書かれた数字は0〜6。
6人握手をした人の夫(妻)は、誰とも握手をしていない。
5人握手をした人の夫(妻)は、1人とのみ握手をしてた。
4人握手をした人の夫(妻)は、2人とのみ握手をしてた。
3人握手をした人の夫(妻)が、紙を配った人であり、その人も3回握手をしている。

さて、もしAさんが女だったら(略

336 名前:323投稿日:03/05/04 16:56
Aさんは男です。書き忘れ。すみませぬ。

337 名前:132人目の素数さん投稿日:03/05/04 17:21
>>335
>6人握手をした人の夫(妻)は、誰とも握手をしていない。

なんで?

338 名前:mathmania ◆uvIGneQQBs 投稿日:03/05/04 17:25
マルチポスト御免。
A,B,Cは、正の整数で、平方数でないとする。
このとき、√A+√B+√Cが無理数であることを証明せよ。

339 名前:132人目の素数さん投稿日:03/05/04 19:25
>>337
図を描いてみるとわかるかも。

340 名前:337投稿日:03/05/04 19:29
>>339
さいですか。
いわゆる「自明だけど説明はめんどい」って奴か。
あとでゆっくり考えてみよっと。

341 名前:132人目の素数さん投稿日:03/05/04 20:14
>>340
まず、0〜6が1つずつ書かれていることに注意。

Bさんが、6人と握手をしたとすると、
Bさんは、自分の夫(妻)以外全員と握手をしたことになる。

じゃ、誰とも握手をしていないのは誰だろうと考えてみる。
すると、Bさんの夫(妻)しかありえない。

342 名前:322投稿日:03/05/04 20:33
>>334
正解。じゃあ少し発展させて、

自分のことをカウントしてもよい場合はどうか。
ただし、自分をカウントするかしないかは人によって違う。
他の条件は同じとする。

343 名前:132人目の素数さん投稿日:03/05/05 02:12
x+y+z=3,x^2+y^2+z^2=1,xyz=2が成り立つとき、
xy+yz+zx,x(y^3+z^3)+y(z^3+x^3)+z(x^3+y^3)
のそれぞれの値を求めよ。

344 名前:132人目の素数さん投稿日:03/05/05 02:13
>>343
簡単すぎやせんか?

345 名前:132人目の素数さん投稿日:03/05/05 02:14
>>343はマルチ

346 名前:132人目の素数さん投稿日:03/05/05 02:15
>>343
http://science.2ch.net/test/read.cgi/math/1052060918/13

347 名前:13投稿日:03/05/05 02:21
こっちに書き込んでるのは俺じゃない。

348 名前:343投稿日:03/05/05 02:22
いや俺こっちにしか書き込んで無いけど・・・。

349 名前:13投稿日:03/05/05 02:22
俺です。

350 名前:13、343投稿日:03/05/05 09:38
どっちでもいいじゃん

351 名前:132人目の素数さん投稿日:03/05/06 22:35
十二指腸潰瘍で病院に入院している時考えてた問題です。

半径1の円に内接する三角形の面積の最大値を求める時、以下の2変数
関数fを考察する。

f(A,B)=sin(A)+sin(B)+sin(A+B)

(1)t=tan(A/2),s=tan(2/B)とするとき、f(t,s)を求めよ
(2)x=t+s,y=ts と置くときf(x,y)の最大値を求めよ。

面白くないかもしれないけど、よかったらどうぞ


352 名前:132人目の素数さん投稿日:03/05/06 22:39
面白い面白くない以前に関数の書き方勉強してから出直してこい。馬鹿。

353 名前:132人目の素数さん投稿日:03/05/06 22:48
今日高校の図書館でピーターフランクルの本を読んでいて底から出題します
なにも面白くはないんだけど
太郎君は
金貨を1枚持っています
次郎君は
太郎君と同じ金貨を3枚
見た目はまったく同じだけど
重さがちがう金貨を1枚
持っています

天秤を2回使って
重さのちがう金貨を見つけ出せ

354 名前:132人目の素数さん投稿日:03/05/06 22:50
>>353
見つけました

355 名前:132人目の素数さん投稿日:03/05/06 22:58
>>352
(;゚д゚) ハァ?

356 名前:132人目の素数さん投稿日:03/05/06 22:59
(1)のsの角度が分母分子 逆だと言いたいのでは?

357 名前:132人目の素数さん投稿日:03/05/06 23:00
>>326
の答え教えて

358 名前:132人目の素数さん投稿日:03/05/06 23:01

イラク戦争の写真がいっぱい!!!!

まともに見れるかよ!!!!

心臓の悪い方はご遠慮ください!!!

http://www.marchforjustice.com/3.30.php

359 名前:132人目の素数さん投稿日:03/05/06 23:37
AとB地点が平面上にある。
A地点からB地点に移動したい。

そのためには、AとBの中点を通らなくてはならない。
更にその中点と終点の中点を通らなくてはならない。
更にその中点と終点の中点を通らなくてはならない。
こうして無限に多くの点を通らなくてはBにたどり着けない。
しかし実際にはたどりつける。
これはなぜ?

360 名前:132人目の素数さん投稿日:03/05/06 23:50
当然、無限に多くの点を通ってたどり着いたから。

361 名前:132人目の素数さん投稿日:03/05/06 23:54
点には大きさがないから、その点にたどり着いた時点で、そのまわりの無限個の点にも
同時にたどり着いていることになるのではないだろうか。

362 名前:工房投稿日:03/05/07 00:08
素数のおもしろい性質とかありますか?

363 名前:132人目の素数さん投稿日:03/05/07 00:10
気になるなら調べろ!
工房にもなって、オムツの世話が必要なのか?

364 名前:132人目の素数さん投稿日:03/05/07 00:31
>>359
時間を止めようとしている。

365 名前:132人目の素数さん投稿日:03/05/07 00:32
>>362
俺も工房だけど、エラトステネスが素数を発見する方法を考えた。
記憶が曖昧だけど、確か
1から順に数字を並べる。
で、1は素数じゃないので印をつける。
2は素数で、2を残して他の2の倍数にチェックを入れる。
次に印が無いのは3。3素数なのでチ印をつけない。
んで3の倍数にチェックを入れる・・・

説明が死ぬほど下手でスマソ。
まぁ、つまり数字の倍数の数は印がつけられて消されてくから、素数は残るってこと
C言語なんかで1から任意の数字までの素数を計算するプログラムとか作ると面白いかも。


366 名前:工房投稿日:03/05/07 00:53
>>363
オムツなんかはいてねえよ!
・・・てことで自分で調べてみた。

「いろいろなところ見て思ったこと」
・小学校の頃から正の整数扱ってるけど整数自体が結構特別でおもしろい数字かも。
・天才って本当に天才だ。

まぁちょろっと調べてみただけだからまた気が向いた時にでも調べてみるかなー。
簡単で面白いもの見つけたいし。

>>365
それは昔見たことあったー。結構単純な方法だよね。

367 名前:132人目の素数さん投稿日:03/05/07 00:57
>>366
ご褒美だ。これを買って読め!

素数入門 (講談社ブルーバックス)
http://www.bookclub.kodansha.co.jp/Scripts/bookclub/intro/intro.idc?id=31789

368 名前:365投稿日:03/05/07 01:01
>>367
それ、俺もホスィ・・・

369 名前:132人目の素数さん投稿日:03/05/07 01:18
>>326
一辺3の正方形を
201個
544個
873個
1188個
1480個
1776個使う
上記の6つの内の何個かが答えになるはずもしくは全部

370 名前:132人目の素数さん投稿日:03/05/07 01:59
篩と言えばこんなのもある
ttp://logic.pdmi.ras.ru/~yumat/Journal/Sieve/Sieve.html

この Matiyasevich は素数生成多項式で有名な人

371 名前:132人目の素数さん投稿日:03/05/07 02:20
>>369
せめて、理由をつけて答えてやれよ
その数字がどこから出たのかさっぱり分からないぞ

>>326
一辺3の正方形を16個使い、一辺が12の正方形を作る
 −−> これを正方形Aとする

同様に
一辺4の正方形を9個使い、一辺が12の正方形を作る
 −−> これを正方形Bとする

まず、正方形Aを196個つかい、一辺が168の大きな正方形を作る
この時、一辺が3の正方形は
16×196=3136
個、使っていることになる。
ここで、
3136−2002=1134=7×162
なので、
196個の正方形Aのうち、162個を正方形Bに入れ替えれば(ry

372 名前:あぼーん投稿日:あぼーん
あぼーん

373 名前:あぼーん投稿日:あぼーん
あぼーん

374 名前:132人目の素数さん投稿日:03/05/09 03:00
ゲ〜ムするでつ。
http://f8.aaacafe.ne.jp/~testest/game/plus1index.html

375 名前:132人目の素数さん投稿日:03/05/10 12:15
暇なので問題を出しておく
m階立てのビルがあるとします
そのビルの全てのエレベーターはn個の階で止まるようになってます
さて、この時どの階からも1回エレベーターに乗るだけでどこの階にでもいける
ような最小のエレベーターの個数をmとnを用いて表せ。
かなり前どっかのスレで見たんだが答えが出てなかったようなので。

376 名前:132人目の素数さん投稿日:03/05/10 19:52
>全てのエレベーターはn個の階で止まるようになってます

これが理解できん・・・

377 名前:132人目の素数さん投稿日:03/05/10 20:56
>>376
例えばn=3だったら
2階と3階と5階の3つの階だけに止まるってことじゃないかな


378 名前:132人目の素数さん投稿日:03/05/10 21:10
数列 {θ[n]} は公比 r(0<r<1) で、Σ[n=1,∞]θ[n]=π/2 であるような等比数列とする。
次の条件(@)〜(B)で点P[n]を定めるとき、lim[n→∞]P[n-1]P[n]/P[n]P[n+1] を求めよ。
ただし、P[n-1]P[n]は点P[n-1]から点P[n]までの距離を表わすものとする。

条件
(@) 定点OからOを含まない定直線 l へ下ろした垂線の足をP[0]とする。
(A) n≧1 のとき、∠P[n-1]OP[n]=θ[n]
(B) P[0], P[1], P[2], …… は l 上の点でこの順番に並んでいる。


結果が自分には面白かった。

379 名前:376投稿日:03/05/10 21:43
>>377
ああ、そういうことか。
サンクス。チュッ

380 名前:132人目の素数さん投稿日:03/05/11 01:24
>>378
正直、問題すら理解できない。俺って馬鹿だな〜♪

381 名前:おはよ投稿日:03/05/11 07:19
>>375
(m-1)/(n-1)ですね
新しく繋がる階はエレベータの設置によって
n個,n-1個,n-1個,…と増えていくので、解をxとすると
n+(n-1)(x-1)=m
よってx=(m-1)/(n-1)

382 名前:132人目の素数さん投稿日:03/05/11 07:29
>>381
解説の意味が分からないけど、間違ってると思う
n=2の場合。必要なエレベータの数は C(m,2)でしょう。

383 名前:おはよ投稿日:03/05/11 07:36
>>382
>1回エレベーターに乗るだけで
か。
ごめんして

384 名前:おはよ投稿日:03/05/11 07:40
n=3の場合、
i,j,k階にとまるエレベータは
n=2の場合の
i,j階に止まるエレベータ、j,k階に止まるエレベータ、k,i階に止まるエレベータ
の組と同値である。
ひとつのn=3エレベータで3つのn=2エレベータの代わりをするので、
n=3のときC(m,2)/3
同様にして全てのnにつき
C(m,2)/C(n,2)=m(m-1)/n(n-1)

385 名前:あぼーん投稿日:あぼーん
あぼーん

386 名前:132人目の素数さん投稿日:03/05/11 08:22
>>384
「同値」としてしまうのには、少し無理があるよ。

387 名前:bloom投稿日:03/05/11 08:23
http://homepage.mac.com/ayaya16/

388 名前:132人目の素数さん投稿日:03/05/11 17:45
>1回エレベーターに乗るだけ
これだとその階を除いた(n-1)種類の階にしか行けないと思うのは私の勘違い?

389 名前: ◆BhMath2chk 投稿日:03/05/11 18:00
>>6
 Σ_{1≦n}Σ_{0≦k<p}(1/(pn−k)−1/pn)
=Σ_{1≦n}(1/pn+1/(pn−1)+...+1/(pn−p+1)−1/n)
=lim_{m−>∞}(Σ_{1≦k≦pm}(1/k)−Σ_{1≦k≦m}(1/k))
=lim_{m−>∞}(log(pm)+C+o(1/m)−log(m)−C−o(1/m))
=lim_{m−>∞}(log(p)+o(1/m))
=log(p)。

>>299
黒の方が大きい。
N=0のとき1でNが増えると
白が有利になっていくが1/2にはならない。

>>384
m=4,n=3のとき3。


390 名前:132人目の素数さん投稿日:03/05/11 20:13
>>388
1回エレベーターに乗るって書いてあるから
乗ってエレベーターを動かさずに降りれば
その階にも行けますよ。

391 名前:132人目の素数さん投稿日:03/05/12 01:14
そんな小話はいらないからさ、

392 名前:132人目の素数さん投稿日:03/05/12 17:36
最近問題が難しすぎるよう
もっとヒラメキでパパッっと解ける問題出して!

393 名前:132人目の素数さん投稿日:03/05/12 20:16
そんなの面白くない

394 名前:132人目の素数さん投稿日:03/05/13 01:25
京大(文系)2001年度後期の問題を簡単にしたものです。

n個の1とn個の-1からなる数列 a[1],a[2],…,a[2n] に対して、数列 b[1],b[2],…,b[2n] を
 b[k]=(ka[k]+Σ[j=1,k]a[j])/2 (k=1,2,…,2n)
で定める。このとき集合
 {b[k]|1≦k≦2n}
を求めよ。

395 名前:132人目の素数さん投稿日:03/05/13 01:39
今日、塾で生徒の問題集にあった問題。

1/a + 1/b + 1/c = 1/(a+b+c) のとき、
nが奇数ならば 1/(a^n) + 1/(b^n) + 1/(c^n) = 1/{(a+b+c)^n} を示せ。

どうかなぁ

396 名前:132人目の素数さん投稿日:03/05/13 02:37
ヒラメキでパパっと解ける問題。
x[i]>0(1≦i≦n) かつ Σ[i=1,n]x[i]=π/2
のとき Σ[i=1,n]sinx[i]>1 を証明せよ。

397 名前:132人目の素数さん投稿日:03/05/13 02:48
>>393なんだが、>>396はちょっと面白かった。ごめん。

398 名前:132人目の素数さん投稿日:03/05/13 03:40
組み合わせに対して、n+1Cr=nCr+nCr-1 が成り立つ。

〈p,q,r〉=(p+q+r)!/p!q!r! について同様の関係式はあるか?
さらに、〈p1,p2,,,,,pn〉=(p1+p2+・・・・+pn)/p1!p2!・・・・pn!
ではどうか。

399 名前:あぼーん投稿日:あぼーん
あぼーん

400 名前:132人目の素数さん投稿日:03/05/13 04:24
<p,q,r> = <p-1,q,r>+<p,q-1,r>+<p,q,r-1>
<p[1],…,p[n]> = Σ[i=1〜n] <…p[i-1],p[i]-1,p[i+1]…>

401 名前:132人目の素数さん投稿日:03/05/13 04:26
>>396
Σ[i=1,n]sinx[i]>1

等号もありえるよ
x[1]=π/2

402 名前:132人目の素数さん投稿日:03/05/13 04:42
>>396
さっぱりヒラメかん (´д`;)

403 名前:132人目の素数さん投稿日:03/05/13 06:13
>>402
図を書くとほとんど自明。証明は帰納法。

404 名前:あぼーん投稿日:あぼーん
あぼーん

405 名前:132人目の素数さん投稿日:03/05/13 11:11
数学の問題で簡単にプログラムできる問題を探しています。
例えば、Collatz予想のように

nが偶数ならば2で割る
nが奇数ならば3倍して1を足す

この操作を続けると必ず1になる
という様に、プログラム能力が低くても簡単に作成できて
面白い結果が得られるものが良いです。
どなたかご紹介お願い致します。

406 名前:132人目の素数さん投稿日:03/05/13 12:25
>405さん

知っており実際に授業で使っているものをいくつか。


A君は所持金の半分を常に賭けるものとする。
勝った場合は賭け金の倍の金額を受け取り、負けた場合は賭け金を没収されてしまうという条件の下、
元金1億円で100回勝負をし50勝50敗となった時の所持金(と途中経過)をシュミレーションしなさい。(これは既出スマソ)



調和数列の無限和は発散することが知られている。
http://www.junko-k.com/cthema/15zeta1.htm
調和数列の和の途中経過を表示するプログラムをつくりなさい。



ある数学教授は、学生の成績にゲタをはかせるうまい方法を考えた。
成績の平方根(ルート)を取り10を掛けるというものである。
「これだと100点は100点になるし、合格点は60点以上から36点以上になるし、
損する人は誰もいないんだよ」と教授は言う。
0〜100点が、それぞれ何点になるか示せ。



(2^n - 2) がnで割り切れるなら、nは素数である、
という定理(?)を使って、50までの素数をすべて求めよ。
(反例は341ではじめて見つかる)

407 名前:132人目の素数さん投稿日:03/05/13 16:15
>>406
面白い問題を有難う御座いました。
早速やってみて出来ました。
ありがとう御座いました。

408 名前:132人目の素数さん投稿日:03/05/13 16:30
>>406
> (2^n - 2) がnで割り切れるなら、nは素数である、
> という定理(?)を使って、50までの素数をすべて求めよ。

割り切れないときのことを言ってないから問題の与え方としてはどうかねえ。

409 名前:132人目の素数さん投稿日:03/05/13 20:09
>>401
悪かった。条件にn≧2を付け忘れた。

>>403
帰納法使わなくても一瞬で解ける技があるよ。
解答は5行でOK。

410 名前: ◆BhMath2chk 投稿日:03/05/13 21:00
>>396
0<x<π/2のとき(2/π)x<sin(x)だから
1=(2/π)Σx(i)<Σsin(x(i))。


411 名前:132人目の素数さん投稿日:03/05/13 21:15
>>408>
>(2^n - 2) がnで割り切れるなら、nは素数である、
> という定理(?)を使って、

これがわからない。
nが素数なら、(2^n - 2) はnで割り切れる  じゃないの?
nが素数なら、2^n=(1+1)^n≡1^n+1^n=2 (modn)で、

(2^n - 2) がnで割り切れるなら、nは素数である
の証明ができない。かといって反例も見つからない。誰か教えてください。


412 名前:396投稿日:03/05/13 21:55
>>410
…確かにそれはいいな。俺の用意していた答えより鮮やかだ。
以下俺の答え。>>410の前には霞んで見えるがな。

0<x<π/2 で y=cosx は正かつ単調減少。
したがって、S[k]=Σ[i=1,k]x[i]、S[0]=0として
Σ[i=1,n]∫[0,x[i]]cosxdx > Σ[i=0,n]∫[S[i],S[i+1]]cosxdx
=∫[0,π/2]cosxdx
よって示せた。


これだと>>410と違って「x[i]>0ならばΣsinx[i]>sinΣx[i]」も示せる。
(あまりにくやしかったのでささやかな抵抗を)

413 名前:132人目の素数さん投稿日:03/05/13 21:57
">"の直後のΣは[0,n-1]だった。

414 名前:132人目の素数さん投稿日:03/05/13 22:01
たぶん前にも書いたけどここまで酷い店も無いと思うのでもう一回。
店は北海道砂川市にあるラーメン屋。
いくら脅されていたとはいえ辞めるとなかなか言えなかったのはすごい敗北感を持ってしまった。
とにかく怒り方が異常で脅しと嫌味で人にものを覚えさそうとする。勤務時間は朝九時から夜九時。
パートの主婦が三人、三時であがるとすぐに店長は奥の部屋に行って寝る。夕方六時まで。
俺の休憩時間は食事の30分だけ。ずっと立ちっぱなし足が痛くて少しでもしゃがめば
「なにさぼってんだコラ」辞める三日前には包丁を投げつけられた。
店の前に置いてた(鍵をかけずに)車を盗まれてのり回されてぶつけられた時なんか
警察に文句たらたら言ってた。「早く犯人見つけろや!」
鍵もかけてない自分が悪いくせに。
ここの店長は典型的な井の中のかわずみたいな人間だった。
自分が中心に世界が回ってると思ってるみたいだった。
俺の叔父が死んだ時、明日、葬式行かせて欲しいといったら
「本当に死んだのか?」だって。パートの主婦も
「皆、そういう嘘ついてずる休みする人が多かったから・・」だって。
俺は一回も今までこの店で働いてきてずる休みなんかしたことなかったのに。
何とか午後から行かせてもらえたけど電話で
「早く帰って来い。葬式くらいで休んだらアンタの信用無くすよ」だって。
でも女のバイトには突然三日休ませて東京にコンサート行かせてた。
女にはメチャクチャ甘かった。
12時間働いてる俺よりも8時間勤務の女のバイトのほうが給料が五万くらい高い。
辞めて半年・・・。今はもう立ち直ったけど
働いてるときは本当に店の中で自殺してやろうと思った。
今は叔父の墓には手をあわせに行ってる。
北海道砂川市のラーメン屋・・・・。
俺は一生この店を忘れないと思う。過ちは繰り返さないと心に誓った。

415 名前:412投稿日:03/05/13 23:34
ささやかな抵抗は多分に間違いを含んでいるので忘れてやって下さい。

416 名前:名無しさん! ◆DwIDVPyFJA 投稿日:03/05/14 02:38
一人が行なえる行動は、部屋に入ることと出ること、及び待機の3つのみ。
行動単位は1時間(1時間に一度、人が出入りする)
ただし、一度部屋に入った人は2時間経過するまでは部屋から出られない。
一人が部屋に入れる回数は3回。
部屋の定員は10人。

60時間の期間の間でn人の人間が出入りする場合
・部屋に入れなくなる(定員に達する)確率を求めよ
・誰かが部屋に入れなくなる確率が10%になる人数(n)を求めよ

417 名前:132人目の素数さん投稿日:03/05/14 02:46
>>416
入室・退室・室内待機・室外待機の確率分布は?

418 名前:名無しさん! ◆DwIDVPyFJA 投稿日:03/05/14 02:51
>>417
すべて等価。
合計n人の任意の行動によります。

419 名前:132人目の素数さん投稿日:03/05/14 05:53
面白い問題おしえて〜な

420 名前:132人目の素数さん投稿日:03/05/14 05:58
>>410>>412
君たち! (・∀・)イイヨイイヨー スゴクイイッ!
また面白い問題を作ってくれ!

421 名前:あぼーん投稿日:あぼーん
あぼーん

422 名前:132人目の素数さん投稿日:03/05/14 06:17
>>421
真性馬鹿

423 名前:132人目の素数さん投稿日:03/05/14 19:59
フィールズ賞受賞者が発見した
初等幾何の定理
ナポレオンの定理の拡張で
n角形の外側に頂角を360/nとする2等辺3角形を作る
ごめん
この後忘れた

似たような作業をn-2回やると正n角形ができるっていう定理なんだけど

424 名前:132人目の素数さん投稿日:03/05/14 21:01
>>423
正5角形から5星形を作るようなイメージじゃ?

425 名前:132人目の素数さん投稿日:03/05/14 21:15
>>424
それは絶対にない

426 名前:132人目の素数さん投稿日:03/05/14 22:37
はい、ありません

427 名前:132人目の素数さん投稿日:03/05/15 06:43
>>424
おまえは今までに一体いくつのクソレスをつけてき…(以下略)

428 名前:132人目の素数さん投稿日:03/05/16 02:27
集合{0,1,2,…,101}から、17つの要素を持つ部分集合を102個取り出します。
これらの部分集合をS(1),S(2),…,S(102)とし、
異なる自然数1≦i,j≦102に対して
集合S(i)∩S(j)の要素数をf(i,j)とおきます。
i,jが上記の条件を満たしながら動くとき、f(i,j)の最大値をMとします。
Mの最小値を求めてください。

429 名前:132人目の素数さん投稿日:03/05/16 02:37
>>428
なんか懐かしい気がするのは気のせい?

430 名前:132人目の素数さん投稿日:03/05/16 03:01
17つ という表現をみて思い出したんだが、
長いこと10つとか書いてた某教授の板所が
実は、10コと書いてたことを半年経って気づいたことがあったよ。

431 名前:132人目の素数さん投稿日:03/05/16 08:03
>>429
なつかしいね

>>428
MathNoriの問題を出すな! (もとは数オリだがな…)
自分の力で解けや、糞ヴォケがぁ!
このうすぎたない阿呆がァァァーーッッ!!

432 名前:132人目の素数さん投稿日:03/05/17 21:09
>>428
Mの最小値は17

証明)
 Mはf(i,j)の最大値
 定義から、f(i,j)≦17
 i=jのとき、常にf(i,j)=17
 ∴Mの最小値は17
 (証明終)

i≠jという条件が抜けているんじゃないの?(笑)>>428

433 名前:132人目の素数さん投稿日:03/05/17 21:11
>>432
俺の目には「異なる自然数」ってはっきり書いてあるように見えるぞ。

目が付いてないんじゃない?

434 名前:132人目の素数さん投稿日:03/05/17 21:16
>>432
いいから、さっさとしぬぇぇぇぇ〜!

435 名前:132人目の素数さん投稿日:03/05/17 21:21
条件は抜けていないで、>432のおつむの具合が抜けていますた。

436 名前:132人目の素数さん投稿日:03/05/17 22:11
パナウェーブ研究員以外で白色が好きな人。
http://plaza16.mbn.or.jp/~satchel/omoshiroi/

437 名前:432投稿日:03/05/18 11:02
・・・・・逝ってきます

438 名前:432投稿日:03/05/18 11:07
>>428
つーか、10くらいなんじゃないの?

439 名前:132人目の素数さん投稿日:03/05/19 17:04
無限の高さを持つ、三角柱を考える。
この三角柱を高さ方向に垂直な平面で切ったときの断面は正三角形である。

この時、
「いかなる形の三角形も、三角柱の切断方法を工夫することにより、
その切断面に現れる事がある」
という文章が正しいかどうか、確認せよ。

440 名前:132人目の素数さん投稿日:03/05/19 21:47
>>439
正しい っと思ったが。














論点は、角度かな?

441 名前:132人目の素数さん投稿日:03/05/20 00:13
角度とか…

442 名前:132人目の素数さん投稿日:03/05/20 09:55
>>439
任意の三角形と合同なものは作れないけれど、
相似なものなら作れると思うナ。
でも、エレガントな証明はいまだ思いつかず。

443 名前:442投稿日:03/05/20 09:55
ごめん、ログの一番下までちゃんと読んでなかった。

444 名前:132人目の素数さん投稿日:03/05/20 10:00
>>441
懐かしい……。角度とか。

445 名前:132人目の素数さん投稿日:03/05/20 10:31
鈍角三角形むりぽ

446 名前:三角柱投稿日:03/05/20 11:29
>445
鈍角でもできるよ。
正三角形の断面のこちらがわに辺がくるように
便宜上考えて、むこうがわの点をA、半時計回りにB,Cとする。
ABを含む面に無限にBよりしたにB’をとりAB’の線を引き
同様にCの無限うえにC’をとってAC’をACを含む面上に引けば
角B’AC’は限りなく180度に近づくので鈍角三角形もOK!!


447 名前:132人目の素数さん投稿日:03/05/20 22:04
次の条件を満たす三角形の形を求めよ。

条件
「どんな形の楕円も、回転、平行移動、拡大縮小を繰り返せば
その三角形に内接させる事ができる。」

448 名前:132人目の素数さん投稿日:03/05/20 22:07
どうやって解くんだろ?
角度とか…

449 名前:132人目の素数さん投稿日:03/05/20 22:09
ただ単に 「角度とか…」 って言いたかっただけだろう? あん?
   .:´ ̄::ヽ
  !::;.w''w;::〉  
  |(l|┬ イl   | | ガガガガガッ
  |ll| ヮ ノ   | |      
 と    )    | |
   Y /ノ    人
   / )    <  >_Λ∩
  _/し' //.  ( `Д´)/ ←
>>448
 (_フ彡          /

450 名前:132人目の素数さん投稿日:03/05/20 22:10
>>447
反例が見つからん。
答えはどんな三角形でもOK?

451 名前:132人目の素数さん投稿日:03/05/20 22:18
>>450
よく考えてください、角度とか…

452 名前:132人目の素数さん投稿日:03/05/20 22:24
にとうへんさんかくけい

453 名前:132人目の素数さん投稿日:03/05/20 22:32
>>447の日本語が怪しいということで

454 名前:132人目の素数さん投稿日:03/05/20 22:38
>>447
任意の三角形Sに対して, 長径と短径の比がa:1 (a≧1)であるような
楕円を内接させることが出来る.

証明:
適当なxy座標系の上にSを取る. 行列
a 0
0 1
で表される一次変換fとし, 三角形S'=f(S)を考える.
S'の内接円をΓとすれば, f^{-1}(Γ) は Sに内接する楕円であり
長径と短径の比は1:aである.

455 名前:132人目の素数さん投稿日:03/05/20 23:29
これで、>>439も解けるよな、
>>447は誘導のつもりだった。

456 名前:132人目の素数さん投稿日:03/05/20 23:59
三角形ABCがある。
AB,BC,CAの中点を、それぞれ、P,Q,R,とする。
この三角形に、三点P,Q,R,で内接する楕円が存在することを示せ。

。。。。
>>439
の誘導としてはこっちの方が適切だと思う。
  「シュタイナー 楕円」

457 名前:132人目の素数さん投稿日:03/05/21 01:59
>>439
流れ変わってるけど高校の範囲で解いてみたので書きます

三角柱の辺と切断面の交点の高さを0,p,q、
三角柱の2辺の距離を1、
切断面にできる三角形の辺の長さをa,b,c(a≦b≦c)とすると、
a=√((p-q)^2+1)
b=√(q^2+1)
c=√(p^2+1)
辺の長さ比1:m:n(1≦m≦n)に相似な三角形を求められたとすると、
a/1=b/m=c/nとなる。

a^2=(p-q)^2+1、b^2=p^2+1、c^2=q^2+1
より、
(p-q)^2+1 = (p^2+1)/m^2 = (q^2+1)/n^2

これから
p^2 + m^2(q+2)p + 1 +(1-2q)m^2 = 0 …(1)
q^2 + n^2(p+2)q + 1 +(1-2p)n^2 = 0 …(2)
・・・続く

458 名前:457投稿日:03/05/21 02:00
いかなるmに対しても(1)を満たすpが存在するには
D = m^4(q+2)^2 - 4m^2(1-2q) ≧ 0
(q + 4/m + 2)^2 - 20/m^2 - 16/m ≧ 0
q>0より、この式はq=0のときに成り立てば十分で
(4/m + 2)^2 - 20/m^2 - 16/m ≧ 0
これは1≧1/m^2より常に成り立ち、
(1)を満たすpが存在する。
同様の理論でいかなるnに対しても(2)を満たすqが存在することもいえる。

よっていかなる辺の長さ比1:m:nが求められても
それと相似な三角形を作る切断位置0,p,qは存在する。
〜完〜


459 名前: ◆qHeRghWv.g 投稿日:03/05/21 02:32
三角形ABCとし、角Aが一番大きいとする。
Bを辺上で動かして、Aと同じ高さにとると、二等辺三角形になる。
この時の角B=角Aをαとおくと、Bを動かすことによって
角Bがα以下の任意の角度を取ることが出来る。
また、このとき、点Cがどの位置にあっても、αは60度以上であるから、
いかなる場合においても、角Bは60度以下の任意の角度を取ることが
出来ることがわかる。
対称性により点Cについても同じことがいえる。
ゆえに、題意は示された。

460 名前: ◆qHeRghWv.g 投稿日:03/05/21 02:37
「いかなる場合においても」よりも、「点Cの位置に関わらず」の方が
わかりやすいね。

461 名前: ◆qHeRghWv.g 投稿日:03/05/21 02:45
穴がありました。
・点Bと点Cを同時に動かさないと駄目
・二つの角が60度以下とは限らない
退散します。

462 名前:山崎渉投稿日:03/05/22 00:24
━―━―━―━―━―━―━―━―━[JR山崎駅(^^)]━―━―━―━―━―━―━―━―━―

463 名前: ◆qHeRghWv.g 投稿日:03/05/22 18:05
>>439
また新しい証明を考えてみました。

ある三角形ABCが存在するとする。
3点A'、B'、C'を取り、直線AA'、直線BB'、直線CC'が互いに
平行となるようにする。このとき、互いの距離のうち、
直線BB'と直線CC'の距離が最短として良い。
三角形の存在する平面上にA'から下ろした垂線の足をA''とおくと、
ベクトルAA''はベクトルABとベクトルACの成分に分解できる。
ベクトルABの成分が小さいほど∠BAA'が90度に近づき、大きいほど
0または180度に近づく。
これと同様のことが、ベクトルACの成分と∠CAA'の関係に
ついても言える。
ここで、互いが平行であるという条件を保ったまま、3点を連続的に動かす
時を考える。
このとき、∠BAA'が90度に近づくほど、直線AA'と直線BB'の距離が長くなり、
0度か180度に近づくと、距離は限りなく0に近づく。
これと同様のことが、∠CAA'と直線AA'と直線CC'の距離の関係に
ついても言える。
これらの変化は全て連続的で、直線BB'と直線CC'の距離も連続的に変化
するから、3点を適当な位置に動かせば、互いの距離を直線BB'と直線CC'の
距離にまで短くできる。
このとき直線に囲まれた領域は無限に長い三角柱となっている。
このことより、どの三角形ABCとも相似の三角形が作れることが示された。

464 名前: ◆qHeRghWv.g 投稿日:03/05/22 18:19
ベクトルの成分のあたりに、また微妙に穴がありますね。
大体の方向性は良いのではないかと思いますが。どうでしょうか。

465 名前:132人目の素数さん投稿日:03/05/23 00:37
長さが一定の弦ABを持つ円Γにおいて、弧ABを三等分する点のうちでAに近いほうをPとする。
A,Bを通るように円Γの半径を変えたとき、∠PABはある値を超えないことを示せ。

他すれでも出したけど誰も解かないんでカキコ

466 名前:132人目の素数さん投稿日:03/05/23 00:43
>>465
意味が分からない。とりあえず角度だから、180°を越えないって言うことにしておくか。

467 名前:132人目の素数さん投稿日:03/05/23 00:48
>>465
無限大に発散するってか?

468 名前:132人目の素数さん投稿日:03/05/23 00:53
弦って言うから、別の問題が思いついたよ。

二点A,Bがあり、その距離は2である。
A,Bを両端とする曲線が存在し、その曲線の長さはπである。
A,Bを結ぶ曲線と、直線の二つによって囲まれる部分の面積の最大値を求めよ。

469 名前:132人目の素数さん投稿日:03/05/23 08:11
>>465
120°かな

>>468
π/2

470 名前:132人目の素数さん投稿日:03/05/23 11:11
表の面に1,1/2,1/3…と書いてあるカードをその順番にずっと並べてゆく。
(1)表が出る確率pのコインを投げて、n回目に裏が出たら
 n枚目のカードを裏にするという作業を繰り返す。
(2)サイコロを転がしていき、n回目までに出た目の和をSnとして、
 Sn枚目のカードを裏にするという作業を繰り返す。
カードの裏の面には0が書いてあるとして、作業が終わった時
n枚目上に向けられている面に書いてある数をanとする。
このとき行った作業が(1),(2)それぞれである場合についてΣanが発散しない確率を求めよ。

471 名前:132人目の素数さん投稿日:03/05/23 11:18
n枚目上→n枚目の上ね。

例えば(1)の場合、コインを投げていって表、裏、表、表、裏、表…となった場合は
Σan=1+0+1/3+1/4+0+1/6+…で、
(2)の場合、出た目が1,3,2,1,4,2,…となった場合は
Σan=0+1/2+1/3+0+1/5+0+0+1/8+1/9+1/10+0+1/12+0+…って事ね。

全部裏だったり全部1だった場合はもちろんΣan=0と収束するし、
全部表だったり全部2だった場合はΣanは∞に発散する。

472 名前:132人目の素数さん投稿日:03/05/23 17:18
完全な球体と面が接する時の面積は、球体の大きさによって面積が変わるか。

473 名前:132人目の素数さん投稿日:03/05/23 18:52
>>472 問題文の意味が分かりません。

474 名前:132人目の素数さん投稿日:03/05/23 19:09
確かに難しいところがあるのかもしれません。
参考になればうれしいです(^^)↓  ↓
http://tmp.2ch.net/company/kako/1005/10055/1005519978.html .


...

475 名前:132人目の素数さん投稿日:03/05/23 21:08
>>472
無限小から、πr^2まであるぞ

476 名前:132人目の素数さん投稿日:03/05/23 21:14
>>472
〜〜時の面積を求めよ、

とは書いてあるけど、どう読んでも何の面積を求めるかは書いてないな。

〜〜〜の時、明らかに面積1の三角形の面積は1である。
よって、答えは1。

我ながらエレガントな解答だ。

477 名前:あぼーん投稿日:あぼーん
あぼーん

478 名前:472投稿日:03/05/23 21:50
スマソ。
球体と面が接した時の接地面積。
接してる部分(点)は、球体の大きさによって変わるか、って事。
                        .   ξ ξ. ξ
スマソ。粗茶ですが、これで許してくだされ。⊃旦旦旦

479 名前:132人目の素数さん投稿日:03/05/23 21:56
>>478
0だろ

480 名前:132人目の素数さん投稿日:03/05/23 22:32
                        .   ξ ξ. ξ
スマソ。粗茶ですが、これで許してくだされ。⊃旦砒の


481 名前:132人目の素数さん投稿日:03/05/24 06:38
0<x≦y≦z を満たす x,y,z が任意に与えられたとき、
√(x-k)+√(y-k) = √(z-k) を満たす正数 k < a が存在する。
真か偽か。

482 名前:132人目の素数さん投稿日:03/05/24 07:09
>>481
aってなんだ?

483 名前:132人目の素数さん投稿日:03/05/24 10:28
正数 k < a が存在する。

484 名前:132人目の素数さん投稿日:03/05/24 11:41
k < x の間違いです。とほほ

485 名前:132人目の素数さん投稿日:03/05/24 12:03
>>481
(x,y,z)=(1,1,100)とかだと存在しないんじゃ?
0<k<1 より、左辺<2、右辺>√99



486 名前:132人目の素数さん投稿日:03/05/24 12:36
>>485 正解。
では x,y,z が 0<x≦y≦z かつ x+y≧z の場合は?

487 名前:132人目の素数さん投稿日:03/05/24 12:59
>>486
f(k) = √(x-k) + √(y-k)
g(k) = √(z-k) とする。

f(0) = √x + √y > √(x+y) >= √z = g(0)
f(x) = √(y-x) <= √(z-x) = g(x)

よって、0<k<=x に、f(k)=g(k) となるkが存在



あってるかな?(ドキドキ

488 名前:132人目の素数さん投稿日:03/05/24 21:13
史上稀に見る基地害が現れました!
  (\          /)
  \\       //
    \\    //
  ,,,,,,,,,,,\\,,/// ̄ ̄ ̄ ̄ ̄ ̄ ̄ ̄ ̄ ̄ ̄ ̄ ̄ ̄ ̄
〜′,,,,,,,,,,ミ,,・ω・彡<もぴゅはモナー板を救うキリストとして神によって創られたもぴゅ!  
 UU"""" U U    \_______________
  (\          /)
  \\       //
    \\    //
  ,,,,,,,,,,,\\,,/// ̄ ̄ ̄ ̄ ̄ ̄ ̄ ̄ ̄ ̄ ̄ ̄ ̄ ̄ ̄
〜′,,,,,,,,,,ミ,,・ω・彡<もぴゅを信じる者だけが救われるもぴゅ!  
 UU"""" U U    \_______________
http://aa.2ch.net/test/read.cgi/mona/1053532472/l50
試しに何か書きこんでみて下さい 決まって荒らしは来るなもひゅと言います

489 名前:パπヤ投稿日:03/05/24 21:20
ある本の問題の改題。
リア厨なんでこんな糞問しか出せんw

A氏は10匹犬を所有していた。
A氏の犬のうち、5匹が黒か灰色、3匹が茶色か白、残り2匹の色は黒、灰色、茶色、白のいずれかである。
ある日、何者かの悪戯により、3匹が撲殺された。
そして、茶色い犬の死体が1体庭に放置されていた。
その犬の死体はA氏の犬の死体であった。
さて、この時同じ色の犬の数が最小である場合、自分と他の犬が同じ色だと言える犬は何匹いるか。

490 名前:132人目の素数さん投稿日:03/05/24 21:31
最後の行の
>同じ色の犬の数が最小
の意味がよくわからん。

491 名前:パπヤ投稿日:03/05/24 21:36
>>490
例えば、
色鉛筆が4本あって、赤、青、緑の三色だとするとどれか一つの色が2本で他の色は一本ずつ
みたいな感じですw
その条件だと、赤3本、青1本とかだとダメです。

492 名前:132人目の素数さん投稿日:03/05/25 05:47
>>487 ナイス。ところでこれがどこから出てきたかというと、
0<x≦y≦z かつ x+y>z のとき中間値の定理を使った >>487 より
√(x-k) + √(y-k) = √(z-k) を満たす 0<k≦x が存在するので、
a = √((x-k)/k),b = √((y-k)/k),c = a + b = √((z-k)/k) とすれば、
√(a^2+1) = x/k,√(b^2+1) = y/k,√(c^2+1) = z/k を満たす。

この a,b,c を使って3つの点
A=(0,0,0),B=(1,0,a),C=(1/2,√3/2,c) をとれば、c=a+b より
AB:AB:CA = √(a^2+1):√(b^2+1):√((a+b)^2+1) = x:y:z となる。
よって >>439 が示された。

493 名前:132人目の素数さん投稿日:03/05/25 09:36
>>489
言えない

494 名前:パπヤ投稿日:03/05/25 17:05
>>493
正解です。
では、犬が日本語を喋れるとすれば、答えはどうなるか。

495 名前:132人目の素数さん投稿日:03/05/25 19:08
>>494
残った7匹全員「言える」

496 名前:パπヤ投稿日:03/05/25 19:22
>>495
正解です。
こんなつまらない問題出してすいませんですた。

497 名前:132人目の素数さん投稿日:03/05/25 19:58
そういうクイズとかは趣味板のクイズスレでやった方がいいぞ。

498 名前:132人目の素数さん投稿日:03/05/25 23:49
Aが3枚、Bが2枚の硬貨を同時に投げるとき、A、B2人とも
同数の表を出す確率は?

499 名前:132人目の素数さん投稿日:03/05/26 00:07
カードが9枚テーブルの上にある。それぞれ表に出ている色は、赤2枚、白3枚、黒4枚。

「これらのカードは、片面が白なら、その裏は赤である」という命題の真偽を確認するために何枚のカードを裏返すことが必要かつ十分か?

500 名前:132人目の素数さん投稿日:03/05/26 00:15
>>499
7枚で十分だが…
命題が偽の時には一枚めくっただけでわかることもあるから
必要ってのは微妙だな。

501 名前:132人目の素数さん投稿日:03/05/26 00:15
>>499 7枚

502 名前:499投稿日:03/05/26 01:22
>命題が偽の時には一枚めくっただけでわかることもあるから
必要ってのは微妙だな。

するどい指摘。「何枚必要か?」なら、「1枚」とか「0枚」とか言われそうだし
「真であると確認するに十分な枚数の最小値は?」ならいいかな?

503 名前:132人目の素数さん投稿日:03/05/26 01:28
>>498
0枚1通り 1枚6通り 2枚3通り
で、13(1/2)^5=13/32

504 名前:499投稿日:03/05/26 01:31
で、答えを書き忘れたけど、7枚で正解です

505 名前:132人目の素数さん投稿日:03/05/26 17:37
問題

漢字「幸」に2本の直線を加えると別の全く違う漢字になります。
この漢字を求めて下さい。

ただし、「倖」ではありません。



506 名前:132人目の素数さん投稿日:03/05/26 17:53
>>505
南かな?

507 名前:132人目の素数さん投稿日:03/05/26 18:04
>>506
あたり、、分かる人は早いんだけど、分からない人は
ぜんぜん分からないよ、、PC使ってる分直ぐ分かっちゃうのかな、

508 名前:132人目の素数さん投稿日:03/05/26 20:21
>>505


509 名前:132人目の素数さん投稿日:03/05/26 21:06
幸二

510 名前:132人目の素数さん投稿日:03/05/26 21:13
倖幸執鐸報圉懌摯擇澤睾繹蟄譯贄逹釋驛鷙A凙啈圛墊墿婞
嬕嶧悻慹斁曎檡歝殬燡瓡盩盭睪礋籜緈縶蓻蕔蘀蠌褺襗醳鸅

511 名前:132人目の素数さん投稿日:03/05/26 21:53
>>510
がんばりすぎ(w

512 名前:132人目の素数さん投稿日:03/05/26 22:50
直線って無限になga(略

513 名前:132人目の素数さん投稿日:03/05/26 22:52
この問題のどこが数g(ry

514 名前:132人目の素数さん投稿日:03/05/27 07:11
幸革尭弆南耍蚩者達圉逹違枽遭遼喜逵遠道報遍阜薘関闥懌饟囊横壙壤攘鑨驖灑壩

515 名前:132人目の素数さん投稿日:03/05/31 06:54
11

516 名前:132人目の素数さん投稿日:03/06/05 19:33
すまん。>>503がよくわからん。
表2枚になるのはAが3/8、Bが1/4で3/32
表1枚になるのはAが3/8、Bが1/2で3/16
表0枚になるのはAが1/8、Bが1/4で1/32
全部足して5/16じゃダメなのか?


517 名前:132人目の素数さん投稿日:03/06/05 19:42
>>516
そんであってる。503がおかしいね。

518 名前:132人目の素数さん投稿日:03/06/05 19:44
>>516
>>503は1+6+3=13と計算ミスしただけだと思う。


519 名前:yu-king投稿日:03/06/05 21:00
>>40を解説していただけませんか?

520 名前:yu-king投稿日:03/06/05 21:18
2つの袋A、Bが用意されてます。
どっちかの袋にはどっちかの袋の二倍の金額が入っているらしい。

さて、Aの袋をあけると10000円入っていた。
で、このままこの10000円を持ち帰ってもいいんだけど、
Bの袋と交換することもできる。(もちろんBの金額はまだわからない)

さぁ、取り替えるべきでしょうか?


期待値を考えてみる。
Bに入ってる金額は20000円かもしくは5000円。
その確率はともに1/2だから、Bに取り替えることで得られる金額の期待値は、

20000×1/2 + 5000×1/2 = 12500円 > 10000円

じゃあ常にこれは取り替えた方が得ってことですか??
それでいいんですかねぇ?
なんか、 = にならないとおかしな気がしません?
どうでしょう?

√(20000×5000)=10000
20000×(1/2) + 0×(1/2)=10000

ガ答えになってますけどどう言う意味ですか…?


521 名前:132人目の素数さん投稿日:03/06/05 22:31



522 名前:132人目の素数さん投稿日:03/06/05 22:40
>>520
>Bに入ってる金額は20000円かもしくは5000円。
>その確率はともに1/2だから

なぜ1/2なんだ?
間違ってるよ。

523 名前:132人目の素数さん投稿日:03/06/06 02:58
>>520
3年前にその問題のスレッドがたって丸一年後に解決している。

もっと単純な形にする。カードの裏に自然数が書いてある。各自然数が同じ確率で書かれているとすると期待値は∞。でもカードをめくると有限の値。
これは、各自然数が同じ確率で書かれているという仮定がありえないからおかしな結果になる。
各自然数の確率がpが与えられたら、全事象の確率の和=∞

524 名前:132人目の素数さん投稿日:03/06/06 06:22
>522
二分の1ではないのですか。
同確率じゃあないのか。

525 名前:132人目の素数さん投稿日:03/06/06 07:11
>>524
ttp://www.u.arizona.edu/~chalmers/papers/envelope.html

526 名前:132人目の素数さん投稿日:03/06/06 08:39
>>524
一見そう見えるが根拠がない。
(5000,10000)の組か、(10000,20000)の組かという話にカンゲンされるが、
これが同確率で現れるという保証はどこにもない。
(あるというなら言ってみよ)


・・・ところで還元と換言どっちが適切かな?

527 名前:132人目の素数さん投稿日:03/06/06 09:19
>>526
換言。この場合単に言い替えであって、より基本的なものに還っているわけではない。

528 名前:132人目の素数さん投稿日:03/06/06 12:58
>>523
解決してたんですか。知らなかった
激しくガイシュツに載せたいので、誰かそれようにまとめてくれると嬉しいです。

529 名前:132人目の素数さん投稿日:03/06/06 13:04
一応俺の認識としては・・・

出てくる数字が[1,∞)で一様分布の場合、有限の値が出てくることはありえないので、

有限の値が出てきた
→ 一様分布でない(高い数字ほど出てきにくい等) or 有限の値しか出てこない

よって、分布が分からない限り、2倍になる確率と1/2になる確率が等しいとは言えない

って感じです。
ただ話が難しくて自信がないので書いてませんでした。
こんな感じでOKなんでしょうか?
1行目についてももう少し詳しく説明する必要があるかな・・・

530 名前:132人目の素数さん投稿日:03/06/06 20:32
http://science.2ch.net/test/read.cgi/math/1013324981/603-623
モナーせんせもこれで四苦八苦してた。

531 名前:132人目の素数さん投稿日:03/06/07 10:52
e^π > 22 を なるべくエレガントに示せ!
なお、エレガントの定義は各人に任せる

532 名前:132人目の素数さん投稿日:03/06/07 11:06
一辺が12cmの正方形の折り紙ABCDがある。
辺BCの中点をMとして、頂点Aが点Mに重なるように
折り曲げるとき、重なり合う部分の面積を求めよ。

なるべく図を描かないでといてみてくださいな。
ちなみに解く方法は何でもあり。

ちなみに答えは"105/2"だが、どう解くか。

533 名前:132人目の素数さん投稿日:03/06/07 13:11


534 名前:132人目の素数さん投稿日:03/06/07 14:45
おもしろい問題出せや

535 名前:132人目の素数さん投稿日:03/06/07 16:45
封筒のパラドクスから確率を取り除いたバージョンは知ってるかな?

正数x,yがあり、他方が他方の二倍になっていることだけ分かっている。

命題:
xがyより大きい場合のx-yの値は、yがxより大きい場合のy-xの値よりも大きい

証明:
x=2yかx=y/2のどちらかである。
x=2yの場合、x-y=y
x=y/2の場合、y-x=y/2
明らかにyはy/2より大きいので命題は示された。

問題:
この証明の誤りを指摘せよ。

536 名前:あぼーん投稿日:あぼーん
あぼーん

537 名前:132人目の素数さん投稿日:03/06/07 16:55
>>535
y >y/2
↑  ↑
これとこれはべつのyだからこういう不等式は意味がない。

538 名前:132人目の素数さん投稿日:03/06/07 19:38
>>531
どーやって証明すんねん?

539 名前:132人目の素数さん投稿日:03/06/07 21:04
エレガントというかむしろ下品だが、

e^πすなわち良いパイは美乳、巨乳(爆乳も含む)、貧乳の3通りが考えられるが、xの胸囲をB(x)で表すと
B(巨乳)≧B(美乳)≧B(貧乳)
であり、B(貧乳)>22は自明である。よって示せた。


スマン、書いててつまらなかった。

540 名前:yu-king投稿日:03/06/07 21:28
高校生です。>>40が解からない、っていったんですけど、
その解説が今ひとつどうもわかりません…。
学校の先生に聞いたら、おまえが説明してみろって言われて
授業中にみんなに問題出したら
75パーセントの生徒が取り替えたほうがいいっていうんです。
>カードの裏に自然数が書いてある。各自然数が同じ確率で書かれているとすると期待値は∞。でもカードをめくると有限の値。
の問題もいみがよくわかりません。
自然数全部が書かれたカードが自然数全部分の枚数あったら、
期待値は2分の無限大ですよね?カードの半分2階てある数値は
無限大の半分より小さいし、
もう残り半分にかいてある数値は
全ては無限大の半分より大きいはずですよ。
>(5000,10000)の組か、(10000,20000)の組かという話にカンゲンされるが、
>これが同確率で現れるという保証はどこにもない。
>(あるというなら言ってみよ)
とありますが、保証してみたらどうなるでしょうか。
(n/2、n)か(n、2n)の組みのどちらかが半々の確率で入っているとします。
保証されているときに、最初n円入っていたら、やっぱり期待値的に交換しますよ。
でもそれだと自分が最初に選んだほうは必ず期待値小さくて、後者が必ず大きくなる。

やっぱりおかしい…。
友達にこの問題を出したら、無限的に考えちゃって
「もし最初に入ってたのが1円なら…5銭か2円だな。
5銭ってレアだし、俺は交換するよ」って言われました。こいつ面白いですね。







541 名前:132人目の素数さん投稿日:03/06/07 22:12
>>539
その自明なところを、ゼミで教授に指摘されたら どう答える?

542 名前:132人目の素数さん投稿日:03/06/07 22:17
>>539
見るだけの爆乳、
めったに見られない美乳
触れる貧乳

あなたなら、どっち?

543 名前:あぼーん投稿日:あぼーん
あぼーん

544 名前:( ゚Д゚)ハァ?投稿日:03/06/07 22:26
ある村人が旅をしていました。すると、二つの分かれ道に行きました。
一つの道には本当のことを言う鬼もう一つの道には嘘をつく鬼が居ます。
どちらかの道にはその旅人が行こうとしてる村があります。鬼には一つだ
け質問が出来ます。さて、どうやったら旅人の行く村に行けるでしょうか?


545 名前:132人目の素数さん投稿日:03/06/08 00:08
>>540
期待値ではなく確率の総和で考えると良いのでは?
Nを1以上の整数の集合とし、二つの封筒を区別すると、
根本事象全体は Ω = {(n,2n)|n∈N} ∪ {(2n,n)|n∈N} と書ける。
Ωの上の確率を考えるとき、もし各元が等確率 p であるなら、
{(n,2n)|n∈N} と {(2n,n)|n∈N} は排反なので確率の総和は
P(Ω) = Σ[n=1〜∞] p + Σ[n=1〜∞] p = ∞ ≠ 1 となり確率の定義に反する。
逆に p=0 とすれば、P(Ω) = 0 となるのでこれは確率の定義に反する。
ゆえに等確率であることはあり得ない。

また例えば対数分布などを仮定することで (n,2n) と (n,n/2) を
等確率に出来るがそのような特別な分布を仮定する根拠もない。

546 名前:132人目の素数さん投稿日:03/06/08 00:21
>>544
鬼が十分な知識を持っていなければ、鬼が返す1つだけの答えからは決定不能である。

547 名前:132人目の素数さん投稿日:03/06/08 00:49
自然対数を死ぬほどわかりやすく説明せよ。

548 名前:132人目の素数さん投稿日:03/06/08 01:02
(ln(x))' = 1/x となるように選ばれた底を持つ対数

549 名前:132人目の素数さん投稿日:03/06/08 01:33
>>540
無限大の半分は無限大

550 名前:132人目の素数さん投稿日:03/06/08 01:56
>>529
> 出てくる数字が[1,∞)で一様分布の場合、有限の値が出てくることはありえないので、

金額の期待値が発散することは有限の金額が出てこないことを含意しない。
だから>>523>>545のように全事象の確率に帰着させるのは良いが、
期待値の発散に帰着させる論法は説明として不完全である。

551 名前:yu-king投稿日:03/06/09 23:37
無限大の半分は無限大はわかっております。
しかし無限大の半分は無限大より小さいのは自明でしょうに。

仮定する根拠が無いなら
「期待値が一致するはずである」という根拠で言ってみたらどうでしょうか。
う〜ん、こうしてみたらどうだろう
20000円の確率をpとし、5000円の確率を1-pとする。

すると20000p+5000(1-p)=10000
計算するとp=3分の1

だから、20000円の確率が3分の1。
5000円は3分の2。
ああ、これでもおかしい、っておもうでしょ。
確率が確かに変な割合で現れる事になる。

これが封筒の矛盾ですか…。(EnvelopePaladox)低級高校3年でした。

552 名前:132人目の素数さん投稿日:03/06/09 23:51
>>551
無限大は数じゃない。
大小を比較するのはまずい。

553 名前:132人目の素数さん投稿日:03/06/10 00:24
四角形ABCDにおいて
AB=28,BC=21,CD=5
∠ABC=∠ACD=∠Rのとき
∠BADを求めてください。

554 名前:132人目の素数さん投稿日:03/06/10 00:24
↑必死だな

555 名前:132人目の素数さん投稿日:03/06/10 11:35
>>551
> しかし無限大の半分は無限大より小さいのは自明でしょうに。

a[n] = n,b[n] = 2n とするとき Σ[n=1〜∞] a[n] < Σ[n=1〜∞] b[n]
とでもいうのかな?

> だから、20000円の確率が3分の1。
> 5000円は3分の2。
> ああ、これでもおかしい、っておもうでしょ。

なぜ?

556 名前:132人目の素数さん投稿日:03/06/10 11:48
>>551
> しかし無限大の半分は無限大より小さいのは自明でしょうに。

小さくないぞ。

557 名前:132人目の素数さん投稿日:03/06/11 12:37
最初のすれ
http://cheese.2ch.net/math/kako/970/970737952.html
>>3にある問題答え出てないけど
そんなNは存在しないであってる?

558 名前:132人目の素数さん投稿日:03/06/11 13:20
問題;6つの連続した整数がある。(最小の数をNとする)
これらを2つのグループに分ける時、
それぞれに入っている数の積が等しくなるように分割できるNの値を
全て求めよ。

559 名前:132人目の素数さん投稿日:03/06/11 13:30
存在しない。証明できる。

560 名前:132人目の素数さん投稿日:03/06/11 13:31
>>557
取りあえず6つの数字の約数に7が一個だけ含まれてる場合は不可だから、
49より小さい数では、

1、2、3、4、5、6
8、9、10、11、12、13
・・・
43、44、45、46、47、48

だけに候補が絞られるな。
でもこれじゃ49以上が絞れないから、アプローチ悪いかも。

561 名前:132人目の素数さん投稿日:03/06/11 13:33
Nは5の倍数であることが必要

562 名前:132人目の素数さん投稿日:03/06/11 13:34
六つの数はすべて2と3と5の冪で書けていなければならない

563 名前:560投稿日:03/06/11 13:35
んなことないか。
49以上でも絞れるや。
7の倍数が含まれてる連続6整数を分けようとすると、片方だけが7の倍数になっちゃうんだ。

564 名前:560投稿日:03/06/11 13:40
>>562
そうだね。
7,11,13・・・なんかが含まれてる数字は6整数の中に高々一個しかないから駄目、と。

565 名前:132人目の素数さん投稿日:03/06/11 13:46
よしもうちょいだガンバレ。

566 名前:132人目の素数さん投稿日:03/06/11 14:08
N+1,N+2,N+3,N+4の中で奇数は2つ。
>>561-562より
その2つの奇数は両方とも3の累乗である。
それを満たすのは、1と3の場合しかない。
しかし、この場合6つの数字の中に0が必ず含まれる。
よって、存在しない。

567 名前:560投稿日:03/06/11 14:14
>>566
おみごと!

568 名前:560投稿日:03/06/11 14:16
頑張ったのに役立てなかった・゚・(ノД`)・゚・。

569 名前:132人目の素数さん投稿日:03/06/11 14:20
>>558
六つの連続した数に7の倍数が含まれていない時を考える。
そのとき、これらの数を7で割ったあまりは
1,2,3,4,5,6
になる。

さて、今これらの数を全てかけたとき、それをMとして7で割ると
余りは6になる。

また、Mは平方数なので、7で割ったあまりは
1,2,4
のどれか。
よって、これはあり得ない。


また、六つの連続した数に7の倍数が含まれているときは
たかだか一つしか含まれていないので、
二つに分けたとき、どちらかに7の倍数が含まれてしまうので、
片方が7の倍数。片方がそうじゃない。
よって、これは矛盾。。。。。


570 名前:132人目の素数さん投稿日:03/06/11 14:20
あぅ。。。既にこんなにレスがついてる。。。

571 名前:132人目の素数さん投稿日:03/06/11 14:28
解決したのを知ったうえで
エレガントな別解を示してくれたのかと思ったよ。

572 名前:132人目の素数さん投稿日:03/06/11 14:29
>>568
ヨシヨシ( ・ω・)ノ(ノД`)・゚・。

573 名前:132人目の素数さん投稿日:03/06/11 15:13
>>568
オラオラ( ・ω・)σ---------c< ノД`)・゚・。 イタイヨ…

574 名前:あぼーん投稿日:あぼーん
あぼーん

575 名前:132人目の素数さん投稿日:03/06/11 16:55
教育実習に行った時、工房に、
「メビウスの輪にフタって出来るの?」
と聞かれて、
「出来ないことはない」
って適当なことを答えてしまった。
どうなんでしょうか・・・?

576 名前:132人目の素数さん投稿日:03/06/11 17:03
>>575
3次元ユークリッド空間の中で,自己交差を持たないように
ふたをすることは不可能

577 名前:132人目の素数さん投稿日:03/06/11 17:16
マジかよ!

578 名前:132人目の素数さん投稿日:03/06/11 17:34
「出来ないことはない」ってのは、
より高次元の空間なら可能っていう意味なのかも

579 名前:132人目の素数さん投稿日:03/06/11 17:37
>>575-578
フタってどういうことだ?

580 名前:575投稿日:03/06/11 18:12
>>579
たとえば、トイレットペーパーの芯の両端にフタをして、
筒→円柱にするイメージ。
「4次元空間ではできるよ」と言おうとしたけど、本当かどうか
わからんかったから、誤魔化したのです・・・。

581 名前:あぼーん投稿日:あぼーん
あぼーん

582 名前:132人目の素数さん投稿日:03/06/11 20:40
>>580
>わからんかったから、誤魔化したのです・・・。

やれやれ、適正がないですね。
さっさと歯車に加わることをお勧めします

583 名前:132人目の素数さん投稿日:03/06/11 21:31
完全に出遅れたが、自分なりに>>558の解答を考えた。

連続した6つの整数の中に0を含む場合は題意のような分割が出来ないのは明らかだから、自然数の範囲で考えれば十分。
6つの連続した整数には5の倍数が少なくとも1つあるのでNが5の倍数であることが必要。
このとき題意の分割が可能であるためにNとN+5は別のグループに分けなければならないが、
(N+5)(N+1)(N+2)-N(N+4)(N+3) = (N+5/2)^2+10-25/4 > 0
であるからそのような分割は出来ない。

584 名前:132人目の素数さん投稿日:03/06/12 00:27
>>561
>>583
「6つの連続した整数には5の倍数が少なくとも1つある」
これはわかる.でも4の倍数も6の倍数も少なくとも1つある.
なぜNが5の倍数であることが必要になるのか
わかんねーよヽ(`Д´)ノウワァァァン


585 名前:132人目の素数さん投稿日:03/06/12 00:49
>>584
Nが5の倍数のときは、N+5も5もの倍数になる。
もしそうでなかったら連続する6整数の中には5の倍数は一つしかない。
これだとこれらを二つに分けたとき片方の積は5の倍数にるのに
もう一方の積は5の倍数でなくなる。
だから5の倍数でないといけない。

586 名前:132人目の素数さん投稿日:03/06/12 00:50
×倍数にるのに
○倍数になるのに

587 名前:他スレ既出であるが投稿日:03/06/12 01:33
次の方程式が表す図形を座標平面に図示せよ。(ただしひとつの平面に書き込むこと)

x^2+y^2=1

x^2+y^2=4

y=±x (−4≦x≦−3,3≦x≦4)

y=0 (−4≦x≦−3,3≦x≦4)

x=0


588 名前:132人目の素数さん投稿日:03/06/12 01:41
>>587
穴が見えます

589 名前:132人目の素数さん投稿日:03/06/12 02:13
>>558関連
ちょっと待ったー!皆さん平方数に関してはどう考えてますか?
たとえば>>561の条件、私も最初に思いついたんですが、間に
5*5=25が挟まる場合に証明が思いつかないんです。



590 名前:132人目の素数さん投稿日:03/06/12 02:26
>>589
Nが5の倍数ってのは「必要条件」だよ。
問題なし。
もっかい考えてみ。

591 名前:590投稿日:03/06/12 02:29
いいや。解説しちゃお。

例えばNが23だったらどうか。
23,24,25,26,27
ほら駄目でしょ?

(ってか、たぶんあなたは問題を勘違いしてるな。25を分割するのは不可ですよ。)

592 名前:589投稿日:03/06/12 02:54
ぐはぁ!
失礼しました……
(それで条件を緩めて「6つの積が非0な平方数になる場合があるか」ゴニョゴニョ)

593 名前:132人目の素数さん投稿日:03/06/12 15:24
自然数nに対し、関数f(n)を次のように定める。
f(n)
=(2^nを10進数展開し、各位の数を全て加える。)

例えば、
f(5)=3+2=5
f(10)=1+0+2+4=7
f(16)=6+5+5+3+6=25
などのように定義する。

このとき、
lim[n->∞]f(n)=∞
を示せ。


大昔の数学セミナーをみてたら載ってたので出題してみた。
1985年か、1995年の問題だったと思う。
間違ってたらごめん。

594 名前:132人目の素数さん投稿日:03/06/12 15:30
零の出現個数を上から押える方法があるのだろうなあ

595 名前:132人目の素数さん投稿日:03/06/12 16:02
フェルマーの小定理使ったら解けそうな気がしないでもないなぁ

596 名前:132人目の素数さん投稿日:03/06/12 16:03
オイラーの拡張版のことね。

597 名前:132人目の素数さん 投稿日:03/06/12 16:05
>>595
小(ry

598 名前:yu-king投稿日:03/06/12 23:59
>>552 確かに無限大は記号であるから大小比較は出来ないが
現実、半分のほうが小さい。
>>555
>a[n] = n,b[n] = 2n とするとき Σ[n=1〜∞] a[n] < Σ[n=1〜∞] b[n]
>とでもいうのかな?
とあるがそういうわけではない。上の不等式は成り立たない。
しかし、無限大の半分は元のものより必ず小さくなっている。
そうでないなら
無限大を2の無限大乗で割ったらどうなるか。
小さくならない「半分にする試行」を何度もすると小さくなっていく。これは矛盾する。
背理法により無限大の半分は元のものより必ず小さいコトが証明される。
>> だから、20000円の確率が3分の1。
>> 5000円は3分の2。
>> ああ、これでもおかしい、っておもうでしょ。

>なぜ?
では一万円と5000円の組になる確率は一万円と20000円の組になる確率の2倍といえるのか。
いえない。よっておかしい。

実際に最初N円みせて交換することを何度もすると、もらえる金額の平均はN円より大きい。…。
記号ばかりに頼って、本質を忘れてはいけない。正数の半分はそれより小さい。という本質。




599 名前:132人目の素数さん投稿日:03/06/13 00:09
>>598
> >a[n] = n,b[n] = 2n とするとき Σ[n=1〜∞] a[n] < Σ[n=1〜∞] b[n]
> >とでもいうのかな?
> とあるがそういうわけではない。上の不等式は成り立たない。
> しかし、無限大の半分は元のものより必ず小さくなっている。

じゃあ Σ[n=1〜∞] a[n] と Σ[n=1〜∞] b[n] はどっちが小さいのさ

600 名前:132人目の素数さん投稿日:03/06/13 00:12
>>598
> >> だから、20000円の確率が3分の1。
> >> 5000円は3分の2。
> >> ああ、これでもおかしい、っておもうでしょ。
>
> >なぜ?
> では一万円と5000円の組になる確率は一万円と20000円の組になる確率の2倍といえるのか。
> いえない。よっておかしい。

なぜいえないのですか?

601 名前:132人目の素数さん投稿日:03/06/13 00:23
>>598
自然数全体の集合をNとし、偶数全体の集合をAとする。
Nの元nに対して、2nを対応させることを考える。
こうするとNのどの元にもAの元が必ず一つは対応しているので、
Nの要素の数はAの要素の数より大きくない

ところで、いま奇数全体の集合をBとする。
対応2n -> 2n-1 を考えると
Aの元にもBの元が必ず対応する。
Aの要素の数はBの要素の数より大きくない.

よって(#AはAに含まれる元の個数)
#N = #A + #B >= 2#A >= 2#N
要するに ∞ >= 2∞ です。


>無限大を2の無限大乗で割ったらどうなるか。
>小さくならない「半分にする試行」を何度もすると小さくなっていく。これは矛盾する。
そんなむちゃくちゃな(藁)
極限をとる順番次第。
lim[n->∞] {lim[m->∞] n/(2^m)}=0 だけど
lim[m->∞] {lim[n->∞] n/(2^m)}=∞
大学では無限の概念をきちんと扱うので大学で集合論を勉強してみてください。

602 名前:132人目の素数さん投稿日:03/06/13 00:27
>>598
大小比較ができないのに小さいとは、こはいかに。
現実って何よ?

603 名前:132人目の素数さん投稿日:03/06/13 01:03
>>598
> いえない。よっておかしい。
「よっておかしい可能性もある」では?

604 名前:あぼーん投稿日:あぼーん
あぼーん

605 名前:132人目の素数さん投稿日:03/06/13 02:23
2ちゃんねらは「あなた」などという単語を用いない。

606 名前:132人目の素数さん投稿日:03/06/13 03:40
最初の2行でいきなり笑った。
無限と有限を同列で扱ってる時点で、既にダメなんです。
やっぱり無限大を数として考えてるんじゃ?>598

607 名前:132人目の素数さん投稿日:03/06/13 14:55
たて61、よこ69の長方形を9この正方形に分割せよ
誰かといてちょ


608 名前:132人目の素数さん投稿日:03/06/13 15:02
ボーカンプの表記法で
(36,33) (5,28) (25,9,2) (7) (16)
ボーカンプの表記法については下記参照
ttp://www.geocities.co.jp/CollegeLife-Labo/6317/seihoukei.htm

609 名前:132人目の素数さん投稿日:03/06/13 15:26

あるクイズ番組のボーナスゲーム。
3つの箱A、B、Cがあり、その中に一つだけ当たりがあります。
それぞれ当たりが入っている確率はA55%、B15%、C30%です。(なぜか偏りがあります)

司会者は予め当たりの箱を知っていて、
あなたが選ばなかった残りの2箱のうち空の箱を1つ開けるのが恒例となっています。
(2箱共に空の時はランダムでどちらか片方を開けます)

今あなたはAの箱を選びました。
すると司会者はBの箱を開けて、中が空であることを見せてくれました。
ここで司会者お決まりの文句「今なら箱を変えてもいいですよ」
さて、どうするのが得策だろうか?

610 名前:132人目の素数さん投稿日:03/06/13 15:36
> ランダムでどちらか
これが 50% : 50% という意味なら、変えた方が当たる確率は高い。
それより、このルールを事前に知っているなら、最初に選ぶ箱は
Bであるべき。

611 名前:132人目の素数さん投稿日:03/06/13 15:38
>>609
ガイシュツじゃないのか?

612 名前:132人目の素数さん投稿日:03/06/13 15:42
>>610
さすが数学板。

>>611
確率が違うタイプはさほどガイシュツではないかと・・・
ガイシュツならスマソ

613 名前:132人目の素数さん投稿日:03/06/13 16:01
>>612
問題よく読んでなかった、スマン

614 名前:132人目の素数さん投稿日:03/06/13 16:04
等しい問題 http://science.2ch.net/test/read.cgi/math/1040540700/74
解答例 http://science.2ch.net/test/read.cgi/math/1040540700/105-106

615 名前:609,612投稿日:03/06/13 16:18
みんなひっかからないなあ・・・
まあどうせここで釣れるとは思ってなかったから別にいいんだけどね。

でさ、一般人に出すときはどういう風に確率を設定したら驚くかな?

例えば、Aを選び、Bが開けられるとして、
A55%、B15%、C30% → 変えた方が得
A45%、B35%、C20% → 変えない方が得

どういうのがいいんだろう?

616 名前:132人目の素数さん投稿日:03/06/13 16:25
>>615
数学的に正しい解答を納得出来ない人に出題する場合は工夫しがいがない

617 名前:609投稿日:03/06/13 16:35
>>616
そこをなんとか。

618 名前:132人目の素数さん投稿日:03/06/13 17:02
>>617
なぜ工夫しがいがないかというと大抵の人は正解を示しても驚かんからだ

619 名前:132人目の素数さん投稿日:03/06/13 17:17
>>618
三枚の赤青カードの話なんかは、「1/2でしょ?」という錯覚があるから驚くわけだよね。

この問題の場合、どうすれば素人にうまく錯覚を起こさせることができるかな。

620 名前:132人目の素数さん投稿日:03/06/13 18:12
A:90%, B:10%, C〜L:1%
C〜Lを全部開ける
とか。

621 名前:132人目の素数さん投稿日:03/06/13 19:27
110%?

622 名前:132人目の素数さん投稿日:03/06/13 20:26
とりあえず、数列に関する問題を三題連続で
次の三つの漸化式を解き、a(n)の一般系を求めよ。

1. a(1)=sqrt(3) a(n+1)=( a(n) )^2 + 2*a(n)
2. a(1)=sqrt(3) a(n+1)=( a(n) )^2 - 2*a(n)
2. a(1)=sqrt(3) a(n+1)=( a(n) )^3 - 2*a(n)


それと、亀だが
>>583へ。
間違ってるよ。6つの数を三つずつに分けるとはどこにも書いてない。

623 名前:620投稿日:03/06/13 20:40
間違えた。A:80%でした。

624 名前:132人目の素数さん投稿日:03/06/13 22:56
ある国では、死刑囚が死刑執行されるとき、「死ぬ前の一言」を言うことが許される。
その内容が正しければ斬首刑、違っていれば絞首刑にされることになっている。
そこで、ある死刑囚は最後に「私は絞首刑にされる」と言った。
その死刑囚は何刑にされただろうか

625 名前:132人目の素数さん投稿日:03/06/13 23:00
首を絞められた上に斬られた

626 名前:132人目の素数さん投稿日:03/06/13 23:01
>>622
1.
a_{n+1} + 1 = (a_n + 1)^2
だから a_n + 1 = (a_1 + 1)^(2^{n-1})

2, 3 はどうしよう…。
なにか一般的な方法があるのかな。

627 名前:132人目の素数さん投稿日:03/06/13 23:02
三人の囚人がいた。その三人をA,B,Cと呼ぼう。
そのうち二人は死刑だそうだ。
しかし、誰が死刑なのかは三人とも知らないが、
死刑である確率は同じである。つまり、A,B,Cが死刑になる確率は2/3である。
死刑が実行される前日になって、Cが監守に、A,Bのどちらが死刑になるかを
尋ねた。(A,Bのどちらか又は両方は死刑になるのである。)
すると監守はBは死刑になると答えた。
(Aについては触れていない)
Cは、もう一人は自分かAのどちらかが死刑になる、つまり自分が死刑になる確率
は1/2になったと喜んだという。(つまり2/3が1/2になったのである。)
これは正しいのか。また、間違っているならどこがおかしいのか。


628 名前:132人目の素数さん投稿日:03/06/13 23:06
>>627
たぶん既出

629 名前:132人目の素数さん投稿日:03/06/13 23:12
>>628
で答えは?

630 名前:132人目の素数さん投稿日:03/06/13 23:25
>Cは、もう一人は自分かAのどちらかが死刑になる、
>つまり自分が死刑になる確率は1/2になったと喜んだという。
が明らかにおかしい。

631 名前:583投稿日:03/06/13 23:36
>>622
指摘サンクス。では
n(n+1)(n+2)(n+3) > (n+4)(n+5)
n(n+1)(n+2)(n+3)(n+4) > n+5
を示せばいいかな。やってないけど出来るだろうね。

632 名前:132人目の素数さん投稿日:03/06/13 23:38
>>627
条件が足りず答えは出ない。

>Cは、もう一人は自分かAのどちらかが死刑になる、つまり自分が死刑になる確率
>は1/2になったと喜んだという。

が、この推論は明らかに変。

(数学は過程も大事。このCはアホ。)

633 名前:132人目の素数さん投稿日:03/06/13 23:38


634 名前:132人目の素数さん投稿日:03/06/13 23:51
お年玉の袋がふたつある。一方は他方の2倍の金額が入っているという。
Aさんが一つもらって開けてみたら、1000円入っていた。
そうすると、もう一方は500円か2000円入っていることになる。
したがって、他方のお年玉袋をもらったBさんが得る金額の期待値は、
1/2*500+1/2*2000=1250(円)になる。
つまり、後から選んだほうがより多くもらえるであろうとよそうできる。
これはいったいどういうことなのか。


635 名前:132人目の素数さん投稿日:03/06/13 23:54
>>634

>>520あたりから超ガイシュツ

636 名前:132人目の素数さん投稿日:03/06/14 01:10
あなたは1000円持っている。
コインを投げて表なら、1000円あげる。裏だったら500円くれ、という
賭けを持ちかけられる。期待値は+250円。
あなたはこの賭けを受けるか?

637 名前:132人目の素数さん投稿日:03/06/14 01:21
絶対乗る〜

638 名前:132人目の素数さん投稿日:03/06/14 01:27
500円持ってたらやるな。

639 名前:132人目の素数さん投稿日:03/06/14 03:45
半径1の円周上に定点Aがある。次の条件を満たす点Pの存在領域の面積を求めよ。
<条件>
Pは円内の点で、APを1辺とする正三角形APQ,APR(Q≠R)の頂点Q,Rはともに円内又は周上の点である。

640 名前:132人目の素数さん投稿日:03/06/14 06:35
(π/3)-((√3)/2)かな

641 名前:あとむ投稿日:03/06/14 06:49
39 :132人目の素数さん :03/04/09 19:12
2つの袋A、Bが用意されてます。
どっちかの袋にはどっちかの袋の二倍の金額が入っているらしい。

さて、Aの袋をあけると10000円入っていた。
で、このままこの10000円を持ち帰ってもいいんだけど、
Bの袋と交換することもできる。(もちろんBの金額はまだわからない)

さぁ、取り替えるべきでしょうか?


期待値を考えてみる。
Bに入ってる金額は20000円かもしくは5000円。
その確率はともに1/2だから、Bに取り替えることで得られる金額の期待値は、

20000×1/2 + 5000×1/2 = 12500円 > 10000円

じゃあ常にこれは取り替えた方が得ってことですか??
それでいいんですかねぇ?
なんか、 = にならないとおかしな気がしません?
どうでしょう?


40 :132人目の素数さん :03/04/09 19:18
√(20000×5000)=10000
20000×(1/2) + 0×(1/2)=10000


この40の式だけじゃイマイチ分かりません。
どなたか文章交えてなぜこの式になるのか教えてもらえませんか?

642 名前:132人目の素数さん投稿日:03/06/14 10:24
>>622
(1)は既に答えがでている。(3)は実は一定値をとる。
(2)a(n)=2cos( b(n) )+1とおけば、二倍角の公式になる。

643 名前:639投稿日:03/06/14 12:10
>>640
正解。

では条件を「Q,Rの一方が円内または周上の点で他方が円外または周上の点」とすると面積はいくらか。

644 名前:132人目の素数さん投稿日:03/06/14 17:57
>639
複素平面で考えると、単位円上の150度の点を中心とした半径√2の円と、
270度の点を中心にした半径√2の円の、
重なっている部分の面積でいいですか?

645 名前:132人目の素数さん投稿日:03/06/14 20:37
>>644
違うよ。まず、どこを基準として150゚、270゚なのか。そして√2はどこから出てきたのか。

646 名前:132人目の素数さん投稿日:03/06/15 01:57
ごめんなさい。
計算間違えしました。
半径60度の扇形の重なる部分ですね(レンズ形?)
たぶん面積はπ/3-√3/2

647 名前:132人目の素数さん投稿日:03/06/15 01:59
わーい
あってた

二題目いってもよう

648 名前:132人目の素数さん投稿日:03/06/15 02:14
ずばりπ/3+√3/2
でござるね?

649 名前:132人目の素数さん投稿日:03/06/15 02:14
あちがうや
√3

650 名前:132人目の素数さん投稿日:03/06/15 02:42
問題
k=1,2,3・・・n-1 について、
tan(kα)*tan((k+2)α)の総和をnを使った式で表してください。
ただし、α>0,(n+1)α<π とします。

651 名前:132人目の素数さん投稿日:03/06/15 03:30
みんな頭いいなぁ。すごい。尊敬する

652 名前:132人目の素数さん投稿日:03/06/15 04:21
問題の意味からして既にわかりません。

653 名前:132人目の素数さん投稿日:03/06/15 06:36
>>643
2π/3

654 名前:132人目の素数さん投稿日:03/06/15 09:45
>>643
タダ

655 名前:607投稿日:03/06/15 13:31
608さん遅くなりましたがありがとうm__m


656 名前:かかか投稿日:03/06/15 13:44
健君は、円形のコースを一定のスピードで走っています。同じコースで恵子さんも
      一定のスピードで自転車で走っています。
     今、恵子ちゃんは、健君の後方300メートルのところにいるが、健君があと
      500メートル走ったら、ちょうど彼を追い越します。
      さて、この2人がこのペースで走ってると、恵子さんが健君を追い越す地点は
      全部で何ヶ所でしょう?(コースの全長は知らなくてもできます

657 名前:607投稿日:03/06/15 13:46
608の人は計算で出したんですか?解き方もできれば、教えて下さいm__m

658 名前:132人目の素数さん投稿日:03/06/15 14:08
たまには簡単目の問題も悪くないでしょう。

1個以上のさいころを同時に振ったとき、
出た目の合計が偶数になる確率は1/2である。

これをなるべく簡単に示せ。

659 名前:132人目の素数さん投稿日:03/06/15 14:13
ある占い師が自分で『私の占いは当たらない』といった。
さてこれはどういう事か。

660 名前:132人目の素数さん投稿日:03/06/15 14:16
彼は愚痴をこぼしているということだろう

661 名前:132人目の素数さん投稿日:03/06/15 14:18
>>650
(tan(nα)+tan((n+1)α)-tan(α)-tan(2α))/tan(2α)-n+1

662 名前:mathmania ◆uvIGneQQBs 投稿日:03/06/15 14:19
Re:>659
きっと自分自身の占いをしたのだろう。
この場合、『私の占いは当たらない』という文の真偽は問題にされていないので、
パラドックスにはならない。

663 名前:132人目の素数さん投稿日:03/06/15 14:23
一辺が1の立方体を切ったとき。
その断面積の最大値を求めよ。

664 名前:132人目の素数さん投稿日:03/06/15 14:24
>>662
わざわざRe:とか書かずに普通に>>でアンカーしろヴォケ

665 名前:132人目の素数さん投稿日:03/06/15 14:24
>>658
6×6の将棋盤の各位置が(x,y)∈{1,2,3,4,5,6}×{1,2,3,4,5,6}で指定されるとせよ。
x+yが偶数になるところに色を塗れば市松模様になる(上下左右は1だけ違うので色違い)。
6は偶数なので色が塗られたところはマス目全体のちょうど半数ある。

666 名前:661投稿日:03/06/15 14:25
>>650
そのαに関する制約は問題を解くうえで必要ないと思うのだが。
tan(kα)が定義される限りにおいてではあるけど。

667 名前:132人目の素数さん投稿日:03/06/15 14:38
>>593
まず、ある自然数m,A,Bが考え、B<10^mであるとする。
このとき、
2^n=A*10^(4m)+B
を満たすような、n,m,A,Bは存在しないことを示す。
2^n=A*10^(4m)+B>A*10^(4m)>2^(4m)
が成立するため、n>4mが成り立つ。
このことから、2^nは2^(4m)で割り切れる。
また、10^(4m)も2^(4m)で割り切ることが可能。
2^n=A*10^(4m)+B
より、結局Bが2^(4m)で割り切れる。
2^(4m)=16^m>10^m>B
が成立するため、結局B=0である。
2^n=A*10^(4m)+B
に代入すれば、
2^n=A*10^(4m)
が成立する。しかし、Aは自然数なのでこれは矛盾。
2^n=A*10^(4m)+B
は成立しない。
このことから、2^nは
m+1桁目から4m桁目までに必ず0でない数が現れる。
また、2^nの一桁目は0でないため。
1桁目 0でない
2〜4桁目 0でない
5〜16桁目 0でない
……
以下一般に4^(k-1)+1〜4^k桁の間に0でない数が少なくとも一つ現れる。
よって、2^nが4^k以下の場合
f(n)≧k+1
が成立する。このことから
lim[n->∞] f(n)=∞

668 名前:132人目の素数さん投稿日:03/06/15 14:39
生徒Aと生徒Bが100m走をして、Aのほうが10m、Bに差をつけて早くゴールしました。
これを見た先生は、二人が同時にゴールできるようにと、Aのスタート位置を10mだけ、後に下げさせました。

さて、このスタート位置でもう一度走らせたとき、はたして、AとBは同時にゴールできるのでしょうか?

669 名前:132人目の素数さん投稿日:03/06/15 15:19
>>668
AがBに1m先着する。

670 名前:132人目の素数さん投稿日:03/06/15 19:56
>>668
AもBも最初から一定の速度で走りつづけるなら、>>669のようになるかもしれない。
しかし、最初は両者とも止まっているわけで、加速度などを考えないとどうなるかは分からない。
同時にゴールさせたいなら、Bを10m進ませるべき。

671 名前:132人目の素数さん投稿日:03/06/15 19:59
>>663
確か正六角形よりも長方形のときのがでかくなるんだよね。
√2

>>665
サイコロ3個以上の場合はどうすんの?

672 名前:643投稿日:03/06/15 21:05
>>648,649
違うよ。答えは2π/3のはず。以下解答。

Aを原点とする複素数平面を考え、題意の円を |z-1|≦1―(*) で表す。(Pの座標をP(z)とした)
α=cos60゚+isin60゚,β=cos(-60゚)+isin(-60゚) とすれば、Q(zα),R(zβ)(逆もある)だから、題意を満たす点Pの存在領域は
((*)かつ|zα-1|≦1かつ|z-β|≧1)または((*)かつ|zα-1|≧1かつ|zβ-1|≦1)
⇔((*)かつ|z-α|≦1かつ|z-β|≧1)または((*)かつ|z-α|≧1かつ|z-β|≦1)
これを図示すると、求める面積は半径1、中心角240゚の扇形の面積と等しいことがわかる。よって2π/3が答え。

673 名前:132人目の素数さん投稿日:03/06/15 23:31
>672
そっかあ空白のレンズ形部分は扇形からはみ出た部分でちょうど埋まるから639のときの面積を引かなくていいんだね。
はやとちりばっかりだ

674 名前:132人目の素数さん投稿日:03/06/16 04:16
>>658 >>671 >>665
1個のときは明らか。
さてここでn個のときに偶数である確率が1/2であると仮定すると
n+1個めに振ったさいころの(ry

675 名前:132人目の素数さん投稿日:03/06/16 12:49
>>659
「私の占ったことの反対のことが起きる」または
「私の占ったことは起こらない」
という自慢

676 名前:132人目の素数さん投稿日:03/06/16 14:51
死して屍拾う者無し の逆を言え。

677 名前:132人目の素数さん投稿日:03/06/16 14:55
屍拾う者無ければ死んでいる

678 名前:132人目の素数さん投稿日:03/06/16 14:56
腹が減ったならば飯を食う の対偶を言え。

679 名前:mathmania ◆uvIGneQQBs 投稿日:03/06/16 15:02
Re:>678
この手の問題はもう数学者の間では有名だ。
飯を食っているならば、その直前に腹が減っていた。

680 名前:132人目の素数さん投稿日:03/06/16 15:02
死して屍披露野茂7勝目

681 名前:132人目の素数さん投稿日:03/06/16 15:04
同情するならカネをくれ! と言え。

682 名前:132人目の素数さん投稿日:03/06/16 15:05
同情するならカネをくれ!

683 名前:132人目の素数さん投稿日:03/06/16 15:06
同情するならカネをくれ! と何度も言え。

684 名前:132人目の素数さん投稿日:03/06/16 15:07
同情するならカネをくれ!
同情するならカネをくれ!
同情するならカネをくれ!
同情するならカネをくれ!
同情するならカネをくれ!
同情するならカネをくれ!
同情するならカネをくれ!
同情するならカネをくれ!
同情するならカネをくれ!


685 名前:132人目の素数さん投稿日:03/06/16 15:08
同情するならMr.オクレ!

686 名前:132人目の素数さん投稿日:03/06/16 15:09
武士は食わねど高楊枝
渇すれども盗泉の水を飲まず

687 名前:132人目の素数さん投稿日:03/06/16 15:10
祐実タン ハァハァ

688 名前:mathmania ◆uvIGneQQBs 投稿日:03/06/16 15:10
Re:>681
スレタイが読めないのか?
Re:>683
スレッドのタイトルが読めないのか?
Re:>684
いちいち釣られるな。

689 名前:132人目の素数さん投稿日:03/06/16 15:12
>>688
お前スレ違い

690 名前:689へ投稿日:03/06/16 15:16
688mathmania ◆uvIGneQQBs は前々からスレの流れというものを読めないヤシなんでつ。。。
 どうか勘弁してやってくださいでつ。。。。。。。

691 名前:132人目の素数さん投稿日:03/06/16 15:19
これだから初心者は困るよねー
>>683-684の展開は「釣られた」とは言わない

692 名前:132人目の素数さん投稿日:03/06/16 15:20
禿同。

693 名前:132人目の素数さん投稿日:03/06/16 15:23
>>691
しいて言うなら、
「あえて乗ってみた」あたりか?

694 名前:132人目の素数さん投稿日:03/06/16 15:24
>>679
てゆーか、間違ってるしな(藁

695 名前:132人目の素数さん投稿日:03/06/16 15:25
以下の者、しばらくの期間書き込みを慎み、
スレ流れの研究に精進すること。

mathmania ◆uvIGneQQBs

696 名前:massmania ◆uvIGneQQBs 投稿日:03/06/16 15:27
あぁ、間違っている。
飯を食っていないならば、その直前にはらが減っていない。

697 名前:132人目の素数さん投稿日:03/06/16 15:28
天狗になってるヤシの鼻をへし折るスレはここですか?

698 名前:mashmania ◆uvIGneQQBs 投稿日:03/06/16 15:30
天狗は架空の生き物だ。

699 名前:132人目の素数さん投稿日:03/06/16 15:30
名前を変えて登場してるぞ(藁
少しは成長したようだ
父さん嬉しい

700 名前:伊東家投稿日:03/06/16 15:35
ではあらためまして。

同情するならカネをくれ! と何度も言え。

>mashmania ◆uvIGneQQBs
お前この問いやってみれ。がんがれ。


701 名前:matzmania ◆uvIGneQQBs 投稿日:03/06/16 15:36
Q. 写像 f:R→Rが、(通常の位相において)右連続ならば、
fは下限位相において連続であることを示せ。
(この問題はあまり難しくないか?)
Q. 上限位相と下限位相のどちらよりも強い位相は離散位相のみであることを示せ。
Q. 連結で弧状連結でない位相空間の例を挙げよ。

702 名前:132人目の素数さん投稿日:03/06/16 15:37
>>699
ほんとだw
ワラタw
一気に好感度アップw

703 名前:mashmania ◆uvIGneQQBs 投稿日:03/06/16 15:38
Re:>700
お前のパソコンに"同情するならカネをくれ!"と何度も書き込むウイルスを侵入させてやってもいいのだぞ。

704 名前:132人目の素数さん投稿日:03/06/16 15:39
あのぉ、mathmania ◆uvIGneQQBsをNG登録してあるんで、
名前をころころ変えるのやめてくれませんか?

ウザイだけなんで (;^_^A アセアセ…

705 名前:nathmania ◆uvIGneQQBs 投稿日:03/06/16 15:42
Q. y'+ay=0の自明でない解が、実数変数に関して周期的になるように定数aの値を定めよ。

706 名前:132人目の素数さん投稿日:03/06/16 15:43
晒し上げしなくちゃいけなくなりますた。
とても残念な結果でつ。。。

703 :mashmania ◆uvIGneQQBs :03/06/16 15:38
Re:>700
お前のパソコンに"同情するならカネをくれ!"と何度も書き込むウイルスを侵入させてやってもいいのだぞ。




707 名前:132人目の素数さん投稿日:03/06/16 15:44
703 :mashmania ◆uvIGneQQBs :03/06/16 15:38
Re:>700
お前のパソコンに"同情するならカネをくれ!"と何度も書き込むウイルスを侵入させてやってもいいのだぞ。




708 名前:132人目の素数さん投稿日:03/06/16 15:45
703 :mashmania ◆uvIGneQQBs :03/06/16 15:38
Re:>700
お前のパソコンに"同情するならカネをくれ!"と何度も書き込むウイルスを侵入させてやってもいいのだぞ。

709 名前:htammania ◆uvIGneQQBs 投稿日:03/06/16 15:46
Re:706
さっきからずっと上がっていたが。

710 名前:132人目の素数さん投稿日:03/06/16 16:00
自信満々にマジ解答して間違った挙句、他人に「釣られるな!」と言いながら
実は自分1人が釣られまくってる「◆uvIGneQQBs」が
居るスレはここですか?

711 名前:132人目の素数さん投稿日:03/06/16 16:04
もう勘弁してやれよw

712 名前:132人目の素数さん投稿日:03/06/16 16:05
ちょっとした作図問題を出してみる。

平行四辺形ABCDと、その4つとは違う点Eが与えられたとする。
この時定規で直線を引いていく操作だけでABCDと合同であり
Eを頂点とする平行四辺形EFGHを作図する事は可能か?

713 名前:132人目の素数さん投稿日:03/06/16 16:07
自信満々にマジ解答して間違った挙句、他人に「釣られるな!」と言いながら
実は自分1人が釣られまくり、更には他人になりすましてまで自己弁護に必死な
「◆uvIGneQQBs」が居るスレはここですか?


714 名前:132人目の素数さん投稿日:03/06/16 16:08
>>710
つーかどっちもスレ違いのネタずっと続けるのは迷惑なので止めてくれると嬉し。

715 名前:132人目の素数さん投稿日:03/06/16 16:10
Re:>714
お前のパソコンに"同情するならカネをくれ!"と何度も書き込むウイルスを侵入させてやってもいいのだぞ。


716 名前:714投稿日:03/06/16 16:12
いいのだぞ。と、言われてもなぁ、、、

717 名前:712=714投稿日:03/06/16 16:13
>>715
浸入させてもいいから、私の問題を解いてくれぇ。

718 名前:132人目の素数さん投稿日:03/06/16 16:18
可能。ガイシュツ。

719 名前:132人目の素数さん投稿日:03/06/16 16:20
不可能
外出

720 名前:132人目の素数さん投稿日:03/06/16 16:22
化膿 中出

721 名前:mathmania ◆uvIGneQQBs 投稿日:03/06/16 16:46
Q. 1+2=3,1+2+3+4+5+6+7+8+9+10+11+12+13+14=15+16+17+18+19+20,
さて、次に来る式は何か?
(、左辺は1から適当な数nまでの全ての正整数の和で、右辺はn+1からある適当な数までの全ての正整数の和である。)

722 名前:あぼーん投稿日:あぼーん
あぼーん

723 名前:132人目の素数さん投稿日:03/06/16 19:07
A男、B男、C子、D子の4人が、乱交パーティーをする。
A男はC子、D子とB男もC子、D子とせーくすしたい。
病気が怖いので、コンドームは使いたい。
しかーし、コンドームは2枚しかない。
果たしてせーくすは可能か?

724 名前:132人目の素数さん投稿日:03/06/16 19:16
コンドーム重ねて使うんでしょ。
そんなことやるやつ絶対いねー。

725 名前:132人目の素数さん投稿日:03/06/16 19:48
>>723
ttp://mathworld.wolfram.com/GloveProblem.html
にある Hajnal and Lovasz が初出だったと思う。

726 名前:132人目の素数さん投稿日:03/06/16 21:38
>>674
略さんと全部書いてケロ

727 名前:132人目の素数さん投稿日:03/06/16 23:42
>>704
トリップのみをNGワード登録するという知恵がないわけではあるまいな?

728 名前:132人目の素数さん投稿日:03/06/17 00:48
716 :さかな :03/06/17 00:40
ひろし君はすすむ君と2人でジャンケン(アイコがあっても決着がつくまでを1回とする)をして、
一勝でも勝ち越した時点で勝ち逃げでやめようと思っている。

ジャンケンをn回したとき、既に勝ち越してやめている確率は?


729 名前:132人目の素数さん投稿日:03/06/17 01:00
>>728
どっちの?

730 名前:132人目の素数さん投稿日:03/06/17 01:09
>>729
ひろし君に決まってるだろ

731 名前:132人目の素数さん投稿日:03/06/17 01:20
1-(n/2^n)ka?

732 名前:132人目の素数さん投稿日:03/06/17 01:40
>>730
ひろし君「と」すすむ君「は」
に見えたのだ。スマソ

733 名前:あぼーん投稿日:あぼーん
あぼーん

734 名前:132人目の素数さん投稿日:03/06/17 05:39
>>679は有名な問題だということは知っていたが
答えについては知らなかったと見える

対偶の意味をもう少し考えれ

735 名前:132人目の素数さん投稿日:03/06/17 05:49
死して屍拾う者なし の対偶って
屍を拾う者があるならば死んでいない なのか?
なんか変な気もするが…

736 名前:132人目の素数さん投稿日:03/06/17 06:01
「死して屍拾う者なし」 の対偶は
「屍拾う者あるならば死んでいない」である。
そして、死んでいないということは屍はないから、
屍を拾う者がいるのは物理的に不可能である。
これにより、命題「屍拾う者あるならば死んでいない」が
偽であることがわかる。
命題の真偽値はその対偶と一致するので、
最初の命題「死して屍拾う者なし」も偽である。

∴隠密同心が死んでも屍は拾ってもらえる


737 名前:132人目の素数さん投稿日:03/06/17 06:02
いいのだぞ

738 名前:132人目の素数さん投稿日:03/06/17 06:17
隠密同心とクレタ人はうそつきである

739 名前:310投稿日:03/06/17 12:06
「(隠密同心が)死して屍拾う者なし」 の対偶は
「屍を拾う者がいたら、その屍は隠密同心ではない」だと思うんだが。

740 名前:132人目の素数さん投稿日:03/06/17 17:33
「腹が減ったならば飯を食う」の対偶は
「腹が減っても飯を食わなければ隠密同心でない」

741 名前:132人目の素数さん投稿日:03/06/17 17:41
>>740
あー、なんかそれもよく見掛ける > 腹が減ったら

742 名前:132人目の素数さん投稿日:03/06/17 18:16
いまごろなにを… おまいは>>679か?

743 名前:132人目の素数さん投稿日:03/06/17 20:28
「死して屍拾う者なし」 の対偶は
「屍を拾う者がいたら、死んでいない」で正しい。
これは一見奇妙な文章だが、括弧の中を補って考えると
そう無理のある文章でもない。

「屍を拾う者がいたら、(その屍は隠密同心ではないので、
 屍になったと思われた隠密同心は実は) 死んでいない」


744 名前:132人目の素数さん投稿日:03/06/17 22:07
みんな若干ずれてると思われ。

「死して屍拾う者なし」
=「隠密同心が死んだならば、その隠密同心の屍を拾う者はいない」

対偶は
「隠密同心の屍を拾う者がいるならば、その隠密同心は死んでいない」

この対偶は一見奇妙だが、前件が偽なので問題ない。
                ^^^^^^^^^^^^^^^^^^^^^^^^^^

745 名前:132人目の素数さん投稿日:03/06/17 23:57
>>726 >>674(ryの続き
‥‥出目の合計が偶数になるのは、

・n個の合計が奇数で、n+1個目も奇数
・n個の合計が偶数で、n+1個目も偶数

の2パターン。よってその確率は
(1/2)*(1/2)+(1/2)*(1/2)=1/2
以上より、n+1個の時も成り立つ。■

746 名前:132人目の素数さん投稿日:03/06/18 01:03
命題「死して屍拾う者なし」には、隠密同心は含まれないので
この命題の真偽は隠密同心とは独立である。

747 名前:132人目の素数さん投稿日:03/06/18 01:10
>>744が言いたいことはよくわかるが
前件の真偽にかかわらず対偶の真偽は同一でなくてはならない
>>746のいうように隠密同心とは独立に対偶をつくべきであろう

748 名前:132人目の素数さん投稿日:03/06/18 01:11
× 対偶をつくべきであろう
○ 対偶を作るべきであろう

749 名前:744投稿日:03/06/18 01:49
>>746,>>747

なるほど。
確かにちょっとまずかったですね。

750 名前:744投稿日:03/06/18 02:28
2通りの解釈がある。

1.
「死して屍拾う者なし」=「ある特定の人間Aについて、Aが死んだならば、Aの屍を拾う者はいない」
対偶は、「ある特定の人間Aについて、Aの屍を拾う者がいるならば、Aは死んでいない」

2.
「死して屍拾う者なし」=「全ての人間は、死んだならばその屍を拾う者はいない」
対偶は、「『ある屍』を拾う者がいるならば、その人間(屍側)は死んでいない」


これでどうだ?

751 名前:132人目の素数さん投稿日:03/06/18 05:22
どっちにしろ
「屍を拾う者がいるならば屍は死んでいない」
になる。

752 名前:132人目の素数さん投稿日:03/06/18 15:50
「怒られないと勉強しない」の対偶は?

753 名前:132人目の素数さん投稿日:03/06/18 16:29
>>752
勉強してるならその前に怒られてた

754 名前:mathmania ◆uvIGneQQBs 投稿日:03/06/18 16:34
「明日晴れたら空を見よう。」の対偶は?(対偶というのはおかしいか?)

755 名前:744投稿日:03/06/18 16:57
>>754
これ難しいな。
引っかかりそうになる。

「明日晴れたら空を見よう」
=「明日晴れたならば、その日空を見るつもりだ」
=「明日晴れたならば、その日空を見るつもりが今の私にはある」

対偶は、
「明日空を見るつもりが今の私にないならば、明日は晴れない」

756 名前:mathmania ◆uvIGneQQBs 投稿日:03/06/18 17:01
ざんねんだが、755は不正解だ。
「『明日の空を見なかったならばそれはその日が晴れではなかったからだ。』をしよう。」
(これは確かに754の文の対偶ではない。)

757 名前:132人目の素数さん投稿日:03/06/18 17:04
(明日∧晴れ)⇒空を見る

¬空を見る⇒¬明日∨¬晴れ

空を見ないならば、明日でないかまたは晴れではない



758 名前:あぼーん投稿日:あぼーん
あぼーん

759 名前:mathmania ◆uvIGneQQBs 投稿日:03/06/18 17:05
恐らく757が正解のようだ。

760 名前:744投稿日:03/06/18 17:17
>>757
「見よう」の「よう」の部分を無視していいのだろうか?

761 名前:うねうね投稿日:03/06/18 18:58
http://members13.tsukaeru.net/michi/hushigi.gif

これわかる?
なんでこうなるのぉ?(*´д`*)ハァハァ

762 名前:うねうね投稿日:03/06/18 19:05
スレ違いの予感(┯_┯) ウルルルルル

763 名前:132人目の素数さん投稿日:03/06/18 19:07
>>761
http://www.geocities.co.jp/CollegeLife-Club/7442/math/index.html#triangle

764 名前:うねうね投稿日:03/06/18 19:10
なるほどw
騙されたわけだw

765 名前:132人目の素数さん投稿日:03/06/18 23:38
「空を見よう」と思わないならば、(以下同文


766 名前:744投稿日:03/06/19 09:07
>>756
>>755が不正解である理由をもっと詳しく説明してくれませんか?



>>757
「明日晴れたら」の「明日晴れ」は「明日の天気が晴れ」という意味。
「(その日が)明日かつ晴れである」という意味ではない。

よって、これの否定は「明日は晴れではない」だと思うのだがどうか?

767 名前:132人目の素数さん投稿日:03/06/19 09:31
もしA:「明日晴れたら空を見よう」を
B:「明日(晴れているならば空を見ている)に従う予定である」と同義とするならば
Bは仮言命題ではないので対偶を持たない。

768 名前:132人目の素数さん投稿日:03/06/19 10:02
Jをユリウス日とするとき

明日晴れたら空を見よう
≡ 今日がJ日ならばJ+1日の天候が晴れでないかJ+1日に空を見ている

対偶

J+1日の天候が晴れでかつJ+1日に空を見ているならば今日はJ日ではない
≡ 晴れなのに空を見ていない日は明日ではない

769 名前:yu-king投稿日:03/06/19 10:40
599>>等しいです。
601>>ありがとう御座います。このような解答を待っていました。
その他>>無限と有限
無限と有限はどちらが大きいのですか。

問題の解決に自分はどうやらたどり着いたようです。
問題の決着は

・無限数に端点はない

という一言に尽きるでしょう。
無限に広がる空間を二分しても無限の空間で、斬っても斬れない空間のようである。

そのことに基づくと「一万円が出たときは交換したほうがよい。」
が結論。しかしこれをN円が出たとき…という議論へ一般化することは不可能である。
有限数議論を無限数議論へ持っていく事が誤りである。
僕はこう解釈します。
無限大理論の欠陥:
無限大円が出たときはどうなるか。
(現実ではインフレが無限大に起こって日本の円の価値は無くなります。)
無限大円が出たら交換は必要無い。2倍の無限大円も半分の無限大円も同価値。
よってこの無限大議論はこの試行において成立できない。
実際のこの実験ではでうる金額の最大値が存在する。
袋に入っているのだからさいだいでも日本にあるありったけの円。
さらに最大数は偶数である。
その最大値が出たときは、交換すべきでない。
そのときの交換した場合の期待値がとてつもなく
他の金額が出たときのそれよりも小さい。
平均すると期待値は日本円の存在する金額の半分。になる。




770 名前:あぼーん投稿日:あぼーん
あぼーん

771 名前:132人目の素数さん投稿日:03/06/19 11:20
平面上に異なる5つの点がある。
全ての互いに異なる二つの点を直線、または曲線で結ぶとき
必ず、どれか二つの線は交わることを示せ。

772 名前:132人目の素数さん投稿日:03/06/19 11:23
>>769
> 無限数に端点はない

無限数というのは意味不明だがある意味間違ってはいない。
集合論は無限公理というものでそれを保証する形をとる。

> そのことに基づくと「一万円が出たときは交換したほうがよい。」

これは全く意味不明。もしそのように結論するなら次の五つの真偽を理由つきで答えよ。
100円が出たときは交換したほうがよい。
100万円が出たときは交換したほうがよい。
100億円が出たときは交換したほうがよい。
100兆円が出たときは交換したほうがよい。
100京円が出たときは交換したほうがよい。

773 名前:132人目の素数さん投稿日:03/06/19 12:18
>>771
多分これが一番簡単 ttp://www.mathreference.com/gph-planar,k5.html
同じようによく知られた結果だが平面ではなく球面やトーラスの上ではどうか。

774 名前:132人目の素数さん投稿日:03/06/19 12:43
>>769>>771
無限数の定義が分からんが、点集合[0,1]を考えれば端点は存在するが…?

775 名前:132人目の素数さん投稿日:03/06/19 14:04
>>774
後者が一意な全順序なんでしょうな

776 名前:132人目の素数さん投稿日:03/06/19 14:36
>>767
>もしA:「明日晴れたら空を見よう」を
>B:「明日(晴れているならば空を見ている)に従う予定である」と同義とするならば

同義じゃないだろ。

「明日晴れたら空を見る」「つもりだ」 ではなく、
「明日晴れたら」「空を見るつもりだ」 でしょ。

「つもり」がかかっているのは「空を見る」という自分の行動に対してだけであって、
明日の天気には関知していないと解するのが妥当。

777 名前:132人目の素数さん投稿日:03/06/19 14:43
うーん混乱してきた・・・

>>755正解に一票

778 名前:mathmania ◆uvIGneQQBs 投稿日:03/06/19 14:51
「明日晴れたら空を見よう。」は、発言者の意思を表すものなので、これはそもそも命題ではない。
ついでに言うと、「明日晴れるならば、その日に私は空を見る。」
の対偶は、「私が明日空を見ないならば、その日は晴れではない。」となる。

779 名前:yu-king投稿日:03/06/19 18:12
そうだな
明日晴れたら…といっている時点で
それは予定ではなく、晴れたことが解かって見るのであって
待遇は「見ないのなら晴れていない」だ

>> そのことに基づくと「一万円が出たときは交換したほうがよい。」

>これは全く意味不明。もしそのように結論するなら次の五つの真偽を理由つきで答えよ。
>100円が出たときは交換したほうがよい。
交換したほうがよい。円の最大値より十分遠いからである。
>100万円が出たときは交換したほうがよい。
交換したほうがよい。円の最大値より十分遠いからである。
>100億円が出たときは交換したほうがよい。
真偽は問えない。実際そのようなお金が袋に入っていることは無いからである。
>100兆円が出たときは交換したほうがよい。
上記どおり。
>100京円が出たときは交換したほうがよい。
上記どおり。


現実の話が始まっている時点で貴様の負けは確定していたな
残念だ、こんな低脳な数学者が…。他のものが口を出さない理由がわかっていない。
反対するのなら解決したまえ。
非を論ずる事が出来ても真は論ずる事が出来ない
高校生以下の存在に必要性は無い。

少なくともこの板には。



780 名前:132人目の素数さん投稿日:03/06/19 18:36
遅レスだけど

「死して屍拾う者なし」
aを"文脈上明らかになるであろう屍を拾う行為"とすると、
「aをする者はいない」とできる。

ここで、者集合NについてNの任意の要素nの行動をf(n)とする。
すると命題は
「f(x)=a を満たすxはN内に存在しない」
とする事ができ、対偶は

「全てのnについて f(n)≠a である」
=「全ての者の行動にaは無い」
=「全ての者がその屍を拾わない」

781 名前:132人目の素数さん投稿日:03/06/19 19:24
>>779の言ってる事は要するに
全てのnに対してP(n/2)=P(2n)となる確立分布は存在しないよん
ということでは?

782 名前:132人目の素数さん投稿日:03/06/19 21:40
確率分布が存在しないのはその通りだけど存在しない理由が問題かと

783 名前:132人目の素数さん投稿日:03/06/19 21:48
>>781
そういう確率分布は存在しないし、
適当な分布を与えればそれに応じて変えたほうが良いかそのままかの分布が得られるってこと?

784 名前:132人目の素数さん投稿日:03/06/19 21:49
AとBの箱にお金が入っていてBの箱にはAの2倍か半分が入ってるって問題がいまだにわからん

785 名前:132人目の素数さん投稿日:03/06/19 21:59
>>783
「そのままの分布が得られる」というのは書き間違い?

786 名前:132人目の素数さん投稿日:03/06/19 23:14
封筒の問題、ガイシュツページに加えておきましたので
突っ込みがあったらお願いします

787 名前:132人目の素数さん投稿日:03/06/19 23:33
>>786
等確率でないことに証明をつけたほうが良いのでは?

金額の組(a,2a)の確率を P(a,2a) とするとき確率の総和は
Σ[a∈N] P(a,2a) であり,この値は1でなければならない.
もし任意の (a,2a) に対して P(a,2a) が定数 p ≧ 0 なら
Σ[a∈N] P(a,2a) = Σ[a∈N] p となる.右辺は p>0 の
とき発散し,p=0 のとき零になるのでいずれにせよ1にならない.
ゆえに P(a,2a) は定数ではない.すなわち等確率ではない.

788 名前:132人目の素数さん投稿日:03/06/19 23:50
有限の値が出てくる確率は "0"の証明。
もし例えば1000の出る確率がεだとする。
すると例えば2000の出る確率もε、
3000の出る確率もε。
どの数の出る確率もεならば
すべての確率を足すと
εがゼロでないならば無限大に発散してしまう。
しかしすべての確率を足すと
1にならなければならないはずなのでこれは矛盾。
よってεは0でなければならない。

789 名前:132人目の素数さん投稿日:03/06/20 00:27
>>786の封筒の問題と言うのは>>39の問題に関してですか?
違うのであれば、どなたか問題を簡潔に述べてもらえませんか?

790 名前:132人目の素数さん投稿日:03/06/20 00:32
>>789
それのことです

791 名前:132人目の素数さん投稿日:03/06/20 00:33
球体を転がしたときに、北極が上になる確率は0。
球状のどの点についても同様。
だから、確率の総和は0っていう議論はどうなん?

792 名前:791投稿日:03/06/20 00:34
ごめん、かんちがいした

793 名前:132人目の素数さん投稿日:03/06/20 00:37
そういうのを議論してるスレがあったような。

794 名前:132人目の素数さん投稿日:03/06/20 01:26
>>791
勘違いと断ってるのにレスするのもなんだけど、
「だから」以降がおかしいね。

0(=無限小)は、有限個加えても0だが、
無限個加えるとどうなるかわからん。

795 名前:132人目の素数さん投稿日:03/06/20 03:37
>>794と同じことが>>787でも言えないのかな?

796 名前:132人目の素数さん投稿日:03/06/20 04:02
>>795
任意の整数 k に対して a[k] = 0 なら lim[n→∞] Σ[k=1,n] a[k] = 0

797 名前:132人目の素数さん投稿日:03/06/20 04:39
(1)(2)をともに満たす四面体ABCDの体積の最大値を求めよ。

(1)AD⊥平面BCD
(2)BC+CA+AB=4

答えは1/6になるそうなんですがその過程がわかりません。
どうか教えて下さい。


798 名前:132人目の素数さん投稿日:03/06/20 04:54
>球体を転がしたときに、北極が上になる確率

球面上に存在する点の個数
lim[n->∞]n
北極が上になる確率
1/(lim[n->∞]n)
そしてこれは球面上の各点において同様だから、確率の総和は
lim[n->∞]n*1/(lim[n->∞]n)
で表される?
(そんでこれは1であるはずだよなぁ)

・・・するってぇと
「球面上に存在する点の個数」を「数直線上に存在する点の個数」
とした時に2つの封筒の問題と何が違うのか・・・

教えてエロい人

799 名前:132人目の素数さん投稿日:03/06/20 06:28
>>798
「球面上に存在する点」を「数直線上に存在する点」に一対一対応させれるかな?

800 名前:132人目の素数さん投稿日:03/06/20 09:48
らぬきことばいや〜ん

801 名前:132人目の素数さん投稿日:03/06/20 12:37
>>799
できるよ。
まず(0,1]*(0,1]->(0,1]の一対一対応があるのはいいよね?
これを使って実際に対応させる手順を考える。
球面を半分に分けて考える。
南半球からだけど、上からxy平面に射影することで同じ半径の円と一対一に対応する。
その後中心に関して半径が1/√2になるよう相似に拡大(縮小)する。
中心が0になるように平行移動する。
-1/2<=x<=1/2の部分をy方向に、-1/2<=y<=1/2の部分をx方向に適当に
縮めて一辺の長さが1の正方形に一対一に対応させる。x,y両方向に1/2ずつ平行移動。
これで南半球が(0,1]*(0,1]に一対一に対応する。
したがって(0,1]に対応させることができた。

北半球も同様に(-1,0]に対応させれば
球面がが(-1,1]に一対一に対応する。

802 名前:132人目の素数さん投稿日:03/06/20 13:06
>>798 は図形の面積は各点の面積の総和では得られないのは何故か
ということが訊きたいんじゃないかな。

803 名前:132人目の素数さん投稿日:03/06/20 13:29

(1) [1,∞)で一様に数字が出てくる確率分布は存在するか?

(2) (1)が真であるとして、そのような確率分布において、有限の値が出てくることはありえるか?


誰か教えてください。

804 名前:132人目の素数さん投稿日:03/06/20 14:16
>>803
「数字」というのが数つまり実数のことなら存在しない。

805 名前:803投稿日:03/06/20 14:17
(1)について。

例えば、「リングの周上の点を[1,∞)に一対一対応させて、転がす」なんてのは駄目ですか?

806 名前:132人目の素数さん投稿日:03/06/20 14:30
>>805
対応させる方法は?

807 名前:132人目の素数さん投稿日:03/06/20 14:55
>803
(1)のような確率変数 X が存在しない理由は次の通り:

一様なので、P{0≦X<a} = a P{0≦X<1} である。
しかしこれは lim[a→∞] P{0≦X<a} = 1 とは両立しない。
なぜなら lim[a→∞] P{0≦X<a} = lim[a→∞] a P{0≦X<1} より
P{0≦X<1} > 0 ならばこの極限は存在しない(発散する)。
P{0≦X<1} = 0 ならば lim[a→∞] a P{0≦X<1} = lim[a→∞] 0 = 0 となる。
よって P{0≦X<a} = a P{0≦X<1} ならば lim[a→∞] F(a) = 1 ではない。
つまり lim[a→∞] F(a) = 1 ならば P{0≦X<a} ≠ a P{0≦X<1} なので、
X は一様でない。

808 名前:803投稿日:03/06/20 15:11
>>806>>807

1.リング上の一点に印を付けて円周を[0,1)と見なす。
2.転がして静止したときに地面と接する一点は一つの実数を与える。
3.どの実数が出やすいということはないので一様分布。

どこが駄目かな?

809 名前:132人目の素数さん投稿日:03/06/20 15:17
>>808
> 2.転がして静止したときに地面と接する一点は一つの実数を与える。

それは N×[0,1) と [0,∞) の1対1対応であって
[0,1) と [0,∞) の1対1対応ではない。

810 名前:803投稿日:03/06/20 15:47
>>809
ごめんちょっとわかんない。
[0,1) の一様分布も [0,∞) の一様分布も本質的に同じものだと思ってた。

[0,1) の一様分布はあるけど、[0,∞) の一様分布はないってことですか?

811 名前:132人目の素数さん投稿日:03/06/20 16:26
 アイウエ
×    オ
─────
 エウイア

ア〜オは一桁の自然数で、同じ全て値が異なっている。
つまり筆算の穴埋めです。ア〜オの値を求めよ。

812 名前:132人目の素数さん投稿日:03/06/20 16:47
2178*4=8712ですな

813 名前:132人目の素数さん投稿日:03/06/20 16:51
>>812
正解。
これクイズ本から出したんだけど、
答えしか載ってないから途中式を知りたい。

814 名前:132人目の素数さん投稿日:03/06/20 20:07
>>810
その疑問は >>807 によって解決されるべきだが、
周が1の輪っかを転がして [0,∞) に一対一対応させるには、
きっちり転がった回数と半端に転がった角度のそれぞれが必要。
前者は自然数 N で後者は区間 [0,1) となる。
その方法では N上の分布がなければ [0,∞) 上の分布が作れない。

815 名前:803投稿日:03/06/20 20:32
>>814
なるほどなるほど。
俺のやり方じゃまだ他にNの一様分布が必要なんですね・・・




・・・
ところで、[0,1) の一様分布から、Nの一様分布を取り出すことってできないですか?
少数部分をゴニョゴニョ

816 名前:132人目の素数さん投稿日:03/06/20 21:57
>>815
出来ないことは >>787 と同様に示されるから、納得するには
一様分布や長さがどういうものかをよく考えてみるといいかもね。
例えば正規数を手がかりにしたらどうかな。
ttp://www.wikipedia.org/wiki/Normal_number
ttp://mathworld.wolfram.com/NormalNumber.html

817 名前:132人目の素数さん投稿日:03/06/21 02:36
>>813
"オ"で場合分けする。"オ"が3以上なら4桁目で繰り上がらない"ア"を考え
"オ"が2の時は1桁目の"ア"が偶数である事を利用する。
この時1,4桁目の計算を"オ"=2,3,5,6,7,8では上手く合わせられない。
後は繰り上がり等を利用すればaaaa*1=aaaa,2178*4=8712,1089*9=9801しかない事が分かる。

818 名前:132人目の素数さん投稿日:03/06/21 02:37
猿がでたらめにタイプライタを叩いて、 ABRACADABRA とタイプするまでの時間の期待値を求めよ

819 名前:132人目の素数さん投稿日:03/06/21 13:22
>>818
一秒間にどれだけタイプできるんだ?
猿が疲れて打たなくなることも考慮するのか?

820 名前:ぼるじょあ ◆yEbBEcuFOU 投稿日:03/06/21 13:28
(・3・) エェー
猿が高齢でタイプ中に氏ぬ確率も考慮するんだYO!

821 名前:132人目の素数さん投稿日:03/06/21 14:51
Backspaceとか半角/全角とか色々押しちゃう確率も考えなくてはいけないわけで。

822 名前:132人目の素数さん投稿日:03/06/21 14:53
Caps Lockがどうなってるのかにも注意しなきゃだわ。

823 名前:132人目の素数さん投稿日:03/06/21 14:57
オナニー覚えちゃったらその瞬間確率0ね!

824 名前:ぼるじょあ ◆yEbBEcuFOU 投稿日:03/06/21 15:28
(・3・) エェー >>820-823おもしろすぎだYO!

825 名前:ぼるじょあ ◆yEbBEcuFOU 投稿日:03/06/21 18:10
nを正の整数、aをnと互いに素な整数、1からnまでの整数のうちnと互いに素となるものの個数をbとするとき、
ab ≡1(mod n)となることを、高校数学の範囲で証明してNE!

826 名前:132人目の素数さん投稿日:03/06/21 20:35
n=1のとき、aをいくつにとっても、1からnまでのnと互いに素となるものの個数b=0


827 名前:ぼるじょあ ◆yEbBEcuFOU 投稿日:03/06/21 21:04
>>825の「互いに素となる」とは、最大公約数=1のことと解釈してNE!

828 名前:サミュエルソン投稿日:03/06/21 21:13
太郎君は赤い玉を3つ持っています。
花子さんは青い玉を5つ持っています。
ただそれだけです。

829 名前:132人目の素数さん投稿日:03/06/21 21:52
ぼくわ金の玉を2つ持っている
と思っていたのに
黒い色ですた

830 名前:132人目の素数さん投稿日:03/06/21 22:20
n=11、a=7とすると
7*10≡4だけど・・・


831 名前:あぼーん投稿日:あぼーん
あぼーん

832 名前:ぼるじょあ ◆yEbBEcuFOU 投稿日:03/06/21 22:27
たびたびスマソ
>>825の「ab」⇒「a^b」としてNE

===かきなおし===
nを正の整数、aをnと互いに素な整数、1からnまでの整数のうちnとの最大公約数=1となるものの個数をbとするとき、
a^b ≡1(mod n)となることを、高校数学の範囲で証明してNE!

833 名前:132人目の素数さん投稿日:03/06/21 22:47
n=1、a=2のときb=1

2^1≡1(mod1) か?

834 名前:132人目の素数さん投稿日:03/06/21 22:49
おもしろいのは ぼるじょあの醜態であって 問題ではないようだ

835 名前:132人目の素数さん投稿日:03/06/21 22:52
>>832
もうお前市ね

836 名前:132人目の素数さん投稿日:03/06/21 23:07
ぼるじょあの偉大なる軌跡 1〜10
ttp://www.geocities.co.jp/Milano-Kotto/1831/index2.html
ぼるじょあの偉大なる軌跡 11〜
ttp://www.geocities.co.jp/Milano-Kotto/1831/

837 名前:132人目の素数さん投稿日:03/06/22 04:46
>>820
>>821
>>822
>>823
キーボードにはA〜Zしかない。
猿は永遠に1秒に1個のキーを打つものとする。

838 名前:132人目の素数さん投稿日:03/06/22 04:58
1/{x^(2n)+1}を部分分数分解してください

839 名前:132人目の素数さん投稿日:03/06/22 08:50
サイコロで1が5連続、といった類のものだが
5文字目と7文字目のパンクの一部がリスタートになるのが特徴的。

840 名前:132人目の素数さん投稿日:03/06/22 10:46
正6面体のサイコロ(普通のサイコロ)を投げて、
1が出る確率は1/6。
2も1/6。

6も1/6。

さて問題。
サイコロを何度も投げて、途中で2,3,4,5,6が順不同で出たとする。
すると次に投げたら1がでるはずだが、なぜ出ないのか?

841 名前:ぼるじょあ ◆yEbBEcuFOU 投稿日:03/06/22 11:38
(・3・) エェー
無限の試行を行なった時にそれぞれの目が出る確率が1/6に収束するんだYO!

842 名前:132人目の素数さん投稿日:03/06/22 11:49
>>840
やってみなよ。でるかもしれないじゃん。

843 名前:132人目の素数さん投稿日:03/06/22 11:49
>>840
この手の問題はいい加減うざいな


844 名前:132人目の素数さん投稿日:03/06/22 12:14
>>837
Aを打つ確率とBを打つ確率が等しいとは思えません

845 名前:132人目の素数さん投稿日:03/06/22 12:54
>>844
確かに。
それぞれが独立であるとも思えません(同じものを何度も打つ確率が高いかもしれない)。


846 名前:132人目の素数さん投稿日:03/06/22 20:39
1億年くらいかな

847 名前:132人目の素数さん投稿日:03/06/23 19:06
おもろい問題おしえんかい!

848 名前:132人目の素数さん投稿日:03/06/23 19:25
↓その問題面白すぎ

849 名前:132人目の素数さん投稿日:03/06/23 20:11
大きさが異なり、互いに交わらない3つの円A,B,Cがあったとする。
2つの円A,Bに接する接線のうち、
A,Bが同じ側にあるような接線は2本引けるが、
これらの交点をPとする。
同様のことをB,Cに対しても行い、この交点をQとする。
またC,Aに対しても同様のことを行い交点をRとする。
P,Q,Rは一直線上に並ぶことを証明せよ。

850 名前:132人目の素数さん投稿日:03/06/23 22:21
>>845
心理学まで必要になるとは。

851 名前:132人目の素数さん投稿日:03/06/23 23:56
ひょっとして落ち?

852 名前:132人目の素数さん投稿日:03/06/23 23:58
気のせいでした、スマソ・・・

853 名前:132人目の素数さん投稿日:03/06/24 01:30
つーか、このスレ、OpenJaneで読めないんですけど

854 名前:132人目の素数さん投稿日:03/06/24 02:43
OpenJane の開発者にいわなきゃ直らない罠

855 名前:132人目の素数さん投稿日:03/06/24 03:04
OpenJaneで読んでいるんですけど。

856 名前:132人目の素数さん投稿日:03/06/24 03:06
>>853
ログ再取得(もしくはそれに類するもの)しなされ。

857 名前:132人目の素数さん投稿日:03/06/24 03:06
>>844
>>845
この問題ではどのキーも同じ確率で猿は打つ。だったはず。


858 名前:132人目の素数さん投稿日:03/06/24 03:15
どっちにしても、一秒間に何回うつか教えてくれなきゃ
解けないわけだけど。

859 名前:132人目の素数さん投稿日:03/06/24 03:18
猿といえばこういう話もある
ttp://www.ietf.org/rfc/rfc2795.txt

860 名前:132人目の素数さん投稿日:03/06/24 03:42
すみません、友達に
1/7で当たり、6/7でハズレのクジがある
3回連続で当たりを引く確率は?

A1/343

これが間違いという事が理解してくれませんでした。
なんて説明したらいいのか、誰か教えてぽ。
俺自身が糞馬鹿なのでよくわからなくなってしまいました

861 名前:132人目の素数さん投稿日:03/06/24 03:47
何回続けて引いても「1/7で当たり、6/7でハズレ」なら
3回連続で当たりを引く確率は1/343で良いのでは?

862 名前:132人目の素数さん投稿日:03/06/24 03:53
わかりました。
俺がアフォやってん。サンクスコ

863 名前:132人目の素数さん投稿日:03/06/24 03:56
お気に入り集 ☆
http://pleasant.free-city.net/


864 名前:132人目の素数さん投稿日:03/06/24 04:55
>>858
1秒に1回で考えてください。

865 名前:132人目の素数さん投稿日:03/06/24 12:25
>>860 >>861
3回連続で当たりを引く確率は1/343だが
「3回連続で当たりを引くまでの平均試行回数は?」という
問題になると343より大きくなる。
昔、パチスロ機「宝船」で話題になった問題。

866 名前:860投稿日:03/06/24 14:11
>>865
そうです、宝船の問題なんです。
もう少し詳しく教えてくれませんか?
試行回数と確率では答えは違うんですね。

867 名前:132人目の素数さん投稿日:03/06/24 15:38
当選確率が1でも、平均試行回数が3になるように
確率の分母<平均試行回数となる。

ただし、当たり(リプレイ)を試行回数に数えなければ
(3回連続当たりの場合は1回と数える)
確率の分母=平均試行回数となる。

平均試行回数の計算は多分難しい。
プログラムが組めるならシミュレータでの計算をお勧めする。


868 名前:132人目の素数さん投稿日:03/06/24 19:51
>>865-867
平均試行回数を E とする。
(平均試行回数が存在することは証明すべきだが、ここでは略)

開始後3連続で当たりを引いた場合、試行回数は 3
開始後2連続で当たり、次にはずれた場合、試行回数の期待値は E+3
開始後1回当たり、次にはずれた場合、試行回数の期待値は E+2
開始後1回はずれた場合、試行回数の期待値は E+1
なので、

E = 3 * 1/343 + ( E + 3 ) * 6/343 + ( E + 2 ) * 6/49 + ( E + 1 ) * 6/7

を解けばよい。

869 名前:132人目の素数さん投稿日:03/06/24 21:51
>>868
そーいえばそんなのあったな。久しぶりに見た

870 名前:860投稿日:03/06/25 01:56
なるほど、皆さんアリガトウございまふた

871 名前:132人目の素数さん投稿日:03/06/25 17:13
>>859
めんどくさいので日本語訳を探して読んだ。ワロタ
ttp://www.chibutsu.org/iorin/rfc/rfc2795.txt

別の数学っぽいネタ
ttp://www.chibutsu.org/iorin/rfc/rfc3091.txt

日本語訳リンク集
ttp://www.imasy.or.jp/~yotti/rfc-joke.html

872 名前:849投稿日:03/06/25 20:23
スルーされちゃったんですけど、
既出or問題がおかしかったでしょうか。

873 名前:132人目の素数さん投稿日:03/06/25 20:26
「あー解けない。ちょっと待ってろ」ってレスを一々するのは恥ずかしいんじゃ、それぐらい察しれ

874 名前:132人目の素数さん投稿日:03/06/25 21:42
ジサクジエンハケーン!!

875 名前:132人目の素数さん投稿日:03/06/25 22:04
>>874 ハァ?下らない等号ネタ振ってんじゃねぇよクズ。死ね。

876 名前:132人目の素数さん投稿日:03/06/27 15:56
濃度C(%)の食塩水M[0](g)がある。この食塩水を次の操作を何回か行って薄める。

(操作)
食塩水に純水を加えてよくかき混ぜたのちに加えた純水と同じ分量の食塩水を捨てる。

加えられる純水の総量がV(g)であるとき、この食塩水は何(%)まで薄められるか。理論値で答えよ。
ただし、最初の食塩水に食塩の沈澱はなく、操作はすべて同じ条件(気温、気圧など)で行うものとする。。

877 名前:mathmania ◆uvIGneQQBs 投稿日:03/06/27 16:02
Re:>876
濃度は限りなく0に近づけられると思うが。

878 名前:132人目の素数さん投稿日:03/06/27 16:27
>>877
加えられる純水の総量V(g)は定数ですが。

879 名前:132人目の素数さん投稿日:03/06/27 16:34
>>876
全部でVの食塩水が捨てられるわけか。
できるだけ濃いのを捨ててった方が得だから、ちょっと入れては捨てての繰り返しだな。
こいつは積分ですな。

計算はめんどいのでパス
あとよろしく


880 名前:132人目の素数さん投稿日:03/06/27 16:57
     _____
     ヾ;::l_~''ヽ
         ~ヽヽ
       /|  ) \|\
       |:::::|/   /|:::::|
       ヽ/ ̄ ̄ ̄ヽ ノ
       /      ヽ     __,,,,,
    / ヽ(・))    ((・)'i   l:::::/
   ノノヽ_ i ___ --- ___i  l _ ノ丿
  (_(_    ~l:::::.   .:::::lヾ - ~~~"
   |;;/    .l::::::. .:::::l
          (::o::..::o::)
           ヾ;;;;;;;/
     _____
     ヾ;::l_~''ヽ
         ~ヽヽ
       /|  ) \|\
       |:::::|/   /|:::::|
       ヽ/ ̄ ̄ ̄ヽ ノ
       /      ヽ     __,,,,,
    / ヽ(・))    ((・)'i   l:::::/
   ノノヽ_ i ___ --- ___i  l _ ノ丿
  (_(_    ~l:::::.   .:::::lヾ - ~~~"
   |;;/    .l::::::. .:::::l
          (::o::..::o::)
           ヾ;;;;;;;/


881 名前:mathmania ◆uvIGneQQBs 投稿日:03/06/27 17:51
Re:>876
食塩水はM[g]で、最初の食塩はCM/100[g]で、W[g]の純水を入れて攪拌してW[g]の食塩水を捨てるとき、
純水を入れたときの濃度はCM/(M+W)%となる。
つまりW[g]の純水を加えて攪拌してW[g]の食塩水を捨てると濃度はM/(M+W)倍される。
このWをV/nにして、操作回数をnにすると、食塩の濃度は
(M/(M+V/n))^n倍される。
この式を整理すると、
(M/(M+V/n))^n=(1/((M+V/n)/M))^n=(1/(1+V/M/n))^n=1/(1+(V/m)/n)^nとなる。
これをn→∞とすると、exp(-V/M)となる。
よって、食塩水はCexp(-V/M)%まで薄められる。
(注意:これは厳密な解答ではない。(答えの数値は正しい。))

882 名前:132人目の素数さん投稿日:03/06/28 01:04
面白いか面白くないかは人によると思いますが(僕はこういうの好きなんですが)

足してnになる1以上の整数の組み合わせを並べる時、並べ方が最大になるような
整数の組み合わせをnを使って表せるんでしょうか?

わかりにくいので例を挙げますと n=5 のとき
足して5になる1以上の整数の組み合わせは5, 41, 32, 311, 221, 2111, 11111 であり
並べ方が何通りあるかはそれぞれ 1, 2, 2, 3, 3, 4, 1 で
最大値は 2111 の場合で 4 に、 n=6 のとき 最大値は 321 と 2211 の場合で 6 になります。
一般に足してnになるような並べ方の総数は、数字の個数(311なら3個、11111なら5個)がm個、
整数の組み合わせの中に 1がk_1個、2がk_2個、…、nがk_n個 ある場合は
 m!/(k_1!・k_2!・・・k_n!) になります。
明らかに、最大値を取る時 k_n=0 であり、
恐らく n≧8 では k_j=0 (j≧[n/2]) になるとか制限はつけれますが
最大を取る時の k_1,k_2,…を n で表すことは無理でしょうか?

883 名前:132人目の素数さん投稿日:03/06/28 03:20
ムズカシーネー

884 名前:882投稿日:03/06/28 10:52
Mathematicaスレでnが任意の自然数の時
並べ方が最大である自然数の組み合わせを求める
プログラムは教えていただきました。

885 名前:132人目の素数さん投稿日:03/06/28 11:26
なるべく違う数字を使えばいい(はず)だから、
n-1-2-3-・・・-k≧0なる最大のkを求めて
n-1-2-3-・・・-k-1-2-・・・-m≧0なる最大の以下略

とやればいいだろうけど、数式でどうやって表すのやら・・・。

886 名前:132人目の素数さん投稿日:03/06/28 11:27
プログラムならできそうだけどね

あ、漏れは最大である証明はできないよw

887 名前:132人目の素数さん投稿日:03/06/28 11:32
訂正
×≧
○>

888 名前: ◆BhMath2chk 投稿日:03/06/28 11:40
>>876
同じ分量というのが同じ重さということなら
ag加えたとき濃度はM/(M+a)倍される。
log(M/(M+a))=−log(1+a/M)>−a/Mなので
log(Π(M/(M+a)))>−Σa/M=−V/M。
Π(M/(M+a))>exp(−V/M)。


889 名前:132人目の素数さん投稿日:03/06/28 23:23
1/(3^n) の循環節の長さを求めよ。
例えばn=3のとき、1/27=0.037037037… だから循環節の長さは3となる。

890 名前:132人目の素数さん投稿日:03/06/28 23:33
>>889
n=1のときは循環節の長さは1でいいのか?


891 名前:132人目の素数さん投稿日:03/06/28 23:44
n=1のとき 1 0.333333・・・
n=2のとき 1 0.111111・・・
n=3のとき 3 0.037037・・・
n=4のとき 9 0.012345679012345679・・・
n=5のとき 27 0.004115226337448559670781893004115226337448559670781893・・・

3≧nのときの循環小数の数は3^(n-2)となってるかも。
これ以上は調べるのマンドクサ
合ってるかどうか誰か検証キボン

892 名前:891投稿日:03/06/28 23:50
あ、2≧nでも正しいのか・・・

893 名前: ◆BhMath2chk 投稿日:03/06/29 00:00
>>882
最大になるときのsの個数をk(s)とすると
2<nのとき任意のsに対してk(s)≧k(s+1)。


894 名前:891投稿日:03/06/29 00:01
ひまなんで1/(2^n)を考えてみた

n=1のとき (桁数が)1 0.5
n=2のとき 2 0.25
n=3のとき 3 0.125
n=4のとき 4 0.0625
n=5のとき 5 0.03125

よってy=xの直線になる・・・

895 名前:891投稿日:03/06/29 00:05
1/(4^n)

n=1のとき (桁数が)2 0.25
n=2のとき 4 0.00625
n=3のとき 6 0.015625
n=4のとき 8 0.00390625
n=5のとき 10 0.009765625

よって桁数をyとするとy=2n・・・


896 名前:891投稿日:03/06/29 00:10
1/(5^n)

n=1のとき (桁数が)1 0.2
n=2のとき 2 0.04
n=3のとき 3 0.008
n=4のとき 4 0.0016
n=5のとき 5 0.00032

よって桁数をyとするとy=n・・・

897 名前:132人目の素数さん投稿日:03/06/29 00:27
一般のnで求めてもらいたい。

898 名前:132人目の素数さん投稿日:03/06/29 00:39
>>891
以下の命題を証明すれば充分である。
 0以上の整数 n について、
 10^(3^n) - 1 は 3^(n+2) で割り切れるが、
 3^(n+3) では割り切れない。

命題の証明は
(10^(3^(n+1))-1) / (10^(3^n)-1)
= 10^(2・3^n) + 10^(3^n) + 1
≡ 3 (mod 9)
を用いて、 n についての帰納法。

899 名前:132人目の素数さん投稿日:03/06/29 02:17
>>721
2n(n+1)=m(m+1)
を満たす自然数m.nを求めればよい。

900 名前:132人目の素数さん投稿日:03/06/29 09:51
余裕で900get

901 名前:数学が得意な皆様に質問です。投稿日:03/06/29 10:23
初めまして。現在大学受験を控えている高校三年生の男子です。
僕は数学が凄く苦手で、今の実力では大学合格は難しいと先生にも
言われています。そこで、数学が得意な皆様に質問させて頂きます。
皆さんは数学をどのように勉強しましたか?勉強のコツなど、教えて下さい。


902 名前:132人目の素数さん投稿日:03/06/29 10:35
>>901
やだ

903 名前:数学が得意な皆様に質問です。投稿日:03/06/29 10:50
氏ね

904 名前:132人目の素数さん投稿日:03/06/29 10:54
マルチ氏ね
http://science.2ch.net/test/read.cgi/math/1056800354/42
http://science.2ch.net/test/read.cgi/math/1045496321/31
http://science.2ch.net/test/read.cgi/math/1045714866/58
http://science.2ch.net/test/read.cgi/math/1049652059/142
http://science.2ch.net/test/read.cgi/math/1042776592/111
http://science.2ch.net/test/read.cgi/math/1048445371/197

905 名前:132人目の素数さん投稿日:03/06/29 11:14
某スレッドで面白い問題を発見しました。

 f(x)が凸関数、つまり
  ∀a, b∈R:0<∀t<1:f(ta+(1-t)b)≦tf(a)+(1-t)f(b)
 のとき、fは連続だそうですが、直感的には判るものの証明が思いつきません。
 どう証明するのか、教えてください。
 お願いします。

誰も解けていないようなのでよろしくお願いします。

906 名前:132人目の素数さん投稿日:03/06/29 11:14
>>905
ツマラン

907 名前:supermathmania ◆ViEu89Okng 投稿日:03/06/29 11:37
Re::>905
それは、テキストにも載っているようなことだ。
a<x<c<bに対して、f(x)=f((c-x)a/(c-a)+(x-a)c/(c-a))<=(c-x)f(a)/(c-a)+(x-a)f(c)/(c-a)
f(c)<=(b-c)f(x)/(b-x)+(c-x)f(b)/(b-x)よりf(x)>=(b-x)f(c)/(b-c)-(c-x)f(b)/(b-c)
a→x-0と、c→x+0を考えると、fは点xにおいて連続であることがわかる。
ずっと昔に解かれていることだ。

908 名前:132人目の素数さん投稿日:03/06/29 13:41
>>849
鉄塔3本の問題の変形だな。

3つの円を、同一平面上に中心を持つ3つの球の断面だと考える。
すると題意の接線は、2つの球を包み込むようにして得られる
円錐の断面と考えることができる。

3つの円錐の頂点は当然、問題の平面上にあり、なおかつ
3つの球に接する平面上にもある‥‥

909 名前:407投稿日:03/06/29 15:15
いちおう亀レスだけど、>>905
http://science.2ch.net/test/read.cgi/math/1056800354/407
を荒らしが嫌がらせでコピペしたもので、マルチではありません。
新手の荒らしですが、大変悪質ですね。
もう・・・尻拭いが大変。

910 名前:132人目の素数さん投稿日:03/06/29 15:22
>>909
うざい死ね

911 名前:849投稿日:03/06/29 15:30
>>908
正解。

912 名前:132人目の素数さん投稿日:03/06/29 15:33
>>910=407が解けなくて荒らしていたヴァカ

913 名前:132人目の素数さん投稿日:03/06/29 20:42
>>882
未解決問題じゃ無いの?

914 名前:882投稿日:03/06/29 23:31
ちなみに
n=5のとき、最大値を取る自然数の組み合わせは 2111であり最大値は4
n=6のときは 321, 2211 であり最大値は6
n=7のときは 3211  であり最大値は12
n=8のときは 32111 であり最大値は20
n=9のときは 32211, 321111であり最大値は30
n=10のときは 322111 であり最大値は60
n=11のときは 3221111 であり最大値は105
n=12のときは 32211111 であり最大値は168
n=13のときは 32221111 であり最大値は280
n=14のときは 322211111 であり最大値は504
n=15のときは 43221111, 3222111111 であり最大値は840
になるはずです。 

915 名前:132人目の素数さん投稿日:03/06/30 05:38
>>914
以前ここで、同じか似たような問題を見た記憶があるのだが
今ちょっと見ても、どのページだったか見つけられなかった

http://www.junko-k.com/index.html


916 名前:132人目の素数さん投稿日:03/06/30 11:18
8|1|6
――――
3|5|7
――――
4|9|2
上図のような縦横斜の数字の和が同数になる物を三方陣という。
では、上下可逆数字「0、1、6、8、9」の5つを組み合わせた以下の2桁の25個の数字を使い、
上下可逆の五方陣を作成せよ。
「00,01,06,08,09,10,11,16,18,19,60,61,66,68,69,80,81,86,88,89,90,91,96,98,99」

917 名前:supermathmania ◆ViEu89Okng 投稿日:03/06/30 12:45
[916]の略解:
縦横斜め列それぞれに1の位と10の位に1度ずつ0,1,6,8,9があらわれるように配置すればよい。

918 名前:132人目の素数さん投稿日:03/06/30 13:17
00|81|16|98|69
18|99|60|01|86
61|06|88|19|90
89|10|91|66|08
96|68|09|80|11
何通りの解が出来る?

919 名前:_投稿日:03/06/30 13:20
http://homepage.mac.com/hiroyuki44/jaz04.html

920 名前:132人目の素数さん投稿日:03/06/30 17:08
test

921 名前:132人目の素数さん投稿日:03/06/30 19:57
Testはご遠慮ください

922 名前:132人目の素数さん投稿日:03/07/01 10:35
testってなに?期末?

923 名前:_投稿日:03/07/01 10:50
http://homepage.mac.com/hiroyuki44/hankaku10.html

924 名前:132人目の素数さん投稿日:03/07/02 04:32
野球でm対n(ただしm>=n)で勝負がついたとき、
一度も同点がない「つまらない」ゲームであった確率はいくらか。

925 名前:132人目の素数さん投稿日:03/07/02 04:39
原点を通る傾き正の直線Lを考える
今 点Pが原点から直線 L に沿って第一象限を直進し、
直線Lと曲線C:y=x^3/√3の交点Q(a、a^3/√3)
で反射した後再び直進する ただし点QにおいてCの接線に対し
入射角と反射角は等しいとする
反射後の点Pの進行方向がy軸と平行になる時つぎの問いに答えよ

1)aの値を求めよ
2)直線Lと曲線Cが第一象限で囲む範囲の図形の面積を答えよ


これを解いてみろYO

926 名前:132人目の素数さん投稿日:03/07/02 05:09
qをせいの実数とし

f(z)=(z+1-q)/(z+1)

とおく

f1(z)=f(z)と書き 以下n=2,3,4・・・・に対して
f2(z)=f(f1(z))、f3(z)=f(f2(z))、・・・・・・、fn(z)=f(fn-1(z))、・・・・・・とおく
このときfi(z) ( i =1,2,・・・,8)が意味をもつすべての数zにたいしてf8(z)=f4(z)
が成立するような q の値をもとめよ



927 名前:132人目の素数さん投稿日:03/07/02 09:14
>>924

0じゃないの。
開始時は0対0で同点だよ。
「つまらない」に当てはまるかは知らんけど

928 名前:132人目の素数さん投稿日:03/07/02 18:26
>>924
逆転する時は必ず2点以上取らないといけないのか?

929 名前:_投稿日:03/07/02 18:29
http://homepage.mac.com/hiroyuki44/

930 名前:132人目の素数さん投稿日:03/07/03 04:50
                       ,____
                     r' ~   ~|,rーー、-、
    ,ー-、-‐、 _,_,_,, ,、;'' ''~,゛';' "~'゛ヽ 、  /`i    i
   ,/^\   v'{   ヽ;;;z' ,'; 、 ,   、z ヽレ'{  " |,   |
  /    \ レ/ |    |;;;;;~,,;; ,、、;  ,,` , 〈 ,i ,r'|    |
  /,     /;V ,r'|    lr';;r'   ゛ "   `、ヽ=ト/   /
 /_,    ,};{=}《 }   /r'  ," ,i'  ! `、`i、 ヽ ゛ヽ/  ,,イ ̄l
  \   /{  ヽ/  ,r ; :r';r; ,'! i!  i|`i !i i,ヽ )})、/  `ヽト、
   ,\ノ__/\,/  l;r' !;|i_l;|-i| !   i, |i‐|_リ=リ、レ';'ヽ   /
 /    /゛  ゙ '-,_(i |;!=_リニヽ、  ,!'' r"q`、i'  ,!;; \,/
 ゙ ' rz,_/゛    |;r'  ,;r'",Oi`     ,{:;,::j,. i  !;; ,!;;' ,!
    i;/     ,|:ヽ,  、'、;;,:;},.       ̄ ,.iヽ  ,i!; ;;'!  
>>1
   i '-ー";;\、__,|、ヾ,   ̄    _゛   , '!;;;)、';;; !冫     。
   ゝ、;_ヾー ノ   ;;/ 、_      '‐'  , -´ ,r''  ゛ー"   ー┼‐  ___ ヽ
      ゛~'~    r'  ,゙、i ‐-- 、. -i ''  , -、          ノ | ヽ       _,ノ
            ,. - !ー-`= 、 ,、!)‐"  |、__

931 名前:132人目の素数さん 投稿日:03/07/03 06:25
このスレの問題を問題-解答形式にまとめたHPとかって
誰か作ってくれないかな・・・。

他人任せでスマソ・・

932 名前:_投稿日:03/07/03 06:26
http://homepage.mac.com/hiroyuki44/jaz09.html

933 名前:132人目の素数さん投稿日:03/07/03 06:53
問題だけ(全部じゃないけど)を集めてるスレならあるけどさ。

934 名前:132人目の素数さん投稿日:03/07/03 08:21
・゚・(ノД`)・゚・。
ttp://pink.sakura.ne.jp/~erotan/

935 名前:132人目の素数さん投稿日:03/07/03 12:51
二つの整数 p , q、1<p,q < 100がある。
P氏はその二つの数の積を知っている。
S氏はその二つの数の和を知っている。
P氏はS氏に「私はp,qがいくつか分かりません」と言いました。
するとS氏は言いました。
「あなたがp,qがいくつかわからないことは分かっていました。
そして私にもp,qがいくつかわかりません」

P氏は言いました、「感謝する。今ので私はp,qがいくつか分かりました」と。

さて、p,qはいくつでしょう。


936 名前:132人目の素数さん投稿日:03/07/03 15:02
>>935
p,qは順不同だよな?

おかしいな。俺の勘違いかな。

(1,4)でも(1,8)でも上の会話は成り立つぞ?

937 名前:132人目の素数さん投稿日:03/07/03 15:09
1<p,q

938 名前:132人目の素数さん投稿日:03/07/03 15:11
>925
a=1
>935
p,qが互いに素だったらP氏の最初のセリフに反する。
S氏のセリフ1行目を解釈すると「自分の手がかりから考えられる積全てで
つねにP氏が答えをしぼりこめないはずだ」
ここでp+q>=55であったら組み合わせの中に50以上の素数である53が現れ
(p+q>=99だったら97が出るしp+q>=197なら97は出ないけど問題ないし)
P氏が答えをしぼりこめる可能性が出てくるので不可。
4以上の偶数は常に2つの素数の和で表せるのでこれも不可。
あと素数+2も不可。
p+q=11,17,23,27,29,35,37,41,47,51,53

p*q=9*2=18だとしたら
(6,3) 6+3=9 範囲外
(9,2) 9+2=11 唯一の正解
p*q=15*2=30だとしたら
(6,5) 6+5=11 正解
(10,3) 10+3=13 一致せず
(15,2) 15+2=17 正解
絞り込めず

p*q=27*26=702だとしたら
(27,26) 27+26=53 唯一の正解
(39,18) 39+18=57 範囲外
p*q=27*24=648だとしたら
(27,24) 27+24=51 唯一の正解
(36,18) 36+18=54 範囲外

まだありそうだ。何か間違えたのか。

939 名前:132人目の素数さん投稿日:03/07/03 15:50
ttp://science.2ch.net/test/read.cgi/math/1020334993/79

940 名前:938投稿日:03/07/03 16:27
ああ、51は34*17が他の分解の仕方が存在しないのね。
考えつくすのが難しい。

941 名前:132人目の素数さん投稿日:03/07/03 22:14
ちょっとパズルっぽいけど。

二次元の格子点上に9コの点を次の条件を満たすように配置せよ。
3点だけを通る直線が9本存在する。

942 名前:132人目の素数さん投稿日:03/07/03 22:23
(0,0) (0,2) (0,4)
(2,1) (2,2) (2,3)
(4,0) (4,2) (4,4)
でどうかな?

943 名前:132人目の素数さん投稿日:03/07/03 22:24
はやいっすね。

944 名前:942投稿日:03/07/03 22:25
だめだった、10本引ける

945 名前:942投稿日:03/07/03 22:31
(0,0) (0,2) (0,4)
(2,0) (2,2) (2,3)
(4,0) (4,2) (4,4)
でどうよ

946 名前:132人目の素数さん投稿日:03/07/03 22:58
9本が解かれたら次10本って言おうと思ってたんだけどさ。

947 名前:132人目の素数さん投稿日:03/07/04 00:21
1<p,q < 100
だよお

948 名前:132人目の素数さん投稿日:03/07/04 00:32
三つの自然数a,b,cがある。
abをcで割ったあまりが1
bcをaで割ったあまりが1
caをbで割ったあまりが1
の時、a,b,cを求めよ。

949 名前:132人目の素数さん投稿日:03/07/04 00:55
>>947
キモイ!

950 名前:132人目の素数さん投稿日:03/07/04 00:58
>>948
分からん!

951 名前:sage投稿日:03/07/04 01:09
a=2 b=3 c=5 って実際はまじで答えてはいけない問題だったりして

952 名前:132人目の素数さん投稿日:03/07/04 01:10
3,6,7

953 名前:132人目の素数さん投稿日:03/07/04 01:16
>>951
一般型を求めよという問題だろうな。
つっても、漏れもその1例しか思いつかんが。

954 名前:132人目の素数さん投稿日:03/07/04 01:47
abc|(bc-1)(ca-1)(ab-1) ⇒ abc|(ab+bc+ca-1) ...

955 名前:132人目の素数さん投稿日:03/07/05 00:25
>>952
ちがうだろ!

956 名前:132人目の素数さん投稿日:03/07/05 01:40
>>952
3,3,4。
16,21,45。


957 名前:132人目の素数さん投稿日:03/07/05 05:11
>>948
bc-1がaで割り切れ、ca-1がbで割り切れ、ab-1がcで割り切れるため
(bc-1)(ca-1)(ab-1)はabcで割り切れる。

(bc-1)(ca-1)(ab-1)
=ab + ac + bc - 1 - abc(a + b + c + abc)
なので、
ab + ac + bc - 1がabcで割り切れることになる。
よって、nを自然数として
ab + ac + bc - 1 = nabc
abcで割って、これ整理すると
1/a + 1/b + 1/c>1/a + 1/b + 1/c - 1/abc = n
また、a,b,cは自然数なので
1/a + 1/b + 1/c≦3
よって、n<3が成立する。従ってn=1,2のどちらか。

ab + ac + bc - 1 = abc の時。
ab + ac + bc > abc
1/a + 1/b + 1/c > 1、a≦b≦cとすれば、3/a > 1なので、a=1,2のどちらかが成立する。
a=1の時、b+c+bc-1=bcなので、b+c=1、1≦b≦cなので条件を満たさない。
a=2の時、1/b + 1/c > 1/2なので、2/b>1/2より、b<4。よって、a=2≦b<4より、b=2,3。
b=2の時。4+2c+2c-1=4c。3=0。これは矛盾。
b=3の時、c=5となり、条件を満たす。

ab + ac + bc - 1 = 2abc の時。
ab + ac + bc > 2abcなので、1/a + 1/b + 1/c > 2。3/a>2より、a=1が成立する。
よって、b+c-1=bc。これを解くと、b=1、cは任意。ところがこの場合
bcをaで割ったあまりは1にならず、条件を満たさない。

結局題意を満たす物は
a=2,b=3,c=5の組み合わせのみ。

958 名前:952投稿日:03/07/05 09:03
ab=a*bのことなのか。てっきり、ab=10*a+bのことだと思った。
>>956
なるほど。

959 名前:132人目の素数さん投稿日:03/07/05 09:05
957は頭いいね!中学の教師にこういう人がいたらな。

960 名前:132人目の素数さん投稿日:03/07/05 10:27
>>957

    ∩
    ( ⌒)     ∩_ _
   /,. ノ      i .,,E)
  ./ /"      / /" .
  ./ / _、_   / ノ'
 / / ,_ノ` )/ /
(       /  good job!
 ヽ     |
  \    \

961 名前:957投稿日:03/07/05 11:39
っていうか、最初からa,b,c>1の条件を追加していれば
もっと簡単にかけたのに・・・馬鹿だなオレ。鬱だ。

さらに
> (bc-1)(ca-1)(ab-1)
> =ab + ac + bc - 1 - abc(a + b + c + abc)
の展開は間違ってるし。。。正しくは
(bc-1)(ca-1)(ab-1) = ab + ac + bc - 1 - abc(a + b + c - abc)
だろうよ。。。

さらに、
> ab + ac + bc - 1がabcで割り切れることになる。
> よって、nを自然数として
> ab + ac + bc - 1 = nabc
は飛躍してるし。。。(少しだけど)

より厳密にやるなら、
『ab + ac + bc - 1がabcで割り切れることになる。 よって、nを整数として
ab + ac + bc - 1 = nabc
ところが、a,b,cは自然数なので
ab + ac + bc - 1>0  かつ  abc>0
よって、n>0であり、自然数となる』

(bc-1)(ca-1)(ab-1) = ab + ac + bc - 1 - abc(a + b + c - abc)
なんだから、 (bc-1)(ca-1)(ab-1) = Nabcと書けば、Nは自然数。
ab + ac + bc - 1
=(bc-1)(ca-1)(ab-1) + abc(a + b + c - abc)
=(N+a+b+c-abc)abc
であり、n=N+a+b+c-abcだが、これが正の数になるという保証はない。
一応、証明しなくちゃいけないので、『』内のような証明が正しいはず。というわけで、>>957は激しく間違い。
氏んでくる。

962 名前:132人目の素数さん投稿日:03/07/05 12:58
>>957
あんた凄い。

963 名前:132人目の素数さん投稿日:03/07/05 22:18
>>957
よくやった。

964 名前:132人目の素数さん投稿日:03/07/06 00:15
>>957
優勝

965 名前:132人目の素数さん投稿日:03/07/06 21:29
4以上の全ての偶数は2つの素数で表すことができる。
この二つの素数を座標にして点を取ると、全ての点は
y=x+2nの一次方程式の上にある事を発見しますた。

なんでか証明できる方いますか?

966 名前:132人目の素数さん投稿日:03/07/06 23:03
nって何?
x+yもy-xも偶数になるのはすごく自明の話なので

967 名前:132人目の素数さん投稿日:03/07/06 23:20
ゴールドバッハの予想?

968 名前:132人目の素数さん投稿日:03/07/07 04:29
>>965
世紀の大発見なので、証明を書いて日本数学会にいますぐ郵送・・・

とかしないで、もっと、勉強しようよ。

969 名前:132人目の素数さん投稿日:03/07/07 04:41
AかBになりきってディベートしてください。


A「もしも俺が鳥ならば月まで飛んでいける(゚∀゚)アヒャ」

B「鳥は月まで行けねーよ( ´,_ゝ`)プッ」


次、Aの番↓

970 名前:_投稿日:03/07/07 05:06
http://homepage.mac.com/hiroyuki44/2ch.html

971 名前:132人目の素数さん投稿日:03/07/07 06:03
A「鳥は月まで行けないが、俺は鳥ではないからな」

972 名前:132人目の素数さん投稿日:03/07/07 06:15
>>971
B「(゚Д゚)ハァ? 今はお前が鳥だという前提で話をしてんじゃねーか( ´,_ゝ`)プッ」

973 名前:132人目の素数さん投稿日:03/07/07 06:37
>>972
A「つまり、ロケットに乗ればって事だよ。
宇宙飛行士を鳥に喩えてるだけ。
鳥という言葉に囚われすぎ。
そのくらい気付いてほしい。
テレシコワだって私はカモメって言ってるだろ?
>>971の意味は(本物の鳥は月まで飛べないが、俺は本物の鳥じゃない。だから宇宙飛行士になれる)って意味。
わかる?」

974 名前:132人目の素数さん投稿日:03/07/07 08:00
これならどうか

A「もしも俺が鳥ならば宇宙の果てまで飛んでいける(゚∀゚)アヒャ」

B「鳥は宇宙の果てまで行けねーよ( ´,_ゝ`)プッ」


975 名前:132人目の素数さん投稿日:03/07/07 08:13
A「鳥は宇宙の果てまで行けないが、俺は鳥ではないからな」

976 名前:132人目の素数さん投稿日:03/07/07 08:14
B「(゚Д゚)ハァ? 今はお前が鳥だという前提で話をしてんじゃねーか( ´,_ゝ`)プッ」

977 名前:supermathmania ◆ViEu89Okng 投稿日:03/07/07 08:18
それ以前に、宇宙の果てとは何だ?
宇宙に果てがあるのなら、その先には何があるのか?

978 名前:背後から投稿日:03/07/07 08:39
いけないなぁ…
ネタにマジレスするのは

979 名前:132人目の素数さん投稿日:03/07/07 08:43
>>952
ちょっと感動。
こういう問題って、怒られたら勉強するの対偶みたいに、有名問題になりそうなのに。
初めて聞いた。

980 名前:132人目の素数さん投稿日:03/07/07 10:34
仮定が偽ならなんでもありw

981 名前:132人目の素数さん投稿日:03/07/07 11:55
カモメ(Chayka)はテレシコワでなくて Vostok 6 のコールサイン

982 名前:132人目の素数さん投稿日:03/07/07 12:16
つーか、>>979は何に感動したんだ?

983 名前:次スレ投稿日:03/07/07 13:21
面白い問題おしえて〜な 六問目
http://science.2ch.net/test/read.cgi/math/1057551605/l50

984 名前:132人目の素数さん投稿日:03/07/07 15:39
ちょっと宣伝。

yahoo ( in English ) watcher
http://science.2ch.net/test/read.cgi/math/1020334993/

985 名前:132人目の素数さん投稿日:03/07/07 17:49
>>977
宇宙が誕生したのが約150億年前なので、
150億光年以上向こうは見ることができないのれす。
そこがうちうの果てなのれす

986 名前:132人目の素数さん投稿日:03/07/07 19:05
>>966
ん、どういうことですか?
スマソ・・・教えてください。

987 名前:132人目の素数さん投稿日:03/07/08 00:52
>>986
4以上の全ての偶数は2つの奇素数の和で表すことができる。

というのはゴールドバッハ予想といわれ未だ未解決。


988 名前:132人目の素数さん投稿日:03/07/08 00:55
>>986
6=3+3
8=5+3
10=5+5
12=5+7
なんだけどnってなに。

989 名前:986投稿日:03/07/08 01:02
>>987
なるほど。
つまりそれが正しければ
>>965のは至極当然ということですか?

ただ、もし証明したならゴールドバッハ予想が予想では無くなる、との見解であってるんでしょうか?

990 名前:132人目の素数さん投稿日:03/07/08 01:10
x−y=(x+y)−2yだからx+yが偶数ならx−yは偶数。


991 名前:132人目の素数さん投稿日:03/07/08 02:34
>>989
そうです。あっています。

もし、ゴールドバッハの予想を肯定的にでも、否定的にでもいいので

証明したら、貴方の名前は後世に伝えられるでしょう。

992 名前:986投稿日:03/07/08 21:30
>>991
ありがとうございました。
今までのをまとめると、私は4以上の偶数は2つの素数で表せる、というのは予想だとは知らなかったんですよ。
それを知らずに偶数はどの二つの素数で表せるかを座標として点に取り、じーっと見ていたら
その座標はy=x+2n nは自然数 のグラフ上にあることに気付いたんですよ。
こんな具合に。
http://www.geocities.co.jp/SiliconValley-PaloAlto/4702/pic/PrimeNumber.jpg
で、全ての点がy=x+2nのグラフ上にあることを証明したら、ゴールドバッハ予想は予想では無くなるのでは?と。
工房なんでこのスレの住人に比べたら無知です。

素数に興味を持ったのでレポートを書いてたときに気付きました。

993 名前:132人目の素数さん投稿日:03/07/09 01:19
素数とか関係なく、(奇数,奇数)および(偶数,偶数) の格子点は全て
その線の上に乗るということが、グラフを見てればわかるだろ。

だから(奇素数,奇素数)が乗るのは当然であり、
残念ながらそれを示しても予想には何の関係もない。

しかしそうやって、自分で疑問を探し出して考える姿勢は大切だよ。
いくらテストの点が良くても、教科書に書かれていることを受動的に
飲み込むことしかできない者は、数学に、否、学問に向いていない。

994 名前:132人目の素数さん投稿日:03/07/09 03:18


995 名前:132人目の素数さん投稿日:03/07/09 03:18


996 名前:132人目の素数さん投稿日:03/07/09 03:18


997 名前:132人目の素数さん投稿日:03/07/09 03:18


998 名前:132人目の素数さん投稿日:03/07/09 07:43
ランダムで一つの平仮名を書き込むとして
994-997のようになる確立を1000までに出して。

999 名前:132人目の素数さん投稿日:03/07/09 07:56
999。


1000 名前:132人目の素数さん投稿日:03/07/09 08:00
1000。


1001 名前:1001投稿日:Over 1000 Thread
このスレッドは1000を超えました。
もう書けないので、新しいスレッドを立ててくださいです。。。


DAT2HTML 0.26 Converted.